Con Law - Barbri

Lakukan tugas rumah & ujian kamu dengan baik sekarang menggunakan Quizwiz!

Scientific studies indicate that emissions from gasoline combustion engines contribute to a reduction in the health and growth rate of trees. To protect trees in the national forests, the Environmental Protection Agency ("EPA"), a federal agency, enacted stringent new fuel emission standards that apply to motor vehicles operating inside or within a 10-mile radius of all federal parks. The EPA regulations expressly preempt any conflicting state or local regulations, unless the state or local regulations require an even greater reduction in automobile emissions. A shuttle service operator transports tourists from hotels to the front entrance of a federal park. If the EPA regulation applies to her shuttle service, the operator will have to spend several thousand dollars for each shuttle to modify the engine exhaust system. The operator brings suit to challenge the validity of the federal regulation as applied to her. How will the court likely rule on the shuttle operator's challenge? A) Find the statute constitutional, as a proper exercise of Congress's power to regulate federal property. B) Find the statute constitutional, as a proper exercise of Congress's power to regulate for the general welfare. C) Find the statute unconstitutional, because Congress cannot regulate activities occurring outside a federal park. D) Find the statute unconstitutional, because this is an area reserved to the states under the Tenth Amendment.

(A) is correct. The Constitution gives Congress the power to "make all needful rules and regulations respecting the territory or other property belonging to the United States." This property power, combined with the Necessary and Proper Clause, which grants Congress the power to make all laws necessary and proper for carrying into execution any power granted to any branch of the federal government, allows Congress to enact any laws necessary and proper to protect trees on federal land, including placing restrictions on vehicles operating outside of federal property but near enough to have an impact on the federal property. (B) is incorrect. Congress may not regulate for the general welfare. Congress may only tax or spend for the general welfare. The regulation of vehicle emissions does not include any new taxes or spending. Thus, this purely regulatory measure cannot be justified under the General Welfare Clause. (C) is incorrect. As discussed above, the reach of the property power is not limited to the physical boundaries of the federal property. Moreover, the Commerce Clause would also serve as a basis for a federal regulation of vehicle emissions occurring outside federal property (although the property power is a more likely source of federal power here because the purpose of the regulation was to protect federal property). (D) is incorrect. Under the Tenth Amendment, the states enjoy police power and may regulate for the health, safety, and general welfare of state residents. However, under the Supremacy Clause, state power must yield to the valid exercise of federal power. Here, Congress properly enacted the statute pursuant to the property power, and the Tenth Amendment is no bar to acts of Congress pursuant to an enumerated constitutional power.

An attorney was employed by the United States Department of Health and Human Services in a regional office located in a tobacco-growing state. A labor contract between the agency and the clerical workers union contained a policy providing for termination of union employees only for certain specified grounds. The attorney, however, was not a member of the union and not covered by such a policy. The attorney was angered by the regional director's refusal to adopt a no-smoking policy for employees and visitors in the office. She posted a notice in the employee cafeteria ridiculing what she called the hypocrisy of an agency promoting health issues and nonsmoking programs while refusing to provide its employees with those same opportunities. The notice prompted a great deal of debate among the employees and was brought to the attention of the regional director, who was very displeased. Which of the following statements is most accurate regarding the director's right to dismiss the attorney? A) The attorney has a liberty interest in the exercise of her First Amendment rights that entitles her to a hearing to contest the grounds of her dismissal. B) The attorney has a property interest as a public employee that precludes her from being fired without notice and an opportunity to respond. C) The attorney has no right to a hearing because her statements were not an expression of views on public issues. D) The attorney has both a liberty interest and a property interest that entitles her to a pre-termination evidentiary hearing.

(A) is correct. If the attorney is fired, she has a right to a hearing to determine whether her First Amendment rights were violated by her dismissal. Under the Due Process Clause of the Fifth Amendment, a person has a liberty interest in the exercise of specific rights provided by the Constitution, including freedom of speech. If a government employer seeks to fire an employee for speech-related conduct when the speech involved a matter of public concern but is not made pursuant to her official duties, the courts must carefully balance the employee's rights as a citizen to comment on a matter of public concern against the government's interest as an employer in the efficient performance of public service. Under the Court's expansive interpretation of what is a public issue in this context [see Rankin v. McPherson (1987)], the attorney's statement would probably qualify. At the very least, she can make a sufficient showing that her termination violates her free speech rights to be entitled to a hearing on the issue under procedural due process principles. [See Givhan v. Western Line Consolidated School District (1979)] (B) is wrong because the attorney does not appear to have a property interest in her job. A public employee who is subject to removal only for "cause" has a property interest in her job and generally must be given notice of the charges against her that are to be the basis for her job termination, and a pre-termination opportunity to respond to those charges. Here, however, the attorney did not have a property interest in her job; she could have been dismissed for no reason at all. She was not covered by the labor contract between the agency and its clerical workers, and there appears to be no other basis for her to claim an entitlement to continued employment. (C) is wrong because the attorney is entitled to a hearing as long as she can raise a prima facie claim that her speech, which was regarding an important health issue and the perception of her agency, was on a public issue and therefore protected by the First Amendment. (D) is wrong for two reasons. As discussed above, the attorney does not have a property interest in her job. Also, due process does not necessarily entitle her to a pre-termination evidentiary hearing; a post-termination evidentiary hearing is probably sufficient. [See Cleveland Board of Education v. Loudermill (1985)]

A city entered into a contract with a cable and Internet provider allowing the provider to be the exclusive cable and Internet provider in the city in exchange for certain rights. To facilitate installation of the provider's fiber optic cables for all of the city's residents, the city passed an ordinance requiring all apartment owners to allow the provider to install cables in their buildings. Installation of the cables involved drilling a hole in one exterior wall of the building and running the cables between the interior walls of the buildings. The owner of an apartment building within the city did not want to allow new cables to be installed in his building, as he had gone through a similar process with another company three years earlier and he believed that the other company had damaged his property. To deter the installation, the owner filed suit against the city, claiming that the ordinance amounts to a taking under federal law. How should the court rule? A) For the owner, because the ordinance amounts to a taking without just compensation. B) For the owner, because the government does not have a right to require owners of private property to allow private installation of cable lines. C) For the city, because the ordinance is a regulation and not a taking. D) For the city, because any taking here is de minimis.

(A) is correct. The court should rule for the owner. The Fifth Amendment to the United States Constitution, which is applicable to the states through the Due Process Clause of the Fourteenth Amendment, provides that private property shall not be taken for public use without just compensation. Where there is a physical appropriation of property, there is a per se taking. Here, the ordinance requires apartment owners to relinquish their right to exclude the cable company from their property. Moreover, the ordinance requires the owners to cede space between the walls of their buildings to the cable company. Although the amount of property taken is small and might not have much value, the physical appropriation here still amounts to a taking. The de minimis doctrine does not apply. Thus, (D) is incorrect. (B) is incorrect because it is untrue. A city can require a property owner to turn over property to another private party as long as it is for a public purpose. The Supreme Court has defined the term "public purpose" very broadly, and cable access for all residents of a city surely would fall within the Court's definition of a public purpose. (C) is incorrect because, as discussed above, at least to the extent that the city has denied apartment owners the right to exclude the cable company and has required the owners to cede space in their buildings to the cable company, there is a physical appropriation for which just compensation is due.

Congress has enacted a statute that requires all companies engaging in business with the federal government to enact certain affirmative action programs in hiring. A small diner has been providing catering services for a local branch office of the United States Department of Agriculture. At lunch one day, a senior enforcement officer of the Department of Agriculture happens to notice the racial makeup of the diner's workforce, and informs the diner that it is in violation of the affirmative action statute, and files charges against the diner. If the diner challenges the validity of the federal statute, what is the government's best response to the argument that Congress has exceeded its legitimate powers? A) The act is a valid enforcement of the Due Process Clause of the Fifth Amendment and a valid exercise of the Enabling Clause of the Thirteenth Amendment. B) The act is a valid exercise of the commerce power and a valid exercise of the Enabling Clause of the Thirteenth Amendment. C) The act is a valid enforcement of the Due Process Clause of the Fifth Amendment and a valid exercise of the commerce power. D) The act is a valid exercise of the federal police power.

(B) is correct. Congress has addressed the problem of racial discrimination primarily under the commerce power, but the Thirteenth Amendment does contain an enabling clause that would authorize this type of statute because it is not limited to governmental action. [See Patterson v. McLean Credit Union (1989)] While the Due Process Clause of the Fifth Amendment has been applied to the federal government in the same way that the Equal Protection Clause of the Fourteenth Amendment has been applied to states in racial segregation cases, the diner is not a government agency. Therefore, the Fifth Amendment argument would not be very effective, because it is not the government itself that is being accused of discrimination. Thus, (A) and (C) do not offer the best response. (D) is incorrect because it misstates the law: There is no general federal police power.

A housing development contained one-, two-, and three-bedroom units. All units were suitable for occupancy, and the developers of the project filed the appropriate documents, including a Declaration of Restrictions that limited ownership and occupancy of the units to families or to groups of unrelated adults of not more than three in number. Each deed to the individual units also contained the restriction. One of the two-bedroom units was purchased by a woman and her boyfriend. They immediately moved into the unit with another unmarried couple who were friends of theirs. Other unit owners brought suit against the woman and her boyfriend to enjoin the occupancy by the other couple. If the other unit owners prevail, what will likely be the reason? A) The litigants are private parties and the restriction was not enacted by the government. B) Enforcement of the restriction is rationally related to a legitimate government interest. C) Notice was not given by the woman and her boyfriend to the sellers of the unit that they intended to occupy the residence with another couple. D) The restriction constitutes a lawful restraint on alienation.

(B) is correct. If the other owners prevail, it will be because the court's enforcement of the regulation is rationally related to a legitimate government interest. The Supreme Court has held that court enforcement of restrictive covenants in deeds constitutes state action, and thus a court may enforce a restrictive covenant only if it is constitutional. Under the Due Process Clause, unless fundamental rights are involved, government action is constitutional as long as it is rationally related to any conceivable legitimate end of government. Zoning regulations prohibiting three or more unrelated persons from living together have been held not to infringe on any fundamental rights in Village of Belle Terre v. Boraas (1974); hence, judicial enforcement of a private covenant with that prohibition likely would also be valid for that reason. (A) is wrong because, as discussed above, government action includes court enforcement of restrictive covenants between private parties, and deed restrictions based on race have been held unenforceable in Shelley v. Kraemer (1948), because enforcement would constitute government action supporting discrimination against a suspect class. Here, however, no suspect class or fundamental right is involved. (C) is wrong because it is not relevant to a determination of whether the restriction can be properly enforced by the court. (D) is wrong because the issue here involves occupancy of the unit rather than transfer or sale.

A state bans the use of disposable diapers to reduce the volume of nonbiodegradable material in its landfills. The ban was a boon for diaper services within the state, but many parents of young children were displeased with the use of conventional diapers. With support from retail establishments that lost business from the disposable diaper ban, a grass roots coalition formed to fight the ban funded a study showing that the trucks and cleaning supplies used by diaper services within the state harmed the environment more than disposable diapers. The coalition and retailers then filed suit seeking to have the ban on disposable diapers declared unconstitutional. If the court strikes down the statute, on which of the following constitutional provisions would its decision most likely be based? A) The Equal Protection Clause of the Fourteenth Amendment. *B) The Due Process Clause. C) The Impairment of Contracts Clause. D) The Privileges or Immunities Clause of the Fourteenth Amendment.

(B) is correct. Of the choices presented, the only likely basis to strike down the statute is under the Due Process Clause as a violation of substantive due process. Substantive due process tests the reasonableness of a statute; it prohibits arbitrary governmental action. Under substantive due process, when government action limits a fundamental right, the government must prove that the action is necessary to promote a compelling interest. If a fundamental right is not involved, the challenging party must prove that the act is not rationally related to any legitimate government interest. The retail sale of diapers is not a fundamental right, and so a challenger must prove that there is no rational basis for the statute. Almost any law can be justified under the rational basis standard. The law need not be the best law for accomplishing the government's goal. Thus, even if it is true that the disposable diaper ban causes more pollution than it prevents, because the ban is rationally related to reducing the volume of trash in landfills, the challenge is unlikely to succeed. Nevertheless, none of the other choices states a viable ground for invalidating the statute, and so (B) is the best choice. (A) is wrong because equal protection applies where a statute or governmental action treats similar people in a dissimilar manner (i.e., classifies people), and here there is no classification—under the statute no one can sell disposable diapers for use within the state. Thus, an equal protection argument is not applicable. (C) is wrong because the Impairment of Contracts Clause prohibits only the substantial impairment of existing contracts (and there are exceptions even where there is substantial impairment), and nothing in the facts indicates that forbidding the retail sale of disposable diapers would substantially impair any existing contract. (D) is wrong because the privileges and immunities covered by the Fourteenth Amendment are those attributes peculiar to United States citizenship (e.g., the right to petition Congress for redress or the right to vote for federal officers). The statute here does not affect such rights.

A state adopted legislation making it a crime to be the biological parent of more than two children. The stated purpose of the statute is to preserve the state's natural resources and improve the quality of life for the state's residents. A married couple has just had their third child. They have been arrested and convicted under the statute. Which of the following is the strongest argument for voiding the convictions of the couple? A) The statute is an invalid exercise of the state's police power because there is no rational basis for concluding that the challenged statute would further the government's stated interests. B) The statute places an unconstitutional burden on the fundamental privacy interests of married persons. C) The statute places too much discretion in state officials to determine who will be permitted to bear children. D) The statute denies married persons equal protection of law.

(B) is correct. The couple's strongest argument is that the law burdens their fundamental right of privacy. The right of privacy is not specifically mentioned in the Constitution, but the Supreme Court has recognized the right on a number of occasions. The right is strong when it involves acts of procreation by married couples. When the government restricts a fundamental right, such as the right here, the government must show a compelling interest in the regulation and that the regulation is necessary to achieve that purpose. This is a difficult standard to meet, and thus the statute will probably be struck down. Here, the stated purposes for the law (improving the quality of life and preserving natural resources) may be found by the Court to be compelling, but the limitation on the number of children that couples may have certainly is not necessary for these purposes. Thus, this is the couple's best argument. (A) is not a good argument for two reasons: (i) because a fundamental right is involved, the rational basis standard (suggested by this choice) is improper, as strict scrutiny is the appropriate standard; and (ii) the statute here in fact is rationally related to its stated purpose because this standard is a very easy one to meet. (C) is not as good an argument as (B) because the facts do not indicate that there is any discretion in state officials under the statute. The statute is violated when anyone is the biological parent of more than two children. (D) is incorrect because the Equal Protection Clause merely prohibits treating classes of persons differently without adequate justification, and here, the statute applies equally to all persons; it does not classify.

The state passed a law stating that "only persons living with their parents or guardians who are bona fide residents of the state shall be entitled to free public education; all others who wish to attend public schools within the state may do so, but they must pay tuition of $3,000 per semester." A 15-year-old girl moved in with her friend so that she could attend the public schools in the state, and the state legislature passed the tuition statute just as she completed her junior year. The girl wants to complete her senior year in the state high school, but cannot afford to pay tuition. If the girl sues in federal court to strike down the tuition statute, is the court likely to rule that the statute is constitutional? A) Yes, provided that the state can show that the statute is necessary to promote a compelling state interest. B) Yes, unless the girl can show that the statute is not rationally related to a proper state interest. C) No, because it infringes on the girl's fundamental right to an education. D) No, because it interferes with the girl's fundamental right to interstate travel.

(B) is correct. The court is likely to rule that the statute is constitutional. A bona fide residence requirement, such as this statute, that is not based on a suspect classification and does not limit the exercise of a fundamental right, is judged by the rational basis test. Thus, (A) is incorrect. The statute provides free education for all children who are bona fide residents of the state. Thus, it uniformly furthers the state interest in assuring that services provided for its residents are enjoyed only by residents. (C) is incorrect because education is not a fundamental right. (D) is incorrect because this statute does not impair the right of interstate travel. Any person is free to move to the state and establish residence there. This statute does not deter people from moving into the state.

A state statute has detailed classifications of civil servants for both state and city positions. It provides that all civil servants who have been employed for over 18 months may be dismissed only for "misconduct" and also requires that state and city agencies comply with all procedures set forth in any personnel handbook issued by that agency. The personnel handbook of the state tollway authority sets forth detailed procedures for dismissal of civil servant employees. The handbook provides that written notice of the grounds for dismissal must be given to the employee prior to dismissal, and that the employee must, on request, be granted a post-dismissal hearing within three months after the dismissal takes effect. An employee is entitled to present witnesses and evidence at the post-dismissal hearing, and is entitled to reinstatement and back pay if the hearing board decides that the employer has not shown by a preponderance of the evidence that the dismissal was justified. A state tollway employee who had been employed for three years recently was fired. After an investigation by state auditors, the employee was notified by registered letter that he was being dismissed because of evidence that he took bribes from construction firms in exchange for steering contracts to them. He was informed of his right to a hearing and requested one as soon as possible. Three weeks after his dismissal, the state personnel board conducted a hearing at which the employee denied the charges and presented witnesses to attest to his honesty. At the conclusion of the hearing, the board upheld his dismissal, finding that it was supported by a preponderance of the evidence. If the employee files suit in federal court challenging his dismissal on constitutional grounds, will he be likely to prevail? A) Yes, because the employee had a right to a pretermination hearing at which he could present witnesses to support his side of the story. B) Yes, because the employee had a right to have an opportunity to respond to the charges prior to his dismissal. C) No, because the state may establish the required procedures for terminating an interest that it created by statute. D) No, because the procedures taken for termination of the employee's job satisfied due process requirements.

(B) is correct. The employee will likely prevail because the procedures taken to terminate his employment did not satisfy due process. Under the Due Process Clause of the Fourteenth Amendment, a public employee who is subject to removal only for "cause" under a statute, ordinance, or personnel document has a property interest in continued employment that cannot be taken away without due process of law. The Court has held that such an employee generally must be given notice of the charges and a pretermination opportunity to respond to those charges. The employee must also be given a subsequent evidentiary hearing regarding the termination (with reinstatement if the employee prevails). [Cleveland Board of Education v. Loudermill (1985)] Here, the employee was notified of the charges but was not given any opportunity to respond to the charges until after his termination. Hence, his termination did not satisfy due process requirements. (A) is incorrect because the employee does not have to be given a full, formal hearing before his termination, as long as he is given oral or written notice of the charges, an explanation of the employer's evidence, and an opportunity to tell his side of the story. (C) is incorrect because the fact that the state created the employee's property interest in his job does not permit the state to define what procedures may be used to terminate the interest. The procedures to which the employee was entitled are determined by independent constitutional standards. (D) is incorrect because, as discussed above, the procedures followed here did not satisfy due process standards. The Supreme Court has held that an employee can be suspended from his job without a prior hearing if the government has a significant reason for removing the employee from the job and providing him with only a post-termination hearing. [Gilbert v. Homar (1997)—campus police officer suspended after being arrested and charged with felony drug offense] Here, however, there is no substantial reason why the employee could not have been given the opportunity to respond to the charges prior to dismissal.

A state engaged a private company to run its lottery, but the state maintained close regulation of the manufacture and distribution of lottery equipment by the private company in order to prevent frauds. One lottery regulation required the company to submit to the state all applications of persons being considered for employment. The state ran background checks on the prospective employees to ensure that they did not have a criminal record. A prospective employee that did not pass the state background check could not be hired by the company. An employee of the company who had a poor work record and called in sick often was spotted by her supervisor dancing at a bar one evening after she had called in sick during the day. The supervisor immediately told the employee that she should consider herself terminated, although the employee tried to explain that she in fact had been sick that morning but began feeling well by mid-afternoon. A state law provided that employees of the state could not be fired from their positions except for cause. The woman sued in federal court, claiming that she was constitutionally entitled to a hearing to determine whether her supervisor had cause to fire her. If the court rules correctly, will it find the employee's termination to be constitutional? A) Yes, because no hearing was required since the supervisor witnessed the employee's misconduct. B) Yes, because the employer is free to fire employees at will. C) No, because it violates the employee's right to procedural due process. D) No, because of the state's regulation of the hiring process.

(B) is correct. The employer was free under the Constitution to fire the employee without a hearing because she was an employee at will. Unless prohibited by a state statute, a private employer usually can fire an employee for any reason or no reason at all, absent a contract providing otherwise. (C) and (D) are related to each other and are both incorrect. Most constitutional guarantees prohibit state action and do not impose duties on private parties. That is not to say that state action can never be found in the acts of a private party—it will be found if the private party performs exclusive public functions or has significant state involvement in its activities. However, there is no state action here. Running a lottery is not an exclusive public function, and merely being the exclusive lottery supplier for a state does not constitute significant state involvement, even where the state controls who may be hired. State action will not be found merely because the state has granted a monopoly to a business or heavily regulates it. (A) is incorrect because whether a hearing is required does not turn on whether the supervisor witnessed the misconduct—if state action were involved, a hearing would have been required because a public employee who is subject to removal only for "cause" has a property interest in her job that cannot be taken without fair process. Since there was no state action, no hearing was necessary.

A town adopted an ordinance providing that a person must have been a resident of the town for at least one year to be eligible to vote in school board elections. A resident who moved to the town seven months ago attempted to register to vote in the school board elections scheduled for the next month. However, the town clerk refused to register the resident because he will not have resided in the town for a full year prior to the election. The resident filed a class action suit on behalf of all of the new residents of the town, challenging the validity of the one-year residency requirement. Which of the following statements is correct? A) If the resident's suit is not heard before the election, it will be dismissed as moot, because the resident will have met the residency requirement by the time of the next annual election. B) The resident will prevail even if the matter is not decided until after next month's election. C) As long as there is some legitimate purpose for the one-year residency requirement, such as the need to prepare voting lists, the residency requirement will be upheld. D) The resident will lose because one-year residency requirements have been held permissible restrictions on the right to vote in local elections.

(B) is correct. The resident will prevail even if the matter is not decided until after the election, because the suit is not moot and the residency requirement is unconstitutional. The resident's suit is not moot even if the matter will not be decided until after the election because other members of the class might have a live controversy. Under the case and controversy requirement of the Constitution, there must be a real, live controversy at all stages of the suit. If through the passage of time, the controversy between the parties is resolved, the case is said to be moot. However, there are exceptions to the mootness doctrine. In a class action, it is not necessary that the suit by the named plaintiff be viable at all stages, as long as the claim is viable by some member of the class. Thus, the suit here would not be moot. Moreover, the residency requirement here violates the resident's fundamental rights to vote and to interstate travel. A restriction on the right to vote is subject to strict scrutiny and is valid only if it is necessary to achieve a compelling state interest (otherwise the restriction violates the Equal Protection Clause by treating new residents differently from old residents). Relatively short residency requirements (e.g., 30 days) have been upheld as being necessary to promote the compelling interest of assuring that only bona fide residents vote. However, the Supreme Court has struck down longer durational requirements for lack of a compelling justification. Thus, the one-year requirement here probably unconstitutionally impinges on the right to vote. The residency requirement also impinges on the fundamental right to travel in the same manner (i.e., it discourages people from migrating by denying them the right to vote without a compelling reason). Thus, the requirement is invalid. (A) is incorrect because, as indicated above, the case will not be moot since other members of the class might have a live controversy. (C) is incorrect because it applies the wrong standard. Because fundamental rights are affected by the residency requirement here, the government must show a compelling justification; a mere rational or legitimate basis is not enough. (D) is incorrect because, as stated above, the Supreme Court has found that there was no compelling interest for a one-year residency requirement in order to vote.

A city's water board election laws provide that, although members of the board are elected at large, one member of the board is required to live within each of the five designated water districts within the city. The city's population was more or less evenly distributed among the districts when this election law was enacted. A resident and registered voter of the city investigated the district residency requirement and discovered that most of the city's newer residents had moved into the same two water districts, so that the city's population was no longer evenly distributed among the five water districts. Instead, 80% of the city's residents lived within its central and eastern water districts, while the other 20% of the city's residents were scattered among its three other, more rural, districts. If the resident files suit in federal court challenging the constitutionality of the residency requirement, how will the court most likely rule? A) The residency requirement is unconstitutional because it impairs the voters' equal protection rights, in that it gives the voters in the less populous districts more effective representation on the water board. B) The residency requirement is unconstitutional because it violates the candidates' equal protection rights. C) The residency requirement is constitutionally permissible because the water board members do not exercise legislative power. D) The residency requirement is constitutionally permissible because the water board members are elected at large.

(D) is correct. The residency requirement is permissible because the water board is elected at large. The Equal Protection Clause prohibits state dilution of the right to vote, so that when a governmental body establishes voting districts for the election of representatives, the number of persons in each district may not vary significantly. This is known as the principle of "one person, one vote." This principle applies to almost every election where a person is being elected to perform normal governmental functions (e.g., an election for trustees for a junior college district). However, the principle of one person, one vote generally is inapplicable where there is an at-large system of election (except where the system is adopted for discriminatory purposes). Here, the water board members are elected by all of the qualified voters in the city in an at-large system (rather than having the voters of each individual district select one board member apiece), and no discriminatory intent is evident. Thus, the statutory provision requiring board members to reside in each of the five districts does not result in an imbalance or a dilution of the voting rights of the citizens of the city. Consequently, (A) is incorrect, and (D) presents an accurate statement of the constitutionality of the residency requirement. (Note that the answer might be different under federal statute because the city would have to prove a valid, nondiscriminatory purpose.) (C) is incorrect even though it reaches the correct result. While the Supreme Court has exempted special purpose water storage districts from the one person, one vote requirement, the basis of the decision was the specialized nature of the entity. The constitutional requirements apply not only to legislators, but also to elected administrative and executive officials. (B) is incorrect because, even assuming that the residency requirement violates the candidates' equal protection rights, the resident would not have standing to raise the issue. Generally, a claimant must assert his own constitutional rights and cannot assert the rights of third parties.

A state legislature passed a law requiring all employers operating in the state's oil and natural gas fields to give preference in hiring to residents of that state. The bill banned the hiring of nonresidents unless no other qualified person could be found to fill an oilfield or natural gas field position. Under prevailing economic conditions, which included a substantial decline in petroleum prices, the statute was tantamount to a total ban on hiring of nonresidents because there were so many unemployed oil and gas workers and little new exploration was taking place. The plaintiff was an experienced oilfield worker who was denied a job in the state because his permanent residence was in another state, even though he had worked in many states and foreign countries and his qualifications were better than anyone else applying for the job. The sole reason given for not hiring the plaintiff was the preferential hiring statute favoring state residents. The plaintiff filed suit in federal district court challenging the statute. Who should prevail? A) The state, because employment discrimination is only unconstitutional if it involves race, religion, alienage, or sex. B) The state, because the state's interest in hiring local residents outweighs the interest of nonresidents. C) The plaintiff, because the law denies him the privileges and immunities of state citizenship. D) The plaintiff, because the preferential hiring law impairs the plaintiff's rights under the Contract Clause of the federal Constitution.

(C) is correct. The plaintiff should prevail because the state's law violates the Interstate Privileges and Immunities Clause of Article IV, Section 2. This clause, which provides that "citizens of each state shall be entitled to all Privileges and Immunities of citizens in the several states," prohibits discrimination by a state against nonresidents when the discrimination involves "fundamental rights." Fundamental rights for purposes of this clause are those involving important commercial activities (such as pursuit of a livelihood) or civil liberties. Here, the statute directly discriminates against nonresidents of the state by banning their being hired as oilfield workers unless no qualified residents can be found. Because employment is a fundamental right for purposes of the Interstate Privileges and Immunities Clause, the court should find the statute unconstitutional. (A) is incorrect. A state law that mandates employment discrimination on grounds of race, religion, etc., will probably violate the Fourteenth Amendment Equal Protection Clause, but that clause is not the only means of challenging discriminatory state laws. Discrimination based on state residency must pass muster under the Interstate Privileges and Immunities Clause. (B) is incorrect because the state cannot show a sufficient justification for discriminating against nonresidents. A state law requiring private sector employers to give hiring preference to residents will only be valid under the Privileges and Immunities Clause if the state can show that (i) the nonresidents are the cause or source of the problem it is attempting to solve, and (ii) there are no less restrictive means to solve the problem. [Hicklin v. Orbeck (1978)] In this case, the state's high unemployment was caused by the drop in petroleum prices rather than by a large influx of nonresident workers, and state encouragement of alternative industries would be a less discriminatory alternative than what amounted to a total ban on the hiring of nonresidents. Hence, neither element of this test is satisfied, and the statute will not be valid under the Privileges and Immunities Clause. (D) is incorrect because the Contract Clause only limits the ability of states to enact a law that retroactively impairs contract rights. It does not affect contracts not yet entered into. Thus, a hiring law would not implicate the Contract Clause.

A state statute requires that all new automobiles sold in the state shall be equipped with a certain safety system to protect passengers in the event of a collision. An automobile company that wants to sell automobiles in the state files an action to enjoin enforcement of the statute, arguing that the statute deprives the auto company of its right to contract freely with customers under the Due Process Clause. What is the appropriate burden of proof? A) The state must demonstrate a compelling state need. B) The state must demonstrate that the law serves a legitimate government interest. C) The challenger must demonstrate the lack of a compelling state need. D) The challenger must demonstrate the lack of a legitimate purpose.

(D) is correct. In reviewing laws under the Due Process Clause, the strict scrutiny test is used when a fundamental right is involved. In all other cases, the rational basis test is used. Under the rational basis test, a law will be upheld if it is rationally related to a legitimate government interest. Under that test, laws are presumed valid; therefore, the challenger has the burden of proof. Here, the statute in question does not involve a fundamental right and so the rational basis test would be used to examine the statute. (A) is incorrect. It reflects the burden when strict scrutiny applies. As explained above, the rational basis test applies here because a fundamental right is not involved. (B) is incorrect. When the rational basis test applies, as here, the challenger must prove the lack of either a legitimate governmental interest or a rational link between the law and the end sought. The state does not have the burden of proof. (C) is incorrect. When the burden of proof requires a showing of a compelling state need, that burden is always on the state. However, such a burden of proof is applicable only when the statute involves a fundamental right. The statute here does not involve a fundamental right and, therefore, the burden is on the challenger to prove that no rational purpose exists for the statute.

A township located in a farming community was composed mostly of persons belonging to a specific religious sect. To help instill proper respect for authority in children, which was a central tenet of the sect, and to maintain order in the classroom, the local school board allowed teachers to inflict corporal punishment. Such punishment was inflicted on a fourth grader in a township school immediately after his teacher saw him pulling a girl's hair. Neither he nor his parents belonged to the religious sect. When the boy's parents learned of the incident, they hired an attorney. Rather than suing the teacher for battery as permitted under state law, the attorney brought an action against the teacher under a federal statute providing a cause of action for damages against any government employee who deprives a person of his constitutional rights. Should the court find the policy allowing corporal punishment to be constitutional? A) No, because the punishment policy violates the First Amendment Establishment Clause. B) No, because the boy was denied any kind of hearing, in violation of his right to procedural due process under the Fourteenth Amendment. C) Yes, because under the doctrine of parens patriae states may impose any punishment they see fit. D) Yes, because the punishment was not grossly disproportionate under the Eighth and Fourteenth Amendments.

(D) is correct. The punishment here is constitutional because it does not violate any constitutional provision. The best answer reflecting this reasoning is (D)—there was no Eighth Amendment violation here—because paddling students as a disciplinary measure has not been found to be cruel and unusual punishment. (A) is incorrect because there is no Establishment Clause violation here. Under the Establishment Clause, if there is no sect preference, government action generally will be upheld if the action serves a secular purpose, its primary effect neither advances nor inhibits religion, and it does not excessively entangle government with religion. There is no sect preference under the school board's corporal punishment rule here, the rule has the secular purpose of maintaining order in the classroom (the fact that this coincides with the tenets of a local religion does not change that conclusion), its main purpose neither advances nor inhibits religion, and there is no excessive entanglement. (B) is incorrect because there has been no deprivation of procedural due process. The Supreme Court has held that although corporal punishment may involve a liberty interest, no hearing is required prior to inflicting such punishment; the possibility of a common law action in tort is sufficient procedural protection. [Ingraham v. Wright (1977)] (C) is incorrect because it is too broad. The doctrine of parens patriae allows the state to stand in the shoes of a parent, but even a parent may not impose any punishment he sees fit (e.g., a parent may not break a child's arm as punishment for stealing).

Congress passed a law allowing widespread oil exploration on federal lands in the western United States. A large deposit of oil sand was discovered in one western state and Congress authorized an oil sand refining plant to be built on federal park land within the state. The refinery was built in compliance with federal pollution regulations. Pursuant to state law, the plant manager allowed the state to inspect the plant before putting it into operation. Because state refinery standards were more strict than the federal standards (in order to better protect state citizens from pollution associated with refineries), the refinery did not pass the inspection, and the state inspector refused to give the manager a permit to run the refinery. The refinery manager nevertheless began to run the refinery and was fined by the state. Which of the following is the manager's best defense against imposition of the fine? A) The state does not have a compelling interest in regulating the refinery, because it is within a federal park. B) The state regulation is invalid because Congress has preempted the field of pollution control. C) The state pollution regulation is invalid because it is inconsistent with the state's compelling interest in providing jobs. D) The state law violates the principles of intergovernmental immunity as applied to the manager.

(D) is correct. The state law violates intergovernmental immunities principles. The states have no power to regulate the activities of the federal government unless Congress consents to the regulation. Thus, instrumentalities and agents of the federal government are immune from state regulations that interfere with their federal functions. Here, the regulation clearly interferes with the manager's duties to run the refinery. While it might be argued that the manager agreed to comply with the state regulations, because he allowed the state inspection, nothing indicates that Congress consented, and so the state regulation cannot be applied to the manager. (A) is factually incorrect; a state may still have a compelling interest in activities on federal lands, and the interest here of preventing pollution that may spread beyond federal lands probably is compelling. Nevertheless, the argument is without merit because it is irrelevant whether the state has a compelling interest in regulating federal activities; a state simply is not allowed to interfere with federal activities. (B) is incorrect because not enough facts are given to make this determination. The Supremacy Clause prohibits states from adopting laws that interfere with federal laws, and this prohibition extends to any law—even one that seeks to support the federal scheme—when Congress has preempted the field. In determining whether a field has been preempted, courts will consider the comprehensiveness of the federal scheme. The question does not give enough facts to make the determination here. (C) is incorrect because it is irrelevant. The Constitution does not require the state to favor one compelling interest over another, and so states are free to adopt laws that interfere or are inconsistent with other state goals, unless the laws are arbitrary, in which case they would violate substantive due process. The laws here do not appear to be arbitrary, and so (C) does not offer a viable defense.

The director of a one-person field station of the United States Department of Agriculture ("USDA") in a small town was instructed by his superiors to sell surplus government cheese and butter to local low income residents at 10% of market value. All sales were conducted at the USDA warehouse next to the field station. Pursuant to state statutes allowing municipal governments to establish reasonable regulations governing the retail sale of foodstuffs, the town in which the field office is located requires any establishment for the retail sale of food to pass a health and sanitation inspection and meet other specified criteria for obtaining a city license. The director of the field office failed to obtain a license from the town and was prosecuted for the failure. Which of the following will provide the best defense for the director in this prosecution? A) The ordinance under which the director is being prosecuted is invalid as an undue burden upon interstate commerce. B) The ordinance under which the director is being prosecuted violates the Equal Protection Clause of the Fourteenth Amendment. C) The ordinance under which the director is being prosecuted deprives him of property without due process of law. D) The ordinance under which the director is being prosecuted violates the principles of intergovernmental immunity as applied to him.

(D) is correct. The town licensing requirement unconstitutionally impinges upon a duly authorized federal program. The United States Government, as well as its agencies and instrumentalities, is immune from state regulation that interferes with federal activities, functions, and programs. To the extent that state regulations substantially interfere with an authorized federal program, the state laws must yield. Here, the director, as an agent of the federal government, was carrying out a duly authorized program of the Department of Agriculture by conducting sales of surplus government food at a federally owned warehouse. To sustain the power of the town to prosecute the director for not having a retail food sale license would give the town overriding authority over the selection of personnel to administer a federal program, as well as over the means by which this program is to be implemented. Thus, the licensing requirement would substantially interfere with the proper functioning of this federal program by directly interfering with a federal employee in the carrying out of his orders. (A) is incorrect because the facts do not indicate that the licensing requirement is in any way a measurable burden upon interstate commerce, much less an undue burden. The licensing regulation appears to affect, almost exclusively, the peculiarly local concerns of health and sanitation. (B) is incorrect because an equal protection violation exists where a law unreasonably limits the liberty of some persons but not others, i.e., where a law treats similar persons in a dissimilar manner. The licensing regulation at issue here is apparently being applied in an evenhanded fashion, and the director is not being treated differently from anyone else who does not have the required license. Thus, there is no equal protection problem. (C) is incorrect because the director is not being deprived of any property or interest to which he has a legitimate claim; e.g., he is not being deprived of employment to which he is entitled. Because there is no deprivation of property, there is no due process issue.

A federal statute authorized the President to designate countries as state sponsors of terrorism and impose economic sanctions on them. When a new President took office, he declared numerous traditional U.S. allies state sponsors of terrorism and imposed sanctions on them. In response, Congress passed a joint resolution, identifying many of the allies as non-sponsors of terrorism and reinstating previously existing trade agreements with those countries. If the President seeks to enforce the sanctions, are they valid? A) Yes, because the joint resolution was not signed by the President. B) Yes, because Congress has the power to regulate domestic commerce, but not foreign commerce. C) No, because Congress has exclusive power to regulate commerce with foreign nations. D) No, because presidential economic power is at its lowest ebb when its use conflicts with the will of Congress.

Choice (A) is correct. The president's action is valid because it complied with a valid statute, and the joint resolution did nothing to change the statute. To modify a statute, Congress must enact a new law, passed by both houses and signed by the President. A joint resolution passed by both houses but not signed by the President has no legal impact. Choice (D) is wrong because the President did not act in the face of Congressional will from a legal perspective (since the President acted pursuant to a statute, and Congress did not repeal or modify the statute). Choice (B) is wrong because it is a false statement. Congress has power to regulate both domestic commerce and commerce with other countries. Choice (C) is wrong because Congress must pass laws to exercise its power to regulate foreign commerce.

Following a series of terrorist attacks in a small foreign country in which five American citizens were taken hostage, the President issued an executive order to cut off all diplomatic ties with that nation, including a prohibition on virtually all trade agreements. Under the terms of the executive order, the country's embassy was closed and its consul was ordered to leave the United States. The Prime Minister of the foreign country, which imports two billion dollars of corn from the United States each year, threatened to stop all U.S. imports immediately unless the embassy was reopened and the consul reinstated. The consul brought an action against the United States in federal district court to enjoin enforcement of the President's action. Will the court hear the case? A) Yes, because the President's action usurps the power of Congress to regulate issues directly or indirectly affecting foreign commerce. B) Yes, because the consul has suffered redressable harm. C) No, because lower federal courts lack original jurisdiction over cases involving ambassadors and consuls. D) No, because Article II gives the President exclusive authority to recognize foreign governments.

Choice (D) is correct. The action brought by the consul should be dismissed by the federal court because Article II gives the President exclusive authority to recognize foreign governments. Because the action calls into question the President's inherent foreign policy powers, it should be dismissed as a non-justiciable political question. Choice (A) is incorrect because, although Congress has the power to regulate commerce with foreign nations, the President alone has exclusive power to represent the United States regarding issues that deal with foreign affairs. Choice (B) is incorrect because the consul has standing since her injury is fairly traceable to the President's decision and the harm suffered could be redressed if the decision were overturned. However, the court will likely dismiss the action, nonetheless, since it presents a non-justiciable issue. Choice (C) is incorrect because, while it is true that the United States Supreme Court has original jurisdiction over cases involving ambassadors and consuls, federal district courts may also exercise concurrent jurisdiction over such actions.

A foreigner in the United States attempted to kill the head of his country while the leader was confined to a state's hospital. The foreigner was tried in the state for the crime of attempted murder, and he was sentenced to 10 years in prison. Two years later, the President wanted to improve diplomatic relations with the foreigner's country, so the President pardoned the foreigner and ordered his release. Which of the following presents the strongest argument available to the governor of the state in declining to release the prisoner? A) The presidential pardon power does not extend to the foreigner. B) The President's power as Commander in Chief does not extend to a state's prisoners. C) When the governor of a state acts pursuant to the state's constitution, he is exempt from regulation by the federal government. D) The President, under the constitutional mandate to execute the laws, must honor any valid judgments of a state's courts.

(A) is correct. Article II, Section 2 of the Constitution gives the President the power to grant pardons only for offenses against the United States. Therefore, the presidential pardon power extends to federal crimes only, not to state crimes. Since the foreigner committed a state crime, the President has no power to pardon him. (B) is incorrect. The President's power as Commander in Chief extends to the criminal justice system, but only in time of war and only to military personnel. This answer choice is not a good argument, because the President is not acting in his capacity as Commander in Chief, but rather he is attempting to exercise his presidential pardon power under the United States Constitution. (C) is incorrect. The issue in this question is not a conflict between state and federal law, but rather the scope of the presidential pardon power. Moreover, even if there is a conflict between a state and federal regulation, the federal law will prevail over the state constitution under the Supremacy Clause. (D) is incorrect. The President has an obligation to execute the laws of the United States, not those of any individual state. Therefore, this answer choice has no relevance to the President's constitutional authority.

Congress enacted a law setting forth new requirements concerning the permitted variance in the size of congressional districts. Because many of the congressional districts in which members of Congress had traditionally run would be changed by these requirements, and because Congress desired a speedy determination of the constitutionality of the statute, the statute specifically grants standing to any sitting Congress member to sue the relevant state redistricting commission to determine the constitutionality of the act as it affects that Congress member's district. The law provides that original jurisdiction for such suits is mandated in the United States Supreme Court. Is the portion of the statute establishing exclusive original jurisdiction in the United States Supreme Court constitutional? A) No, because it is inconsistent with Article III of the Constitution. B) No, because by granting standing only to sitting Congress members, it denies equal protection of the laws to other individuals who may wish to run for Congress. C) Yes, because it is authorized pursuant to the power of Congress to enforce the Fifteenth Amendment. D) Yes, because Congress has the power to determine the jurisdiction of federal courts.

(A) is correct. Article III of the Constitution grants Congress the power to establish federal courts and gives Congress the power to delineate the jurisdictional limits, both original and appellate, of those courts. However, Congress is bound by the standards of judicial power set forth in Article III as to subject matter, parties, and the requirement of "case or controversy." Under Article III, the Supreme Court has original jurisdiction "in all cases affecting Ambassadors, other public Ministers and Consuls, and those in which a State shall be a Party." This provision is self-executing: Congress may neither restrict nor enlarge the Supreme Court's original jurisdiction. Here, the statute attempts to add a new class of cases to the original jurisdiction of the Supreme Court and, thus, is unconstitutional. (B) is incorrect. It reaches the correct result but for the wrong reason. Congress has the power to specifically confer standing in cases brought in federal court. Here, there is a logical nexus between a sitting Congress member and the composition of the electoral district, and there is no constitutional requirement that other individuals be granted similar standing. (C) is incorrect. Congress clearly has the power to pass voting legislation concerning issues of race pursuant to the Fifteenth Amendment. However, the issue here is not the validity of the statute itself, but rather only the provision creating exclusive original jurisdiction in the United States Supreme Court to hear claims raised by redistricting. The fact that the rest of the statute may be valid under the Fifteenth Amendment does not save the jurisdictional provision from constitutional attack. As explained above, Congress cannot add to the original jurisdiction of the Court. Therefore, this attempt to add a new class of cases to the original jurisdiction of the Court is unconstitutional. (D) is incorrect. This answer choice correctly states that Congress has the authority to prescribe the jurisdiction of lower federal courts. However, as explained above, Congress cannot enlarge the original jurisdiction of the Supreme Court.

Congress enacted a statute making it a federal crime for any person not licensed to carry a hand-gun to possess one on the premises of any public or private primary or secondary school. An agent of the Bureau of Alcohol, Tobacco, and Firearms was giving a talk to a group of seniors at a high school. He noticed a handgun in the pocket of a student attending the talk and, after finding that the student did not have a license, arrested him for violating the statute. At trial, the student moved for acquittal on the ground that the act is unconstitutional. How should the court rule on the motion? A) Grant the motion, because the statute does not substantially affect interstate commerce. B) Grant the motion, because the power to regulate conduct in public schools is reserved to the states under the Tenth Amendment. C) Deny the motion, because there is a rational relationship between the regulation of firearms at schools and interstate commerce. D) Deny the motion, because the firearm probably traveled in interstate commerce.

(A) is correct. Congress's power to regulate commerce is very broad, but it does have limits. To be within Congress's power under the Commerce Clause, a federal law must regulate (i) the channels of interstate commerce, (ii) the instrumentalities of interstate commerce and persons and things in interstate commerce, or (iii) activities that have a substantial effect on interstate commerce. When Congress attempts to regulate intrastate activity under the third prong above, the court will uphold the regulation if it is of economic or commercial activity and the court can conceive of a rational basis on which Congress could conclude that the activity, in aggregate, substantially affects interstate commerce. If the regulated intrastate activity is noncommercial and noneconomic, it cannot be regulated under the Commerce Clause unless Congress can factually show a substantial economic effect on interstate commerce. Here, the federal statute barring possession of a gun on the school premises is not an economic or commercial activity and does not substantially affect interstate commerce. (This question is based on United States v. Lopez, in which the Supreme Court struck down the Gun Free School Zone Act on the ground that preventing individuals in schools from carrying firearms does not substantially affect interstate commerce and is, therefore, beyond the scope of the federal commerce power.) (B) is incorrect. This choice is too narrow. While the inevitable result of the Lopez decision is to leave the regulation of firearms at schools to the states, the Court did not indicate that it was an area exclusively in the province of the states under the Tenth Amendment. (C) is incorrect. While the rational basis standard generally is appropriate in reviewing the exercise of Congress's commerce power, as explained above, in this case Congress has exceeded the scope of its commerce power. (D) is incorrect. There is no question that Congress could have enacted a constitutional statute dealing with guns at schools by attaching a jurisdictional tag so that it was regulating guns that had traveled in interstate commerce. It would then be regulating the channels of interstate commerce, an area in which court review is minimal. This act, however, was not limited to guns in interstate commerce.

A city ordinance requires its mayor to have been a resident of the city for at least five years at the time of taking office. A candidate who is thinking about running for mayor in an election that will take place next year will have been a resident for only four and one-half years at the time he would take office. Before he decides whether to run, the candidate wants to know whether he could lawfully assume the position if elected. The candidate filed suit in the local federal district court for a declaratory judgment that the five-year residence requirement is unconstitutional. He named the chairman of his political party as the sole defendant but did not join any election official. The chairman responded by joining the candidate in requesting that the court declare the residence requirement invalid. Which of the following best reflects the action the court should take? A) Refuse to determine the merits of this suit, because there is no case or controversy. B) Refuse to issue the declaratory judgment, because an issue of this kind involving only a local election does not present a substantial federal constitutional question. C) Issue the declaratory judgment, because a residency requirement of this type is a denial of the equal protection of the laws. D) Issue the declaratory judgment, because the candidate will have substantially complied with the residency requirement.

(A) is correct. For a federal court to hear a case there must be a "case or controversy" involved. Federal courts can hear actions for declaratory relief, but there must be an actual dispute between parties having adverse legal interests. Here, the candidate did not sue the election official, and the defendant he did name does not have a legal interest adverse to him. Also, plaintiffs must show that they have engaged in (or wish to engage in) specific conduct and that the challenged action poses a real and immediate danger to their interests. Here, the candidate has not taken specific steps to run for office and appear on the ballot. Therefore, there is no case or controversy. (B) is incorrect. Residency requirements as a condition for a candidacy raise constitutional issues under the right to travel and the Equal Protection Clause. Thus, there is a federal question at stake. (C) and (D) are incorrect. As explained above, before a court can reach the merits, it must determine that there is a case or controversy. In this situation, there is none.

Which of the following acts would be improper for the United States Senate to perform? A) Adjudicating a border dispute between states. B) Defining certain qualifications for being a member in good standing of the United States Senate. C) Sitting in joint session with the House of Representatives. D) Passing a resolution directing the President to pursue a particular course of foreign policy.

(A) is correct. It would be improper for the Senate to adjudicate a border dispute between states. The Constitution divides the power of the federal government among the three branches. The power to adjudicate actions where two or more states are involved, which would include actions to adjudicate border disputes, is vested in the judicial branch by Article III of the Constitution. Thus, it would be improper for the Senate to adjudicate such a border dispute. (B) is incorrect because it would be proper for the Senate to define certain qualifications for being a member in good standing of the Senate. The Constitution lists certain minimal requirements for Senators, but grants each House of Congress the power to be the judge of member qualifications and allows each to determine rules for proceedings. [U.S. Const. art. I, §5] (C) is incorrect because the Constitution contains no prohibition against joint sessions of Congress and, as mentioned above, it grants both Houses the power to regulate their proceedings, which would include the manner of their meetings. (D) is incorrect because Congress has some power over foreign policy—through its war power, its treaty power, and its power to regulate foreign commerce—and a resolution would probably be proper pursuant to such power. Moreover, while the President's power over foreign affairs is paramount, a resolution such as the one here amounts to nothing more than a suggestion to the President, and so would not be a usurpation of the executive.

An ordinance of a city prohibits leafleting on the grounds of any hospital or on the sidewalks within five feet of the hospital entrances during visiting hours. A member of a religious group advocating prayer to restore the sick to good health is arrested for violating the ordinance, is fined $100, and is convicted. She appeals her conviction, claiming that her constitutional rights were violated. The case was heard by the state supreme court, which ruled that while the ordinance was permissible under the United States Constitution, it was unconstitutional under the state constitution because the fine money was designated to go to the city's only hospital, which was privately owned, rather than to the city. The city seeks to bring the case before the United States Supreme Court. Should the United States Supreme Court grant certiorari? A) No, because the case was decided on independent state grounds. B) No, because the case is moot. C) No, because this is a political question. D) Yes, the Supreme Court should hear the case on its merits, because it involves an important federal question.

(A) is correct. The Court should refuse to grant certiorari because the case was decided on independent state grounds. The Supreme Court will not hear a case from a state court, even though it has jurisdiction over the parties and the subject matter, if there are adequate and independent state grounds to support the decision. Here, the state court held the law invalid on state, rather than federal, constitutional grounds. Therefore, the Supreme Court should refuse to grant certiorari. (B) is incorrect because mootness goes to whether there is a real, live controversy at this stage of the proceeding. The case here is not moot because the city wants to challenge the ruling that the law was unconstitutional under the state constitution. If the city wins, it may enforce the ordinance. (C) is incorrect because no political question is involved here. A political question, which is nonjusticiable, is one involving an issue that the Constitution has committed to another branch of the government or that is inherently incapable of judicial resolution and enforcement. Whether a law is constitutional—a determination of law—is certainly capable of judicial resolution and is properly within the judiciary branch's powers. Thus, a political question is not involved. (D) is incorrect because, as explained above, the basis for the decision was a state, not federal, violation. Therefore, there is no federal question.

At a meeting with some of his advisors, the President lost his temper and loudly and unfairly criticized a position paper prepared by a female aide with whom he had had an affair just before he was elected President. He told the aide she was fired and would never work in government again "except maybe in the kitchen." The outrageous remarks caused the aide severe emotional distress. She brought suit for intentional infliction of emotional distress. If the President moved to dismiss, what would be the most important criterion that the court would use to decide the motion? A) Whether the President's actions were predicated on his official acts. B) Whether the President would be hampered in the conduct of his official duties in defending the lawsuit. C) Whether the separation of powers doctrine is served by postponing the case until the President left office. D) Whether the President's affair with a subordinate constituted a crime.

(A) is correct. The President has absolute immunity from civil damages based on any action that the President took within his official responsibilities (even if the action was only arguably within the "outer perimeter" of presidential responsibility). However, the President has no immunity from private suits in federal courts based on conduct that allegedly occurred before taking office. The rationale is that the immunity is intended only to enable the President to perform his designated functions without fear of personal liability. Here, the critical question posed by a motion to dismiss is whether the President enjoys immunity. If the tort was committed as part of his official duties, then his immunity will be absolute. If his conduct could be construed to relate to his unofficial acts, then he would enjoy no immunity. (B) is incorrect. While the effect of the case on the conduct of the presidency is an issue that a court would consider in determining how a case against the President should be scheduled, it would not be grounds for dismissal. (C) is incorrect. Separation of powers does not prevent courts from adjudicating civil suits against the President. (D) is incorrect. The issue of the commission of a crime might be relevant on the issue of impeachment. It has nothing to do with a motion to dismiss.

A federal statute enacted about 100 years ago admitted a state to the Union, granted the state certain public lands, and established conditions on the subsequent disposition of these lands by the state. The federal statute also required the state to write those same conditions into its state constitution. One hundred years later, a statute of the state dealing with the sale of these public lands was challenged in a state court lawsuit on the ground that it is inconsistent with the conditions contained in the federal statute and the state constitution. The trial court decision in that case was appealed to the state supreme court. In its opinion, the state supreme court dealt at length with the ambiguous language of the federal statute and with cases interpreting identical language in similar federal statutes. The state supreme court opinion did not discuss the similar provisions of the state constitution, but it did hold that the challenged state statute was invalid, because it was "inconsistent with the language of the federal statute and therefore is inconsistent with the identical provisions of our state constitution." The losing party in the case wishes to appeal the decision to the United States Supreme Court. Can the United States Supreme Court review the decision? A) Yes, the Supreme Court can accept the case for review and determine the validity and interpretation of the federal statute. B) Yes, because the state statute was adopted pursuant to federal law. C) No, on the ground that the decision of the state supreme court rests on an adequate and independent state ground. D) No, because a decision by a state supreme court concerning the proper disposition of state public lands is not reviewable by the United States Supreme Court.

(A) is correct. The Supreme Court may hear a case from a state court only if the state court judgment turned on federal grounds. The Court will refuse jurisdiction if it finds adequate and independent nonfederal grounds to support the state decision. Here, the decision of the state supreme court was not made on an independent state ground because the state supreme court held that the state constitution must be interpreted in a way consistent with the federal statute. Therefore, the decision can be reviewed by the United States Supreme Court. (B) is incorrect. Although this choice reaches the correct result, its rationale is invalid. The mere fact that a state law is based on federal law does not mean all court decisions involving the state law are automatically reviewable by the Supreme Court. If the state law is ruled invalid relying solely on nonfederal grounds, the decision is not reviewable. (C) is incorrect. The decision will be held to have rested on federal grounds. Nonfederal grounds are adequate if they fully dispose of a case. They are independent if they are not based on federal law. Here, the state court decision was not independent; it was based entirely on interpretation of federal statutes with similar language. (D) is incorrect. It is overbroad. A decision of a state supreme court is reviewable by the United States Supreme Court as long as it was not made on adequate and independent state grounds. In this case, as explained above, it was not.

A cattle-producing state adopted a statute requiring any food service business operating in the state to serve beef raised in the United States. A licensed hot dog vendor who worked at a football field within the state and who had been buying hot dogs made with foreign beef for the past several years estimated that switching to an all-beef hot dog made from United States beef would reduce his profits by 10%. An attorney hired by the vendor to challenge the statute discovered during research into the case that most of the footballs used at the football field at which the vendor worked were made of foreign leather. Which of the following grounds is the vendor's best argument against the constitutionality of the state statute? A) The statute burdens foreign commerce. B) The statute violates equal protection guarantees because it is not rational to prohibit the sale of foreign beef but not foreign leather. C) The statute substantially interferes with the vendor's right to earn a living under the Privileges or Immunities Clause of the Fourteenth Amendment. D) The statute constitutes a taking without due process of law.

(A) is correct. The best argument against the constitutionality of the state statute is that it burdens foreign commerce. For all practical purposes, the power to regulate foreign commerce lies exclusively with Congress. Therefore, a state that adopts legislation requiring private vendors to favor United States products over foreign products, as the state did here, may be acting outside the scope of its powers. (B) is incorrect because the statute is a rational method of protecting local beef interests. The rational basis standard applies when an economic law, such as the one here, is challenged on equal protection grounds. Under the standard, the Supreme Court will usually defer to a legislature's decision that the law is rational notwithstanding the fact that the statute is underinclusive. In other words, the law need not address all of the problems that prompted its passage; it will be upheld even if it is only a "first step" toward a legitimate goal. Here, prohibiting the use of foreign beef appears to be a rational method of protecting state beef raisers. Thus, it is irrelevant that the statute is underinclusive in that it allows the use of both United States beef and foreign leather. (C) is incorrect because the right to earn a living is not a privilege under the Fourteenth Amendment, which protects against infringement of rights of national citizenship, such as the right to petition Congress for redress. (Neither would the statute violate the Privileges and Immunities Clause of Article IV, because that clause only prohibits discrimination against citizens of other states and the statute here treats citizens of all states the same.) (D) is incorrect because the vendor had all of the process that was due him. Because the government action here was a general act and not an individualized adjudication, the vendor had no right to an individual hearing; the normal procedure for adopting a statute is all the process that is due.

Congress created the National Agency for Burglar Alarms ("NABA"), giving it the power to regulate both burglar alarm hardware and installation personnel. NABA adopted a regulation requiring that all burglar alarm installation companies be licensed, and providing that anyone installing an alarm without a license could be fined. The regulation also provided that any company in the installation business on the day the regulation was adopted automatically would receive a license, but to obtain a license thereafter, an applicant would have to show that he has worked as an installer at a licensed company for at least three years. A man who has been installing alarm systems for eight years sold his installation business a few months before the NABA regulation was adopted and went to work for the purchaser servicing his old accounts. A few months after the NABA regulation was adopted, a representative from a national department store chain approached the man with an offer to hire him as an independent contractor to revamp the chain's alarm systems. The man quit his job and applied for a NABA installer's license. His application was denied because he was not in business on the day the NABA regulation was adopted and had worked for a licensed installer for only a few months. The man decided to install the alarm systems anyway. Can the man properly be fined for installing alarms? A) Yes, because the NABA was established under Congress's power to legislate for the general welfare, and Congress may take whatever steps are necessary and proper to enforce its laws. B) Yes, because the regulation falls within the scope of Congress's commerce power, and Congress may delegate its authority to regulate as it has done here. C) No, because the regulation interferes with the man's fundamental right to earn a living without a substantial justification and so violates the Privileges and Immunities Clause of Article IV, Section 2. D) No, because a government agency cannot itself levy fines for a violation of its regulations.

(B) is correct. The man can be fined. Congress has the power to regulate alarm installation companies under the Commerce Clause because the clause permits Congress to regulate any local or interstate activity that, either in itself or in combination with other activities, has an effect on interstate commerce. Burglar alarm companies use instrumentalities of interstate commerce such as phone lines and have a cumulative effect on interstate commerce even though some may only do business locally. Hence, their activities can be regulated by Congress. The delegation to the NABA is valid because Congress has broad discretion to delegate its legislative power; the Supreme Court will uphold almost any delegation of congressional power. Therefore, the man can be fined. (A) is incorrect because it improperly mixes two concepts. Congress does not have the power to legislate for the general welfare—there is no federal police power—but rather Congress has the power to spend for the general welfare. (C) is incorrect because, even assuming that the regulation here interferes with the man's right to make a living, it would not violate the interstate Privileges and Immunities Clause of Article IV because the clause restricts states, not the federal government. (D) is incorrect. Congress can provide that violation of an agency's regulations is a criminal offense that can be enforced through the imposition of fines. Furthermore, an agency has the power to impose civil fines and penalties for a violation of its regulations.

A federal statute imposed criminal penalties for killing certain specified animals that have been determined by Congress to be of importance to the tourism industry in the region in which the animal was located. Among the animals protected was the red fox. However, a state in which the red fox was located classified the animal as a varmint that could be destroyed at will by anyone with a general hunting license. A rancher who possessed a valid state general hunting license regularly shot and killed red foxes that ate his artichoke plants. If the rancher is prosecuted under the federal statute and challenges the constitutionality of the law, which of the following is the strongest constitutional argument in support of the federal statute? A) The commerce power. B) The Necessary and Proper Clause. C) The police power. D) The power to regulate federal lands.

(A) is correct. The commerce power is the strongest argument. Congress has the power, under the Commerce Clause, to regulate any activity that, taken cumulatively, has substantial economic or commercial effect on interstate commerce. Although there are limits on the power of Congress to regulate commerce, in only a few cases has the Court invalidated a federal law as exceeding the scope of Congress's commerce power. Because Congress has concluded that the animal is important to the region's tourism industry, and given the comparative weakness of the other answers, (A) is the strongest argument. (B) is incorrect because the Necessary and Proper Clause must be linked with another constitutional power of Congress. Here it is presented by itself, not in connection with another power, and thus it is incorrect. (C) is incorrect because there is no federal police power. (D) is wrong. The facts of this question do not offer any facts suggesting that the animals are on federal lands. Thus, the power to regulate federal lands is irrelevant.

An employee of the United States Department of Labor was instructed by his superior to solicit subscriptions to the Department's bulletin on a door-to-door basis in the city in which he worked. While doing so, the employee was arrested for violation of a city ordinance that prohibited commercial solicitation of private residences. What is the employee's best defense? A) Intergovernmental immunity. B) The First Amendment freedom of expression as it applies to the states through the Fourteenth Amendment. C) The Equal Protection Clause as it applies to the states through the Fourteenth Amendment. D) The city ordinance effectively restricts interstate commerce.

(A) is correct. The employee's best defense is intergovernmental immunity. State and local governments cannot tax or regulate the activities of the federal government. This principle is often termed "intergovernmental immunity." The arrest and prosecution of a federal employee who was on the job violates this principle, which is based on the supremacy of the federal government and federal law. (B) is not a bad answer because door-to-door solicitation is protected by the First Amendment. However, at best, (B) would subject the city's actions to strict scrutiny and allow the city to prevail if it could prove that its action was necessary to achieve a compelling government purpose. In contrast, (A) would automatically invalidate the city's enforcement of the law against the employee, and so (A) is a better answer. (C) is irrelevant because the Fourteenth Amendment's restriction on the states has to do with persons, not the federal government, and here there is no claim that the city was discriminating against the employee. The city's ordinance, as briefly described, does not seem to provide the basis for an equal protection claim. (D) is wrong because nothing in the facts shows any burden on interstate commerce. Moreover, at most such a claim would trigger heightened scrutiny; it would not automatically invalidate the enforcement of the law as would (A).

A state suffering from a severe loss of tax revenues, due to an initiative that cut state sales taxes in half, enacted legislation that ended cost-of-living increases in all state employees' pensions. A state organization of employees brought suit against the appropriate state official in the federal court to reinstate the increase. What is the most likely result? A) The employees' organization will prevail, because the statute violates the prohibition against the impairment of the obligations of contracts by a state. B) The employees' organization will prevail, if it can show that the statute violates the state's constitution. C) The employees' suit will be dismissed, because the Eleventh Amendment prohibits a state's citizens from suing a state official for official acts in a federal court. D) The employees' organization will not prevail, because the state always has the power to amend its own legislation.

(A) is correct. The employees' organization likely will prevail. The Constitution prohibits the impairment of contractual obligations by a state except in certain narrow circumstances. The sort of "emergency" normally required for such state action is arguably present here, given the loss in tax revenues. But it is unlikely that the state would prevail because the termination of annual cost-of-living adjustments is permanent and appears to be the sort of self-interest driven choice to reduce the state's contractual burdens that the Court has found suspect in comparable cases. (B) is wrong because federal courts do not have the jurisdiction to decide questions regarding an individual state's own constitution. (C) is wrong because the Eleventh Amendment does not bar suits against state officials unless retroactive relief is sought. (D) is wrong because although a state may amend its own statutes, it cannot do so in such a manner as to violate constitutional prohibitions.

A federal statute just signed into law by the President provided that school districts no longer needed to recognize the tenure of elementary school teachers—all tenured teachers would lose their status and would be treated the same as nontenured teachers. The effect of the law would be to allow all tenured teachers to be fired more easily if their performance was not adequate. The law also allowed the salaries of tenured teachers to be lowered, at least until a new contract with the teachers could be negotiated. The law had a two-year grace period before it was to take effect, to give schools and teachers time to adjust to the law; however, it specifically provided that once it is in effect, school board actions under the law supersede any existing contract terms. A public elementary school district is in the first year of a three-year union contract with its teachers. The school board has stated that it plans to abolish tenured positions as soon as the law takes effect. The union, believing that numerous terms of the contract will be invalidated when the law takes effect, filed an action in federal court on behalf of the teachers, asking for an injunction to prevent the school board from abolishing tenured positions and for a declaratory judgment stating that the law is invalid. Should the federal court hear the case? A) No, because a ruling on the law at this point is premature. B) No, because the union does not have standing to sue on behalf of the teachers. C) Yes, because the federal law encourages improper interference with a contract in violation of the Contract Clause of the Constitution. D) Yes, because the teachers' rights and benefits are threatened by the law and the school board's stated plans.

(A) is correct. The federal court should not hear the case because it is not yet ripe for review. A federal court will not hear a case unless there exists a "case and controversy." This has been interpreted to mean, among other things, that a plaintiff generally is not entitled to review of his claim unless he has been harmed or there is an immediate threat of harm. This is to prevent the federal courts from hearing unnecessary actions. There is no immediate threat of harm to the union here because the law does not take effect for another two years. Before that happens, Congress might change the law or repeal it altogether, or the school board may decide to keep the old contract system after all. Thus, (A) is correct. (D) is incorrect. (B) is incorrect because the union would have standing. An association has standing if (i) there is an injury in fact to its members that would give them standing, (ii) the injury is related to the organization's purpose, and (iii) neither the nature of the claim nor the relief requested requires participation of the individual members in the lawsuit. All three of the conditions are met here; thus, the union would have standing. (C) is incorrect because the Contract Clause does not limit federal power, only state power, and because the state would be acting pursuant to a federal law here, there would be no Contract Clause violation. Moreover, even if the Contract Clause limited the state's actions here, it still is not clear that there would be a constitutional violation. The Clause bans only substantial interference with existing contracts (i.e., destruction of almost all of a party's rights under a contract), and it is not clear here that the impairments would be sufficiently substantial.

To provide low-cost housing to the unemployed, a city has a policy of leasing empty city-owned buildings to social agencies that promise to convert or rehabilitate the buildings into habitable, low-cost apartments and to pay the city 10% of any net profit made from rentals. A church entered into such an agreement with the city and converted one of the city's abandoned office buildings into 50 small, low-cost apartments. The lease agreement used by the church provides, among other things, that the lessee must affirm a belief in God. The lease agreement was submitted to the city for approval prior to its use by the church, and it was approved. On the first day that the church made the apartments available for rent, the plaintiff, an avowed atheist, applied to lease a unit. The plaintiff's application was denied for the sole reason that the plaintiff refused to affirm a belief in God. If the plaintiff brings suit against the church on the ground that the required affirmation of a belief in God violates the plaintiff's constitutional rights, who likely will prevail? A) The plaintiff, because denial of a lease to atheists has been held to hinder the free exercise of religion. B) The plaintiff, because the purpose and effect of the church's policy results in a violation of the Establishment Clause. C) The church, because freedom of religion is not protected against acts of private individuals or groups or a private institution. D) The church, because as an atheist, the plaintiff has no standing to challenge the lease requirement on religious grounds.

(B) is correct. The plaintiff will likely prevail because the policy violates the Establishment Clause. The First Amendment prohibits laws respecting the establishment of religion. Governmental action that does not contain a sect preference will pass muster under the Establishment Clause if it has a secular purpose, its primary effect neither advances nor inhibits religion, and it does not require excessive government entanglement with religion. The church's action will be considered to be state action here because of the significant involvement between the church and the city. (The city is leasing the building to the church, the church shares profits with the city, and the church submitted its lease forms to the city for approval.) Requiring a lessee to affirm a belief in God appears to have no secular purpose. Moreover, its primary effect probably is to advance religion. Therefore, the church's action will be found to have violated the Establishment Clause. (A) is wrong because there simply is no such Supreme Court holding. (C) is wrong because, as was pointed out above, state action can be found because of the city's significant involvement in the apartment building at issue. (D) is wrong because a person asserting a violation of the Establishment Clause does not have to allege infringement of a particular religious freedom in order to have standing; it is enough that the person is directly affected by the government action challenged.

A tire company is the manufacturer of, and owns the patent for, a new and unique type of truck tire. The company is the largest employer in the state in which it is located. That state enacted a law requiring all trucks using its highways to be equipped with the company's unique truck tires. An out-of-state trucking company filed suit against the state, and eventually the United States Supreme Court held that the state's statute violated the Commerce Clause. Subsequently, Congress enacted a law that required all trucks engaged in interstate commerce to use the company's unique tires. Is this federal law constitutional? A) Yes, because Congress's control over interstate commerce is very broad. B) Yes, only if Congress can demonstrate that the use of tires of another design somehow unduly burdens interstate commerce. C) No, if most states permit tires of another design. D) No, because the federal law reverses a decision of the Supreme Court, and the Supreme Court is the sole arbiter of constitutional issues.

(A) is correct. The law should be held constitutional. Congress has very broad power over interstate commerce, and it may enact laws that regulate commerce. (B) is a misstatement of the law, because Congress is not required to find that there exists some burden on interstate commerce before it enacts laws regulating commerce. (C) is wrong because it is Congress that is empowered to regulate commerce between the states. Current state laws have no effect on whether Congress can, or cannot, regulate. (D) is wrong because the federal statute did not reverse the decision of the Supreme Court. It merely enacted a requirement as a matter of federal law, while the Supreme Court had ruled that a state law on the subject was invalid.

A man committed a particularly brutal series of crimes that, because of their interstate character, were violations of a federal criminal statute. The man was convicted in federal court and sentenced to life imprisonment. Six months after the man was incarcerated, the President pardoned him. There was a great public outcry, amid charges that the President issued the pardon because the man's uncle had made a large contribution to the President's reelection campaign fund. Responding to public opinion, Congress passed a bill limiting the President's power to pardon persons convicted under the specific statute that the man had violated. The President vetoed the bill, but three-quarters of the members of each house voted to override the veto. Is the legislation constitutional? A) No, because the power to pardon for federal crimes is expressly granted to the President in the Constitution and is an unqualified power (except as to impeachment). B) No, because the President has the duty to enforce the laws, and therefore has plenary powers. C) Yes, under Article I, Section 1. D) Yes, because Congress wrote the federal criminal statutes and has the right to determine who should be convicted under such statutes.

(A) is correct. The legislation is unconstitutional because the President has exclusive power to pardon. Article II, Section 2 of the United States Constitution grants the President the power to grant reprieves and pardons for offenses against the United States, except in cases of impeachment. This pardon power is not subject to control by Congress, and it includes the power to commute a sentence on any conditions the President chooses (as long as the conditions do not offend some other constitutional provision). As applied to the facts here, Congress quite simply lacks the authority to circumscribe the President's power to pardon persons convicted under the statute at issue. (B) is incorrect because it is too broad. It is true that the President has the duty to enforce the laws, and that certain powers are part of and emanate from this duty. However, the call of this question clearly points toward the need to address the pardon power specifically, as well as any possible congressional limitations on that power. (C) is incorrect because Article I, Section 1 of the Constitution simply vests all legislative powers in the Congress. As explained above, this legislative power does not include the authority to set limitations on the President's pardon power. Consequently, the congressional action undertaken here is unconstitutional. (D) is incorrect because it also concludes that Congress is empowered to limit the pardon power. In addition, the language of (D) amounts to a bill of attainder if applied to the man pardoned. A bill of attainder is a legislative act that inflicts punishment without a judicial trial upon individuals who are designated either by name or in terms of past conduct. Both the federal and state governments are prohibited from passing bills of attainder. The assertion by Congress of the right to determine who should be convicted under a statute would amount to an attempt to pass a bill of attainder, because conviction would be imposed without a judicial trial.

A state statute prohibits merchants from selling goods manufactured in a foreign country unless the merchant clearly marks the goods with their country of origin. The United States has a treaty with a foreign country that allows each country to import and sell goods and products from the other country. If a person is prosecuted under the state law for refusing to mark goods as being of that foreign country's origin, which of the following statements reflects the most likely outcome of the case? A) The person should be acquitted because the state statute is unconstitutional. B) The person will be acquitted, but only if the treaty with the foreign country preceded the state statute in point of time. C) The person will be found guilty because the treaty with the foreign country is no defense to a criminal prosecution in state courts for violating state laws. D) The person will be found guilty because the treaty is no defense to the criminal prosecution in the absence of effectuating legislation by Congress.

(A) is correct. The person should be acquitted because the statute is unconstitutional. Like other federal law, a treaty is the supreme law of the land. Consequently, any state action or law that conflicts with a United States treaty is invalid. Some treaties are expressly or impliedly self-executing (i.e., they are effective without any implementation by Congress). Others are not effective unless and until Congress passes legislation to effectuate their ends. The federal treaty with the foreign country, pursuant to which the United States and that country agree to permit the importation and sale of goods and products from each other, does not appear to be in need of congressional action to implement its terms. The treaty on its face allows goods from the foreign country to be imported and sold in this country. With the permission for such importation and sale having thus been granted, there should be no need for any further action to bring about the terms of the treaty. Thus, the treaty is impliedly self-executing, and it has supremacy status over any conflicting state law. Because the state statute places obstacles in the way of the sale of products from foreign countries, the statute is in conflict with the treaty, and is therefore invalid. (D) is incorrect because, as explained above, there is no indication that effectuating legislation is necessary with relation to this treaty. (C) is incorrect because, if the statute under which the person is prosecuted is invalid by reason of a conflict with the federal treaty, then obviously the prosecution can proceed no further. Thus, the treaty does provide a defense to the state criminal prosecution. (B) is incorrect because a federal treaty has supremacy over conflicting state law regardless of the time sequence in which the treaty and the state statute came into being. Consequently, the person will be acquitted even if the treaty was made after the enactment of the state statute.

Congress enacted a statute that gives individuals the right to view any file that the government has assembled concerning that individual. The President was considering the appointment of a candidate as an ambassador and requested that appropriate government agencies conduct a background investigation. The investigation revealed some derogatory information, and the President declined to appoint the candidate. After his attempts to view the file were refused by White House staff, the candidate brought an action to inspect his personnel file under the aforementioned act. Which of the following presents the strongest argument that the statute is unconstitutional? A) As applied in this case, the statute restricts the President's right to frank and unfettered confidential advice. B) The statute interferes with the right of privacy of the informants. C) Congress does not have the power to give the public access to private information. D) The doctrine of sovereign immunity prevents the suit of the candidate.

(A) is correct. The question addresses the issue of executive privilege. In United States v. Nixon, the United States Supreme Court held that the President has a qualified privilege to refuse to disclose any confidential information that aids him in the performance of his duties. None of the requirements to overcome this privilege are present in this fact pattern, e.g., a court's need for the information to properly adjudicate a criminal matter. Therefore, this statute may be unconstitutional to the extent that it interferes with the executive privilege. (B) is incorrect. White House staff probably cannot raise the privacy rights of individuals who disclose information to the public. Moreover, it is unclear that these informants expect the information to remain private. Therefore, although this is an interesting argument, it is not the best answer choice when there is an argument based on clearer constitutional grounds given in another answer choice. (C) is incorrect. Under the federal property power, Congress has the power to disclose government property such as files or documents. (D) is incorrect. The question addresses executive privilege, not sovereign immunity. The federal government is not being sued for money damages; the President is being asked to turn over confidential files that might be protected by the executive privilege.

A state legislature enacted an excise tax on any automobile sold in the state that had not been manufactured within the state. The tax was intended to ease the desperate plight of the thousands of auto workers suffering layoffs, plant closures, and pay cuts from lost sales to foreign competitors. The tax was graduated, from 5% of the sales price of inexpensive automobiles down to 1% for automobiles selling for more than $100,000. A corporation that manufactures automobiles in a neighboring state brought an appropriate action in federal court to enjoin enforcement of the automobile tax statute as to its products. Which of the following is the strongest constitutional argument supporting the invalidity of the special tax? A) It is an undue burden on interstate commerce. B) It violates the Equal Protection Clause of the Fourteenth Amendment. C) It violates the Fourteenth Amendment's protection of the privileges and immunities of national citizenship. D) It violates the Due Process Clause of the Fourteenth Amendment.

(A) is correct. The strongest argument against the tax is that it burdens interstate commerce. The Commerce Clause of the Constitution gives Congress very broad power to authorize or forbid state taxation that affects interstate commerce. Unless approved by Congress, state taxes that discriminate against interstate commerce are invalid. The special tax here does not pass muster because it directly discriminates against out-of-state competition. Thus, the tax violates the Commerce Clause and (A) is correct. (B) is not a good argument because the Equal Protection Clause is not violated here. The Equal Protection Clause prohibits government from treating similarly situated people differently without good reason. What constitutes a good reason depends on the classification and the right involved. If the class is suspect or the right is fundamental, a compelling reason is required. Otherwise, the reason need only be rational. For most state or local government taxes, equal protection requires only that the tax classifications have a rational relationship to a legitimate government interest. Here, there is a rational reason for the regulation—protection of local jobs. Thus, the tax does not violate the Equal Protection Clause. (C) is incorrect because the Fourteenth Amendment clause protecting the privileges and immunities of national citizenship does not apply here. The clause protects only those rights attributable to being a United States citizen, e.g., the right to petition Congress. The clause is inapplicable here. (D) is incorrect because the Due Process Clause is not violated here. For state taxation of interstate commerce to be valid under the Due Process Clause, the benefits and protection afforded by the taxing state must have a sufficient relationship to the subject matter taxed such that it is reasonable for the taxed party to expect to be subject to the taxing state's jurisdiction. Here, the tax is imposed on autos sold in the taxing state; this is a sufficient relationship to satisfy the Due Process Clause.

A state provided for a public school system based primarily on property tax revenues from the various districts. School districts that had a property tax base below a certain threshold received supplemental funds from the state that were derived from state lottery revenues. The school districts receiving the supplemental funds served a predominantly Hispanic population as compared to the school districts funded only from property tax revenues. To help balance its budget this year, the state legislature passed a statute terminating the supplemental funds program and earmarking the lottery revenues for deficit reduction. A group of parents of Hispanic schoolchildren in one of the school districts formerly receiving supplemental funds filed suit in federal court, alleging that the state's action in terminating the funding violates the Equal Protection Clause of the Fourteenth Amendment. Which of the following best describes the appropriate standard by which the court should review the constitutionality of the state action? A) Because the state statute results in discrimination against a suspect class, the state will have to demonstrate that the statute is necessary to vindicate a compelling state interest. B) Because the right to education burdened by the statute is not a fundamental right, the parents will have to demonstrate that the statute is not substantially related to an important state interest. C) Because no suspect class or fundamental right is improperly burdened in this case, the parents will have to demonstrate that the statute is not rationally related to any legitimate state interest. D) Because the state statute is not discriminatory in intent, the state will have to demonstrate only that the statute is rationally related to a legitimate state interest.

(C) is correct. To prevail, the parents will have to show that the statute does not meet the rational basis test. Under that test, a law is presumed to be valid and will be upheld unless the challenger can make the difficult showing that it is not rationally related to a legitimate state interest. Here, the statute terminating the funds did not target a suspect class and did not burden a fundamental right, so the rational basis test applies. (A) is incorrect because it is not enough to show that legislation has a discriminatory effect on a suspect class; there must be an intent to discriminate. To establish a racial, national origin, or ethnicity classification, the party challenging the law must show that (i) the racial classification appears in the law itself (facial discrimination), (ii) the law was applied in a purposefully discriminatory manner, or (iii) the law was enacted or maintained for a discriminatory purpose. None of these situations appears to be indicated under these facts. (B) is incorrect because it states the wrong standard. As that choice indicates, the Supreme Court has not held education to be a fundamental right under the Due Process Clause, nor has it found classifications based on wealth to require strict scrutiny. Hence, the test that is applied is the rational basis standard; the standard in (B) is an intermediate scrutiny test applied to gender and legitimacy classifications. (D) is incorrect because it imposes the burden of proof on the wrong party. For a statute that does not discriminate against a suspect class, the plaintiff bears the burden of proving that the statute is not rationally related to a legitimate state interest.

Which of the following suits would not fall within the United States Supreme Court's original jurisdiction under Article III, Section 2? A) A suit seeking to assert the interest of state citizens in retaining diplomatic relations with a foreign nation. B) A suit seeking to protect a state's timber from allegedly illegal cutting by residents of another state. C) A suit seeking to enjoin enforcement of an allegedly unconstitutional executive order that will greatly limit the state's authority to make policy decisions regarding admission to state universities. D) A suit by the United States Government seeking to enjoin state construction of a bridge over a navigable waterway.

(A) is correct. The suit to assert state citizens' rights is not within the Supreme Court's original jurisdiction. Under Article III, Section 2, the United States Supreme Court has original jurisdiction in all cases affecting ambassadors, other public ministers, and consuls, and in which a state is a party. In (A), the state is not really seeking to advance or protect any interest of its own. Rather, the state is attempting to act in parens patriae (i.e., to act as a representative of its citizens, thereby asserting their interests). Thus, the state is not an actual party in this case in the sense that the Supreme Court has traditionally required to justify exercise of original jurisdiction. (B) would be a proper case for institution under the Supreme Court's original jurisdiction because it involves an attempt by a state to protect its own economic interest rather than to assert the interests of its citizens in a representative capacity. Similarly, (C) sets forth a situation in which a state is attempting to defend its asserted right to render decisions affecting admissions policies relative to its own state universities. Thus, in (C) the state is an actual party to the case. Finally, (D) describes an attempt by the federal government to prevent state construction of a bridge (presumably pursuant to the admiralty power). Clearly, this case involves an alleged grievance that will be directly committed by a state. Therefore, the state is an actual party.

A state passed a statute requiring all employers in the state to provide a medical insurance plan for full-time employees. A state trade association to which many state employers belong brought suit in federal court, asking the court to strike down the statute as unconstitutional. Assume that the state statute is not preempted by any federal law. Which of the following best reflects the burden of persuasion in this case? A) The burden is on the trade association to prove that the statute is not rationally related to a legitimate state interest. B) The burden is on the trade association to prove that the statute is not necessary to achieve a compelling state interest. C) The burden is on the state to prove that the statute is rationally related to a legitimate state interest. D) The burden is on the state to prove that the statute is necessary to achieve a compelling state interest.

(A) is correct. The trade association will have to prove that the statute is not rationally related to a legitimate state interest. Whether the statute is treated as a due process challenge (because it affects all employees), or as an equal protection problem (because employers are singled out for special treatment), the same standard will apply. Because there is no fundamental right involved and employers are neither a suspect nor quasi-suspect class, the rational basis standard will apply. Under the rational basis standard, the party challenging the government action has the burden of proving that the action is not rationally related to any legitimate state interest—a very difficult burden to meet. Thus, the burden will be on the trade association and (A) is correct. (B) is incorrect because it states the wrong standard. (C) and (D) are incorrect because they place the burden on the wrong party.

Congress approved an act that contained an appropriation of $1 million for a professor at a state university to study the effects of volcanic eruptions on the temperature of sea water. The bill contained a second appropriation of $1 million for a professor at another state university to study the effects of oil drilling on the population of bears in a national forest. The President drew a line through the first appropriation, with the intent of canceling the provision, and then signed the bill. The professor studying volcanic eruptions brought suit alleging that the action of the President was unconstitutional and therefore the striking of his appropriation was invalid. Was the President's action constitutional? A) No, because the President has no line item veto power. B) No, because the President may not veto an appropriation passed by Congress. C) Yes, because the President could have vetoed the appropriation if it were contained in a separate bill. D) Yes, because the President has the power to refuse to spend funds that have been appropriated unless the spending is specifically mandated.

(A) is correct. The veto power allows the President only to approve or reject a bill in toto; he cannot cancel part (through a line item veto) and approve other parts. The rationale is that the President's veto power does not authorize him to amend or repeal laws passed by Congress. Here, the President cannot draw a line through the first appropriation with the intent of canceling the provision and enacting the remainder of the bill. The line item veto is unconstitutional as a disruption of the balance of powers among the three branches of government. (B) is incorrect. As explained above, the President has the power to veto legislation (even appropriations) as long as he vetoes the legislation in its entirety. (C) is incorrect. It is irrelevant that the President could have vetoed the appropriation if it were contained in a separate bill. As explained above, the President can only accept the bill in its entirety or veto it in its entirety. (D) is incorrect. While it is true that when Congress has not mandated that the President spend specific money appropriated by Congress, the President has discretion not to spend it. Here, the issue is with the process of enacting a law, and even if the President would have discretion about whether to spend the money after the law is passed, that does not give the President the power to remove the line item from the bill.

A federal act both imposes limits on the amount of sulfur dioxide that can be emitted from electric power plants powered by oil and prescribes that limestone scrubbers be installed on every oil-fueled generator capable of generating more than 10 megawatts of electricity. A state operates two oil-fueled power plants. The first has a capacity of 11 megawatts, but it is used solely to power the state government's facilities in the event of an emergency. The second plant has a capacity of 1,000 megawatts, is operated by the state-owned power authority, and sells electricity both to the state and the general public. Both plants meet the sulfur dioxide emission requirements, but neither has installed limestone scrubbers. The federal entity authorized to enforce the federal act sues the state to enjoin the operation of the two plants until limestone scrubbers are installed. Which of the following is the most likely outcome of the lawsuit? A) The court will enjoin the operation of both plants, because a state must comply with valid federal laws. B) The court will enjoin the operation of only the 1,000-megawatt plant, because the 11-megawatt plant is used by the state in the performance of core state governmental functions. C) The court will not enjoin the operation of either plant, because of the doctrine of intergovernmental immunity. D) The court will not enjoin the operation of either plant, because each plant, by complying with the emission limits, meets the goals of the act, and the state has discretion to decide which means of compliance is most appropriate.

(A) is correct. There is no question that Congress, pursuant to its authority under the Commerce Clause, has the authority to regulate emissions from power plants, even if the plants are small and sell electricity only locally. Electric power plants are a commercial activity that in aggregate would have a substantial effect on interstate commerce. That authority, which was exercised in this case, extends to power plants owned by the state. Although the Tenth Amendment restricts Congress's power to regulate the states, the court will not likely strike down on Tenth Amendment grounds a regulation that subjects states or local governments to regulations that apply to both the public sector and the private sector. (B) is incorrect. As explained above, the regulations here apply to both the state and private sector and, therefore, they are a permissible exercise of the commerce power. This is true even if the plant is used only to make electricity for the purpose of running the state government. (C) is incorrect. As explained above, the state does not have immunity from federal legislation that applies both to public and private entities. (D) is incorrect. The state has no discretion as to whether to comply with all the particulars of the act. The statute sets emissions limits and requires the use of limestone scrubbers. Imposing this requirement is certainly a rational exercise of Congress's power. The federal agency, therefore, has the right and obligation to enforce the statute with respect to the limestone scrubbers.

A state has a retirement system under which a fixed percentage of an employee's pay is deducted each payday. Retirement benefits are paid at age 65 for the life of the employee. The state has obtained reliable actuarial statistics that indicate that those who completely abstain from alcohol have a greater life expectancy than those who use alcohol. Those who abstain, therefore, receive smaller monthly retirement benefits than those who use alcohol. A man who has never had a drop of alcohol in his life challenges the lower benefit standard applicable to persons in his class on constitutional grounds. What would be the appropriate burden of proof? A) The man is required to show that the benefit standard is not rationally related to a legitimate state purpose. B) The man is required to show that the benefit standard is not necessary to achieve a compelling state interest. C) The state is required to show that the benefit standard is necessary to achieve a compelling state interest. D) The state is required to show that the benefit standard is rationally related to a legitimate state interest.

(A) is correct. Where a law treats a person or class of persons differently from others, it is an equal protection question. The Court employs one of three tests in reviewing these laws: The Court uses the strict scrutiny test when a suspect classification or fundamental right is involved, intermediate scrutiny when a classification is based on gender or legitimacy, and the rational basis test whenever the other two standards are not applicable. Here, the lower benefit standard does not involve race, alienage, fundamental interests, or any other category that would trigger a heightened standard of review. It is an economic regulation, and as such, the rational basis test would be used. Under the rational basis test, laws are valid if rationally related to a legitimate state interest, and the challenger has the burden of proof. Answer (A) is correct because it properly places the burden of proof on the challenger to show that the lower benefit standard is not rationally related to a legitimate state interest. (B) is incorrect. First, when the burden of proof requires a showing of compelling interest, that burden is always on the state. Also, such a burden of proof is applicable only when the statute involves race, alienage, or fundamental interests. When the challenge involves an area of economic regulation (as is the case here), the burden is on the challenger, as described above. (C) is incorrect. Although this choice accurately describes the strict scrutiny standard, as explained above, that standard does not apply here to action not impacting a fundamental right or suspect or quasi-suspect classification. (D) is incorrect. While this answer choice correctly states that the rational basis test is the appropriate standard of review, it incorrectly places the burden of proof on the state. The plaintiff must show that the statute is not rationally related to a legitimate government interest. The state has the burden of proof only when the statute involves race, alienage, gender, legitimacy, or fundamental interests.

Congress passed a statute requiring that all beekeepers be licensed and establishing standards for the maintenance of the hives, the removal of honey, and the labelling of honey for sale. Which of the following provides the strongest basis for enactment of this legislation by Congress? A) The General Welfare Clause. B) The Commerce Clause. C) The Supremacy Clause. D) Congress's police power.

(B) is correct. Congress's power to regulate commerce is very broad and includes the power to regulate activities that have a substantial effect on interstate commerce. A regulation on intrastate activity will be upheld if it regulates an economic or commercial activity and the court can conceive of a rational basis on which Congress could conclude that the activity, in aggregate, substantially affects interstate commerce. Here, beekeeping and the sale of honey is an economic or commercial activity, and, in aggregate, it substantially affects interstate commerce. (A) is incorrect. Under the General Welfare Clause, Congress may spend to "provide for the common defense and general welfare." This spending may be for any public purpose, but nonspending regulations are not authorized by this clause. (C) is incorrect. The Supremacy Clause provides that a valid congressional act supersedes any state statute that is inconsistent with it, but the clause is not the source of any congressional regulatory power. (D) is incorrect. Congress has no general police power (power to legislate for the health, welfare, morals, etc., of the citizens). Thus, on the bar exam, the validity of a federal statute cannot rely on a general federal police power.

Congress wishes to enact legislation prohibiting discrimination in the sale or rental of housing on the basis of the sexual orientation of the potential purchaser or renter. Congress wants this statute to apply to all public and private vendors and lessors of residential property in this country, with a few narrowly drawn exceptions. Which of the following provides the strongest basis for enactment of this legislation by Congress? A) The General Welfare Clause, because the conduct the statute prohibits could reasonably be deemed to be harmful to the national interest. B) The Commerce Clause, because the sale or rental of almost all housing in aggregate could reasonably be deemed to have a substantial effect on interstate commerce. C) The enabling clause of the Thirteenth Amendment, because that amendment prohibits discrimination against badges of past discrimination. D) The enabling clause of the Fourteenth Amendment, because that amendment prohibits public and private actors from engaging in discrimination.

(B) is correct. Congress's power to regulate commerce is very broad, but it does have limits. To be within Congress's power under the Commerce Clause, a federal law must regulate (i) the channels of interstate commerce, (ii) the instrumentalities of interstate commerce and persons and things in interstate commerce, or (iii) activities that have a substantial effect on interstate commerce. When Congress attempts to regulate intrastate activity under the third prong above, the court will uphold the regulation if it is of economic or commercial activity and the court can conceive of a rational basis on which Congress could conclude that the activity in aggregate substantially affects interstate commerce. The sale and rental of housing is an economic or commercial activity and the court could find that in aggregate it substantially affects interstate commerce. (A) is incorrect. The General Welfare Clause relates to spending of federal monies: Congress may spend to "provide for the common defense and general welfare." Non-spending regulations are not authorized. (C) is incorrect. The Thirteenth Amendment abolishes slavery and involuntary servitude. The enabling clause of the Thirteenth Amendment gives Congress the power to adopt appropriate legislation proscribing private racially discriminatory acts. It would not apply to discrimination based on sexual orientation. (D) is incorrect. The Fourteenth Amendment prohibits states (not the federal government or private persons) from depriving any person of life, liberty, or property without due process and equal protection of the law. It does not apply to private actors unless the private actors are working in concert with state actors.

A state enacted a statute prohibiting the use in any medical facility in the state of any blood or blood derivative that has not been tested and declared free of HIV antibodies. The statute requires the tests to be done at a testing facility located in the state and approved by the state health commissioner. An established, reputable hospital system has a large hospital in the state, as well as hospitals in several other states. The hospital system has a long-term contractual relationship with a large, state-of-the-art blood testing facility located in another state to perform blood tests for all of its hospitals. The hospital system petitioned the state health commissioner for approval to use blood tested at the out-of-state facility. The commissioner, citing the statute, refused. The hospital system has sued to enjoin the commissioner from preventing the use of blood tested for the HIV antibodies at the out-of-state facility, alleging that the statute, as applied to it, is unconstitutional. Which party is most likely to prevail? A) The plaintiff, because the statute violates the Contract Clause. B) The plaintiff, because the statute violates the Commerce Clause. C) The defendant, because the statute does not discriminate against out-of-state competition. D) The defendant, because the statute is valid under the police power since it is directly related to public health.

(B) is correct. If Congress has not enacted laws regarding the subject, a state or local government may regulate local aspects of interstate commerce if the regulation does not discriminate against out-of-state competition to benefit local economic interests, and it is not unduly burdensome (that is, the incidental burden on interstate commerce does not outweigh the legitimate local benefits produced by the regulation). If either test is not met, the regulation will be held void for violating the Commerce Clause (sometimes called the "Dormant Commerce Clause" or "Negative Commerce Clause" under such circumstances). Here, the restriction of testing to in-state facilities is discriminatory since out-of-state facilities may be able to meet the same testing standards. The requirement of using in-state facilities is, therefore, invalid. (A) is incorrect. The Contract Clause restricts states from enacting laws that retroactively impair contract rights. However, for private contracts, if the law serves an important and legitimate public interest and is reasonably and narrowly tailored to accomplish that interest, the law may be valid despite interference with an existing contract. Here, the law serves an important and legitimate interest, but it is not clear if it is narrowly tailored to accomplish that interest. If so, the plaintiff will lose. Thus, a better reason for a decision for the plaintiff is that it is invalid on Commerce Clause grounds (see above). (C) is incorrect. As discussed above, the statute discriminates against interstate commerce, and it is, therefore, invalid. (D) is incorrect. Even though blood testing is a valid exercise of the police power, it must be done within the limitations of the negative implications of the Commerce Clause. That is, it cannot discriminate against out-of-state commerce in favor of local commerce or unduly burden commerce. As explained above, this statute discriminates against out-of-state commerce.

A state supplies U.S. history textbooks to all public high schools located in the state. The state education department has promulgated a rule requiring all high school students studying U.S. history to cover their books with a state-supplied book cover that has the state motto "Live Free or Die" on the front and a picture of the state governor on the back. A student refused to place this cover on his history book and was disciplined for violating the education department rule. The student challenged the validity of his discipline in a proper court, claiming the requirement violated his constitutional rights. Is the student likely to prevail? A) Yes, because a state cannot prescribe the type of book cover to be placed on schoolbooks. B) Yes, because a state cannot force an individual to display a message. C) No, because the words on the book cover are merely the state motto. D) No, because the books are furnished at the state's expense.

(B) is correct. The freedom of speech includes not only the right to speak, but also the right to refrain from speaking or endorsing beliefs with which one does not agree; the government may not compel an individual personally to express a message with which he disagrees. Here, the education department may not impose state-ordained speech on students. (A) is incorrect. The issue in this question is not the book covers per se, but the message the students are forced to convey. The state probably can prescribe the type of book cover, but it cannot compel the student to express a message with which he disagrees. (C) is incorrect. It is irrelevant that the words are the state motto. The government may not compel an individual personally to express a message with which he disagrees, especially where, as here, there is no compelling state interest in having the book covers on the books. (D) is incorrect. Even if the state is paying for the books (and the book covers), it has no right to impose a particular message on the students.

As part of his official duties, a chief of a state's department of employment services is expected to set forth guidelines for the payment of unemployment benefits to the state's unemployed citizens. Because of a recession and high unemployment, the state receives money from the federal government to extend the benefit period for those citizens who have exhausted their usual employment insurance benefits. The chief, however, decided that there are sufficient employment opportunities and terminated the benefits of those persons who were receiving payments from the federal funds. The citizens of the state affected by this administrative decision brought suit in federal district court against the chief for reimbursement of unpaid benefits and declaratory relief prohibiting the chief from withholding funds in the future. After a trial on the merits, the federal district court ordered that the citizens be reimbursed their lost unemployment benefits and that the chief disburse the emergency funds in the future in accordance with federal law and guidelines. The chief appealed to the federal circuit court which upheld the lower court's decision. How is the Supreme Court likely to rule on the appeal? A) Reverse the decision, because the suit is barred by the Eleventh Amendment. B) Reverse the monetary reimbursement, but uphold the declaratory relief. C) Uphold the decision, because the suit is not against the state, but against the chief personally, even though he was acting in his official capacity. D) Uphold the decision, because by accepting the federal funds, the state has waived its immunity from suit.

(B) is correct. The Eleventh Amendment prohibits a federal court from hearing a private party's claim against a state government. However, a federal court may hear an action for damages against a state officer where the effect of the action will be to force the state to pay money in the future to comply with the court order. The federal court may not order retroactive damages to be paid from the state treasury. Here, the monetary award is barred by the Eleventh Amendment because the funds would come from the state treasury, and therefore the state is the real party in interest. However, the chief may be ordered prospectively to disburse funds in conformity with federal law, and that element of this lawsuit is not barred by the Eleventh Amendment. Therefore, Answer (A) is incorrect. (C) is incorrect. A state official may be sued personally, even if he is acting in his official capacity. However, as explained above, there is an Eleventh Amendment problem in this question regarding the monetary award, because that money will be paid from the state treasury. In such a situation, the state is the real party in interest, and such relief would be barred by the Eleventh Amendment. (D) is incorrect. Acceptance of federal funds does not by itself constitute a waiver of immunity. Without an explicit requirement by Congress that a state waive its immunity in order to receive funds, simply participating in a federal program does not constitute a waiver of immunity.

A state governor proposes to place a nativity scene, the components of which would be permanently donated to the state by private citizens, in the state capitol building rotunda where the state legislature meets annually. The governor further proposes to display the state-owned nativity scene annually from December 1 to December 31, next to permanent displays that depict the various products manufactured in the state. The governor's proposal is supported by all members of both houses of the state legislature. A citizen of the state brings an action in court, arguing that the display would violate the Establishment Clause. Is the suit likely to be successful? A) Yes, because the components of the nativity scene would be owned by the state rather than by private persons. B) Yes, because the nativity scene would not be displayed in a secular context. C) No, because the components of the nativity scene would be donated to the state by private citizens rather than purchased with state funds. D) No, because the nativity scene would be displayed alongside an exhibit of various products manufactured in the state.

(B) is correct. The Establishment Clause prohibits government action respecting the establishment of religion. A government-maintained Christmas holiday display does not violate the Establishment Clause if it (i) has a secular purpose (based on the history of government recognition of holidays), (ii) has a primary nonreligious effect (it does not endorse religion), and (iii) does not create excessive entanglement between government and religion. If a government's holiday display includes religious symbols (a nativity scene or a menorah) as well as other holiday decorations (a Christmas tree or a Santa Claus figure), the courts will allow the display. If the display includes only symbols of a particular faith, it will violate the Establishment Clause because it has a religious effect (it "endorses" religion). Here, the only item in the display would be a nativity scene—a symbol of Christianity. Therefore, the display would be held unconstitutional. (A) and (C) are incorrect. The constitutionality of a nativity scene in a government building as related to the Establishment Clause is based on the context of the display (see above), not on who purchased or owns the components. (D) is incorrect. As explained above, a Christmas display of items depicting the historic roots of the American holiday called Christmas is constitutional, but one focusing on a a particular faith is not. The fact that the nativity scene would be near items manufactured in the state would do nothing to alleviate the appearance of the endorsement of a particular faith that would result from having only a nativity scene in a government Christmastime display.

A state with a number of automobile manufacturing facilities within its borders and a high unemployment rate because of declining sales of automobiles, especially ones built domestically, enacted a statute calling for a $100-per-car tax on all foreign-built automobiles sold within the state. The tax revenues were to be placed into a state fund to be used to retrain the state's unemployed automobile workers. A major automobile importer and dealership owner brings suit in federal district court seeking to halt the enforcement of the statute on constitutional grounds. Should the court find the statute to be constitutional? A) Yes, because it is a proper exercise of a state's rights under the Import-Export Clause. B) Yes, if consented to by Congress. C) No, because the statute violates the Privileges and Immunities Clause of Article IV. D) No, unless the state can show that the statute is necessary to promote a compelling state interest.

(B) is correct. The court should find the statute constitutional as long as Congress has consented to the tax. Article I, Section 10, Clause 2 provides: "No state shall, without the Consent of the Congress, lay any Imposts or Duties on Imports or Exports, except what may be absolutely necessary for executing its inspection Laws." Hence, the Import-Export Clause prohibits the states from imposing any tax on imported goods as such or on commercial activity connected with imported goods as such (i.e., taxes discriminating against imports), except with congressional consent. Thus, (B) is correct and (A) is incorrect. (C) is wrong because the Privileges and Immunities Clause does not apply to corporations or aliens. (D) is wrong because without congressional consent, the statute is unconstitutional despite the compelling nature of the state's interest.

As part of legislation enacted for the stated purpose of improving science skills of schoolchildren, Congress appropriated funds to permit public school teachers who had been certified by state school districts as science lab instructors to provide supplemental science instruction to any students in either public or private schools who did not have access to science lab resources. To help ensure content neutrality, the statute required the instructors coming to the private schools to use portable science labs supplied by the public school districts, which contained the equipment and experiments that the instructors used for the same purpose in the public schools. A citizens' group filed suit in federal district court to challenge the constitutionality of funding the science teachers for private schools, alleging that most of the private schools covered by the statute were religiously affiliated schools. No members of the group have any children in either public schools or private schools affected by the statute. How is the court likely to rule? A) Dismiss the case on the pleadings, because the citizens' group does not have a sufficient stake in the controversy to have standing to challenge Congress's expenditure, which was authorized under its power to spend for the general welfare. B) Decide the case on the merits in favor of the government, because the legislation defines the context in which instruction can be provided in private schools so as to avoid excessive government entanglement with religion. C) Decide the case on the merits in favor of the citizens' group, because the appropriation's primary effect advances religion in violation of the Establishment Clause of the First Amendment. D) Decide the case on the merits in favor of the citizens' group, because the court will presume that any instruction provided on the premises of a religiously affiliated school will be influenced by religion.

(B) is correct. The court will probably decide in favor of the government on the merits. Programs of aid to religiously affiliated grade schools and high schools are subject to the same three-part test as are other laws under the Establishment Clause: The program must (i) have a secular purpose, (ii) have a primary effect that neither advances nor inhibits religion, and (iii) not produce excessive government entanglement with religion. With respect to the first prong of the test, most of the time such programs (including this one) will have a secular purpose—to aid education. With respect to the second prong, the program may be deemed to have a primary effect that advances religion if it results in governmental indoctrination of religion or defines its recipients by reference to religion. Here, the statute establishes a religiously neutral program that funds a supplemental service for the schools, and offers the instruction to all disadvantaged students regardless of whether they choose to attend public or private schools. [SeeAgostini v. Felton (1997)—government program providing remedial education services to all disadvantaged children at their schools, including children at parochial schools, held not to violate the Establishment Clause] Thus, (C) is incorrect. (D) is incorrect because the courts will not presume that the instruction provided by this program will be influenced by religion. Furthermore, with respect to the "excessive entanglement" prong of the test, there is no indication that the program requires detailed monitoring of the government employees to prevent them from incorporating religion in their instruction—the equipment and experiments that they use in the private schools are the same as they use in the public schools. (A) is incorrect because the citizens' group has standing to challenge the expenditure on behalf of its members, who have a right to sue based on their status as federal taxpayers. The one recognized exception to the rule that people do not have standing as taxpayers to challenge the way tax dollars are spent by the federal government is if the expenditure was enacted under Congress's taxing and spending power and allegedly exceeds the specific limitation on that power found in the Establishment Clause. That exception applies here because the citizens' group is alleging that the federal appropriation is an unconstitutional attempt to provide government funds to religiously affiliated schools.

The President of the United States accepted an invitation to give a commencement address at a small, Midwestern university in its auditorium. Pursuant to school rules, no one is permitted to bring posters, banners, or signs of any kind larger than the size of a piece of notebook paper into any event at the auditorium. The main purpose of the rule is to prevent obstruction of the view within the auditorium. Nevertheless, at the commencement ceremony, a student in a front row balcony seat unfurled a banner that he had hidden in his coat with a message supporting the President. The student was promptly arrested and charged in municipal court with trespassing. The student filed suit in federal court to enjoin the municipal prosecution and to have the trespass ordinance declared unconstitutional as applied to him. Will the federal court likely hear the student's case? A) Yes, if he argues that the trespass ordinance is invalid on its face. B) Yes, if he argues that the prosecutor had no hope of conviction and was proceeding to harass him. C) No, if the prosecution argues that the student lacks standing. D) No, if the prosecution argues that the case involves a political question.

(B) is correct. The federal court will hear the student's case if the student argues that the prosecutor was merely trying to harass him. The federal courts generally will abstain from enjoining pending state criminal proceedings, such as the one here, even if they have jurisdiction over the case. For purposes of this rule, a case is deemed to be pending as soon as it is filed. Thus, the federal court would ordinarily not hear the student's claim here until after the state prosecution has ended. However, there is an exception to the general rule—a federal court will hear an action to enjoin a pending state court prosecution if it is being conducted in bad faith; e.g., merely to harass the defendant. Thus, (B) is correct. (A) is incorrect because the abstention rule does not distinguish between claims that ordinances are invalid on their face and claims that they are invalid as applied. (C) is incorrect because the student clearly has standing. A person has standing to challenge a governmental action if he can show an injury in fact from the action that is greater than the injury everyone suffers when the government acts unlawfully, and that a decision in his favor will remedy the injury. If the ordinance here is being applied unconstitutionally, the student has suffered a sufficient injury, and a decision in his favor will remedy his injury. (D) is incorrect because whether an ordinance is being applied in an unconstitutional manner is not a political question. Political questions are those committed by the Constitution to another branch of the government or that are inherently incapable of resolution and enforcement by the judicial process. This case is properly within the judiciary branch and the courts can resolve it.

A special prosecutor was appointed to investigate allegations that a presidential aide accepted bribes to secure the appointment of certain federal judges. The special prosecutor has subpoenaed all memoranda written by the aide on the subject of judicial nominations to vacancies. The President has government attorneys move to quash the subpoena on the ground of executive privilege. How should the trial judge rule on the motion? A) Deny the motion, because the President has no greater privilege in this matter than any other officer of the government. B) Deny the motion, because although there is a presumption of a privilege, it can be overcome by a strong showing of the needs of the criminal justice system. C) Grant the motion, because executive privilege is an absolute bar to the requested information, except in the case of an impeachment of the President. D) Grant the motion, because executive privilege is an absolute bar to the requested information, as the President is entitled to confidential advice from subordinates.

(B) is correct. The scope of executive privilege established in United States v. Nixon extends without qualification to military and diplomatic secrets and with qualification to all other confidential communications between the President and his advisors. This qualified privilege, however, can be overcome if the request comes in the form of a subpoena asking for very specific information and there is a conflicting consideration that outweighs the President's interest in confidentiality, e.g., the need for the court to have available all relevant information in a criminal proceeding. If the claim of privilege is denied, the material must first be submitted to a judge for an in-camera inspection, during which all irrelevant material must be excised. (A) is incorrect. Every confidential communication between the President and his advisors is presumptively privileged. (C) and (D) are incorrect. The President has an absolute privilege only in the areas of foreign diplomacy and defense.

Congress provided by statute that any state that does not enact a ban on texting while driving on state highways within the next three years shall be denied 10% of federal highway construction funding. What is the best argument that can be made in support of the constitutionality of this federal statute? A) The states ceded their authority over highways to the national government when the states accepted federal grants to help finance the highways. B) The federal government can regulate the use of state highways without limitation because the federal government paid for most of their construction costs. C) The requirement is not unduly coercive and is related to making travel on highways safer. D) A recent public opinion survey demonstrates that 90% of the people in this country support the ban.

(C) is correct. The best argument is that the requirement is not unduly coercive and is related to making travel on highways safer. Congress may "regulate" states through the spending power by imposing conditions on the grant of money to state or local governments. Such conditions will not violate the Tenth Amendment merely because Congress lacked the power to directly regulate the activity that is the subject of the spending program if the conditions (i) are clearly stated; (ii) relate to the purpose of the program; and (iii) are not unduly coercive. Here, the condition is clearly stated and appears to be related to a goal of the highway program—safer highway travel. While there is not a brightline test for when a condition is unduly coercive, withholding just one-tenth of a state's federal highway funds would likely be considered reasonable. Hence, (C) states the best argument in support of the statute. (A) is incorrect because it is too broad. States accepting federal grants still retain the bulk of their authority over state highways. (B) is also too broad and assumes facts not stated in the question. (D) is irrelevant to the constitutionality of the statute.

A husband and wife were both professors at a federal military academy. The husband made a speech criticizing United States foreign policy with respect to a foreign country and was dismissed from his teaching position soon thereafter. Six months later, he accepted new employment in another state. The wife then commenced suit in federal court, claiming that the academy violated her husband's right to due process and his right of free speech when it fired him. If the court dismisses the suit, which of the following presents the most likely ground for the dismissal? A) The political question doctrine, because foreign policy is in the domain of the executive branch. B) Mootness, because the husband has a new job. C) The plaintiff's lack of standing. D) The lack of a federal question.

(C) is correct. A federal court will not decide a constitutional challenge to a government action unless the person who is challenging the government action has standing to raise the constitutional issue. To have standing, the claimant must have suffered a direct impairment of her own constitutional rights. A plaintiff may assert third-party rights only where she herself has suffered injury and (i) third parties find it difficult to assert their own rights, or (ii) the injury suffered by the plaintiff adversely affects her relationship with third parties, resulting in an indirect violation of their rights. Here, the wife is attempting to assert a violation of her husband's constitutional rights and we have no facts to support the exceptions for third-party standing. (A) is incorrect. No political question is raised here. The federal courts will not decide political questions, that is, questions committed by the Constitution to another branch of government or those inherently incapable of resolution and enforcement by the judicial process. Here, the issue is not the substance of United States foreign policy, which would be a political question, but rather the husband's First Amendment right to speak out about it, which is not a political question. Therefore, the court is not barred from litigating this issue by the political question doctrine. (B) is incorrect. The husband's new job does not render the issues moot. A federal court will not hear a case that has become moot; a real, live controversy must exist at all stages of review, not merely when the complaint is filed. Here, the husband lost his job. If he was fired in violation of the Constitution, that injury still exists whether or not he found a new job. (D) is incorrect. The subject matter of the suit, the dismissal of an employee for exercising his First Amendment rights, presents a federal question.

In compliance with a federal statute that permits government agencies to sell or give away surplus government property, the Secretary of State directed that one of the State Department's surplus airplanes be given to a church. The Secretary knew that the church planned to use the plane to fly medical supplies to its missions in Third World countries. These missions provide medical assistance, but they also attempt to evangelize residents of the countries in question, and the Secretary was aware that, in addition to medical supplies, the plane might transport Bibles and religious tracts translated into local languages. Had the Secretary not ordered the plane to be given to the church, it would have been sold at a very reasonable cost to a nonprofit organization that helps teach young people the fundamentals of piloting and maintaining aircraft. Which of the following parties would be most likely to have standing to sue to prevent the Secretary of State from making the gift to the church? A) A taxpayer. B) A citizen of the United States. C) A member of the nonprofit flying organization. D) The attorney general of the state in which the airplane is located.

(C) is correct. A member of the nonprofit flying association is most likely to have standing to challenge the gift. To have standing to challenge government action on constitutional grounds, a person must show that he has a concrete stake in the outcome of the litigation. This is to ensure adequate presentation of the issues. To have such a stake, the potential litigant must show that he has an injury in fact caused by the government that is more than the theoretical injury that all persons suffer when the government engages in unconstitutional acts, and that a decision in his favor will eliminate his harm. A member of the flying association can show both components here: If the gift is unconstitutional, the association has suffered more than a theoretical injury—it has lost the opportunity to purchase the airplane from the federal government at a good price, and a decision in the club's favor will eliminate the injury because it will then be able to purchase the plane. Thus, the member of the nonprofit flying organization has standing. (B) is incorrect because the only injury that a citizen would suffer here is the theoretical injury that we all suffer from the government's unconstitutional acts. People have no standing merely as "citizens" to claim that government action violates federal law or the Constitution. (A) is incorrect because a person's injury as a taxpayer is generally held to be insufficient to establish standing. There is an exception where the federal government acts under the taxing and spending power and that action allegedly violates the Establishment Clause, but the government action here falls under the Property Clause and not the Spending Clause; thus, the exception does not apply. (D) is incorrect because the state attorney general has no stake in the outcome of the litigation, and it is not sufficient even if he is deemed to represent the interests of all the citizens in the state.

A hairdresser owns a beauty salon catering to female customers. The hairdresser employs only male hairdressers at her salon. A city ordinance makes it unlawful for any person to operate a beauty salon if the hairdressers provide services to clients of the opposite sex. The hairdresser brings an action in federal court challenging the constitutionality of the ordinance. If the city moves to dismiss the lawsuit, arguing that the hairdresser lacks standing, is the city likely to prevail? A) Yes, because the ordinance does not prohibit the operation of beauty salons per se, only the right of hairdressers to service customers of the opposite sex. B) Yes, because only the employees can raise their rights of association. C) No, because the hairdresser has rights that are harmed by the ordinance. D) No, because the city cannot prove the ordinance serves a substantial governmental interest.

(C) is correct. A person has standing only if she can demonstrate a concrete stake in the outcome of the controversy. She must be able to show an injury in fact caused by the government that will be remedied by a decision in her favor. Here, the city will not succeed in having the lawsuit dismissed. The hairdresser (as well as her employees) will suffer direct economic harm by the statute, and that injury can be remedied if the court rules in the hairdresser's favor. Therefore, she has standing to contest the ordinance. (A) is incorrect because standing exists. While the statute does not prevent the hairdresser from operating a beauty shop, it does restrict the sex of the employees who may be employed. Such a restriction constitutes sufficient harm to confer standing on the hairdresser, because it affects who the hairdresser may hire. Therefore, the suit will not be dismissed. (B) is incorrect. The owner's right of association is affected as well. Moreover, the hairdresser's right to operate her beauty shop without unlawful restriction is also at issue. Therefore, the hairdresser can assert a sufficient injury. (D) is incorrect. The issue here is standing and this choice does not address that issue. Instead, the choice addresses a substantive issue: whether the ordinance serves a substantial government interest. The substantive issue would be resolved only if the plaintiff has standing.

A city that lies astride a major interstate highway recently passed a referendum declaring itself a "nuclear-free zone." The referendum included a provision making criminal any importation of specified nuclear materials into the city limits. The law was immediately challenged in federal court by an interstate trucking firm that regularly transported prohibited nuclear materials through the city on the highway. The case ultimately reached the United States Supreme Court, which held that the challenged ordinance was constitutional because the city had a rational basis for concluding that the citizens of the city would be safer if the prohibited materials were kept outside of town, and because the ordinance did not unduly burden interstate commerce. Many other towns and cities throughout the nation considered similar enactments after the Supreme Court decision was announced. In response, Congress enacted a federal statute prohibiting the state regulation of interstate transportation of nuclear materials. If the statute is challenged in federal court, is it likely to be held constitutional? A) No, because the Supreme Court has already ruled that local governments may prohibit specified nuclear materials from crossing their borders. B) No, because the disparate treatment of interstate versus intrastate carriers of nuclear materials would violate the Equal Protection Clause of the Fourteenth Amendment. C) Yes, because Congress has very broad power to regulate interstate commerce. D) Yes, because the Supremacy Clause requires that state enactments bow to conflicting federal legislation.

(C) is correct. Congress's power over interstate commerce is broad, and the federal statute is clearly authorized by that power. The Court's decision has no bearing here, because it was simply explaining what the standard was in the absence of any express congressional action. (A) is wrong because the fact that a state regulation does not burden interstate commerce in the absence of federal regulation does not preclude the federal government from subsequently enacting conflicting regulations that void the state law. (B) is wrong because the Fourteenth Amendment does not bind the federal government, and because a regulation of interstate commerce would rarely be classified as violating equal protection as to intrastate commerce. (D) is not the best answer because the Supremacy Clause alone does not speak to the validity of a federal statute, merely to its precedence over conflicting state enactments. The Supremacy Clause would operate, accordingly, only after the new statute is enacted, and would render the city ordinance invalid.

The United States Surgeon General was cited for contempt for refusing to answer questions as part of a Senate investigation regarding an issue in the Food and Drug Administration. His contempt citation will be dismissed if he can show which of the following? A) As a member of the executive branch, he is immune from prosecution. B) If he answered the questions, he could be subject to dismissal from his position as Surgeon General. C) The questions do not relate to any matter concerning which the Senate may legislate. D) The questions do not relate to any matter concerning current or planned legislation.

(C) is correct. His contempt citation will be dismissed if he can show that the questions do not relate to any matter concerning which the Senate may legislate. Congress's power to investigate is limited to matters on which it can legislate. Therefore, if the Surgeon General can demonstrate that the questions concerned matters upon which Congress could not legislate (not an enumerated power under Article I, Section 8), then this contempt citation must be dismissed. (A) is wrong because Congress can question a member of the executive branch concerning his duties, and he is not immune from prosecution. (B) is wrong because he would have a privilege not to answer only if he is subject to criminal liability. Merely because he may get fired is not sufficient grounds for him to refuse to answer a lawful question posed by a member of the Senate in an appropriate hearing. (D) is wrong because it is too narrow. As explained above, Congress may investigate any matter on which it may legislate; current or planned legislation is not required.

After a state legislature is presented with compelling evidence that insurance companies that offer burglary insurance policies are charging people different premiums based on race, the legislature passes a statute requiring every insurance company offering burglary insurance policies in the state to charge a uniform rate for customers who reside in the same county in the state. As long as it complies with this requirement, a company is free to charge whatever rate the market will bear for its burglary insurance policies. There are no federal laws covering uniform rates. An insurance company that offers burglary policies in the state but that is located in a neighboring state has filed suit in federal district court against state officials to challenge the statute on constitutional grounds. The insurance company wishes to charge customers residing within the same county varying rates for burglary insurance policies. What is the likely result of the suit? A) The statute will be held unconstitutional, because it is not necessary to achieve a compelling interest. B) The statute will be held unconstitutional, because it imposes an undue burden on interstate commerce. C) The statute will be held constitutional, because it is a reasonable exercise of the state's police power. D) The statute will be held constitutional, because it is narrowly tailored to achieve the compelling interest of remedying racial discrimination.

(C) is correct. If Congress has not enacted laws regarding the subject, a state or local government may regulate local aspects of interstate commerce if the regulation does not discriminate against out-of-state competition to benefit local economic interests, and it is not unduly burdensome (that is, the incidental burden on interstate commerce does not outweigh the legitimate local benefits produced by the regulation). If either test is not met, the regulation will be held void for violating the Commerce Clause (sometimes called the "Dormant Commerce Clause" or "Negative Commerce Clause" under such circumstances). Here, the state has a right to regulate the insurance industry in the state, and insurance rates in particular, under its police power. The state policy requiring that every company maintain uniform burglary insurance rates within a county does not violate the Commerce Clause, because it does not burden out-of-state companies any more than in-state companies and does not discriminate against out-of-state companies. (A) is incorrect. When a state law creates a distinction based on a suspect classification, such as race, the law is invalid under the Equal Protection Clause unless it is necessary to achieve a compelling state interest. But this law does not create a distinction based on race. The law is race-neutral on its face and was not passed for the purpose of creating a race-based distinction. Accordingly, the law does not need to satisfy strict scrutiny to survive. (B) is incorrect. As explained above, the state policy requiring that every company maintain uniform burglary insurance rates within a county does not violate the Commerce Clause, because it does not burden out-of-state companies any more than in-state companies and does not discriminate against out-of-state companies. (D) is incorrect. This choice also suggests that the strict scrutiny test applies. As discussed above, it does not.

A federal statute prohibits discrimination based on age for all persons between the ages of 50 and 70 in matters relating to employment. A state constitutional provision requires that all state judges retire at the age of 65. A judge in the state trial court reached his 65th birthday, and the governor of the state named his replacement. The judge refused to retire and brought suit in federal court to enjoin the governor from replacing him on the bench. Who is likely to prevail in the lawsuit? A) The state, because the Tenth Amendment prohibits the federal government from interfering with matters central to the administration of state government. B) The state, because the Eleventh Amendment prohibits the suit. C) The judge, because the Supremacy Clause invalidates the state constitutional provision. D) The judge, because the Fourteenth Amendment Equal Protection Clause protects the judge from discrimination.

(C) is correct. If state and federal laws cover the same subject matter, the Supremacy Clause provides that the federal law is supreme, and the conflicting state law is rendered void. In other words, a valid act of Congress or a federal regulation supersedes any state or local action that actually conflicts with the federal rule whether by commanding conduct inconsistent with that required by the federal rule or by forbidding conduct that the federal rule is designed to foster. Here, the state constitutional provision that requires all state judges retire at the age of 65 is inconsistent with the federal statute which prohibits discrimination in employment on account of age for all persons between the ages of 50 and 70. Thus, the judge would prevail because of the Supremacy Clause. (A) is incorrect. The court will not likely strike down on Tenth Amendment grounds a regulation that subjects state governments to regulations that apply to both the public sector and the private sector. Here, the federal statute is applicable to both private and state entities. Thus, it will be constitutional, even if it has the incidental effect of interfering with the administration of state governments. (B) is incorrect. The Eleventh Amendment is a jurisdictional bar that prohibits a federal court from hearing a private party's or foreign government's claims against a state government without the state's consent. However, there are exceptions. For example, a federal court may enjoin a state officer to refrain from future actions that violate federal law or to take prospective actions to comply with constitutional mandates. Here, the Amendment would not bar an order to enjoin the governor from enforcing the state statute that is contrary to federal law. (D) is incorrect. The Fourteenth Amendment Equal Protection Clause may be in issue when a state law or regulation treats a person or class of persons differently. The standard used to determine if the law or regulation is valid depends on whether a fundamental right is involved or whether a suspect or quasi-suspect class is involved. If not, then a rational basis standard is used, and the law or regulation will be upheld unless it bears no rational relationship to any conceivable legitimate government interest. Here, the Equal Protection Clause will not be helpful to the judge. Age is not a suspect or quasi-suspect classification, and no fundamental right is involved. More importantly, the Equal Protection Clause is not needed. The judge will prevail under the statute, which is a valid exercise of Congress's commerce power.

Police departments of bordering states have been cooperating in an investigation of a man who appeared to be selling illegal drugs. They had evidence that the man would receive drugs at an office he rented in State A and sell them out of an apartment he rented in State B. He would then deposit the proceeds of the sales into a bank located in State A. Once enough evidence was gathered, the man was arrested by police officers in State A. Each state has a forfeiture statute allowing the state to seize proceeds of drug sales. State A seized the proceeds of the man's bank account. State B requested a share of the proceeds, but its request was denied. Officials in State B file suit in the United States Supreme Court, demanding a share of the drug proceeds from the banking state. Does the United States Supreme Court have jurisdiction to decide this issue? A) No, because the suit must be brought in state court and pursued through the usual appellate channels before a decision of the highest state court may be appealed to the United States Supreme Court. B) No, unless both states agree. C) Yes, even if a state is named as the party-defendant. D) Yes, as long as the suit is not brought against State A but rather against a State A official to prevent that official from taking some action.

(C) is correct. The Constitution provides the United States Supreme Court with original jurisdiction in suits between states. This jurisdiction is exclusive to the Supreme Court. Therefore, the two states in this case must file this suit directly in the Supreme Court. A state official need not be named the party-defendant, because this is not a case where an exception to the Eleventh Amendment is required. The Eleventh Amendment (and thus, the exception) applies only when a private party brings suit against a state in federal court. This is instead a suit between two states, which is within the exclusive original jurisdiction of the United States Supreme Court. Therefore, (A) is incorrect. (B) is incorrect. Because this case falls under the exclusive original jurisdiction of the United States Supreme Court, it must be filed in the United States Supreme Court; it is not a matter subject to the states' discretion. (D) is incorrect. The Eleventh Amendment prohibits a private party from suing a state in a federal court. An exception, described in essence in this answer choice, permits a private party to sue a state official to enjoin or force some action on that official's part to conform to federal law. However, the Eleventh Amendment is not applicable here, because this is not a suit between a private party and a state; rather, it is a suit between two states over proceeds of a crime.

To combat the problem of obesity among the nation's youth, a state adopted a statute providing funds to train a corps of nutritionists who could go to schools and run six-week seminars designed to encourage healthy eating and exercise habits. The administration of a private school that has a stated policy of admitting only white students has requested that a state nutrition corps member be sent to the school to run a six-week seminar. Which of the following is the best constitutional argument supporting the state's rejection of the request? A) The Constitution prohibits private discrimination. B) The state may not aid private schools. C) Segregation would be furthered by the provision of the seminar. D) No legitimate educational function is served by giving nutrition seminars at private schools.

(C) is correct. If the state provides a nutritional corps member to a segregated school, it would represent a significant involvement in activities of a school that practices segregation and would thus violate the Equal Protection Clause. Thus, answer (C) is correct because segregation would be furthered by the provision of a nutritional corps seminar. (A) is incorrect because it is too broad. The Constitution prohibits slavery, and the Supreme Court has interpreted the enabling clause of the Thirteenth Amendment to give Congress the power to outlaw anything that it sees as an incident or badge of slavery. However, here we are dealing with state legislation, and nothing in the facts indicates that Congress has found the discrimination here to be a badge of slavery. (B) also is a gross overstatement. Government may aid private schools as long as such aid does not run afoul of the Constitution (e.g., does not support ethnic, racial, or religious discrimination). (D) is incorrect because that is not the test the courts would apply here.

A state imposed a tax on the "income" of each of its residents. As defined in the taxing statute, "income" includes the fair rental value of any automobile provided by the taxpayer's employer for the taxpayer's personal use. The federal government supplies automobiles to some of its employees, who are residents of the state. The federal government allows the employees to use the automobiles for both official and personal use. There is no federal legislation on this subject. May the state collect the tax from the federal employees on the fair rental value of the personal use of the automobiles furnished by the federal government? A) No, because a tax on the federal employees is essentially a tax on the United States. B) No, because the tax would be a tax on federal property as the automobiles are owned by the federal government and primarily used by federal employees in the discharge of their official duties. C) Yes, because the tax is imposed on the employees, rather than on the United States, and the tax does not discriminate against persons who are employed by the United States. D) Yes, because an exemption from such state taxes for federal employees would be a denial to others of the equal protection of the laws.

(C) is correct. Nondiscriminatory, indirect taxes on the federal government or its property are permissible if they do not unreasonably burden the federal government. For example, state income taxes on salaries of federal employees are valid. Here, the state may collect the tax from the federal employees because the tax on the fair rental value of the personal use of the automobiles furnished by the federal government is a nondiscriminatory, indirect tax to which every resident is subject. (A) is incorrect. Taxation of federal employees of income derived from employment is a tax upon the employee, not the United States. (B) is incorrect. A state tax levied directly against the property or operation of the federal government without the consent of Congress is invalid. However, the tax here is on income of federal employees and not on the United States government. It therefore is valid. (D) is incorrect. The answer choice is nonsensical. The law that imposes the tax does not treat federal employees differently from anyone else. It attempts to treat everyone whose employer provides them with a vehicle equally. Thus, there is no equal protection issue raised.

A state statute imposes limits on the amount of sulfur dioxide that can be emitted from electric power plants powered by oil and prescribes that limestone scrubbers be installed on every oil-fueled generator capable of generating more than 10 megawatts of electricity. There is no contradictory federal statute on this matter. The federal government owns an electric power plant powered by oil in the state. The plant is used solely to power the local federal government facilities and is operated by a private corporation under a contract with the federal government. The plant has a capacity of 11 megawatts, but it meets the sulfur dioxide emissions requirements. It has not installed limestone scrubbers. The state prosecutes the private operator of the plant for violation of the state statute. Is the operator likely to be convicted? A) Yes, because the regulation of pollution is a legitimate state police power concern. B) Yes, because the regulation of pollution is a joint concern of the federal government and the state and, therefore, both may regulate conduct that causes pollution. C) No, because the operations of the federal government are immune from state regulation in the absence of federal consent. D) No, because the power plant is not in substantial violation of the state pollution control standards.

(C) is correct. States have no power to regulate the activities of the federal government unless Congress consents to the regulation. Thus, instrumentalities and agents of the federal government are immune from state regulations relating to performance of their federal functions. In this question, the federal government owns the power plant that the state statute seeks to regulate. The federal government is immune from state regulation, unless it consents to regulation, which is not the case here. (A) is incorrect. Although the regulation of pollution is a legitimate state police power concern, as explained above, the state cannot regulate a federal activity absent federal consent. (B) is incorrect. The regulation of pollution does concern both the state and the federal government, but the state cannot use its police power to regulate a federal activity absent federal consent. (D) is incorrect. It does not matter whether the violation of the law is great or de minimis. The federal activity enjoys intergovernmental immunity because the federal government is immune to state regulation, unless it consents to regulation.

While vacationing, an African-American man entered a private restaurant in a remote town. All other customers in the restaurant at the time were white males who lived within a few miles of the restaurant. The man was refused service. The restaurant purchases a substantial portion of its food from local suppliers, who obtain their goods from a variety of sources, both in-state and out-of-state. The state has no statute prohibiting this type of discrimination. Congress, however, has passed a statute prohibiting restaurants from refusing service based on race and allowing private actions for damages for violations of the statute. Assuming the man can establish a statutory violation, what is the likely outcome of the suit? A) The man likely will prevail, even if Congress had not acted, because this constitutes a denial of equal protection of the laws under the Fourteenth Amendment. B) The man likely will prevail, even if Congress had not acted, because the activities of the restaurant violate the privileges and immunities granted to all citizens under the Fourteenth Amendment. C) The man likely will prevail, because Congress has enacted the statute. D) The man likely will not prevail, because this is local activity which cannot be regulated under the commerce power.

(C) is correct. The Commerce Clause permits federal regulation of almost all types of local economic or commercial activity if there is a rational basis to conclude that the activity, in aggregate, substantially affects interstate commerce. Here, operating a restaurant is a commercial activity and the restaurant's food comes from interstate commerce. Therefore, a court could find that discrimination in restaurants impedes interstate travel. (A) is incorrect. The Fourteenth Amendment does not prohibit private acts of racial discrimination. State action is required for the Fourteenth Amendment to apply. Since the Fourteenth Amendment does not provide a cause of action against private individuals, the man cannot prevail for this reason. (B) is incorrect. The Privileges and Immunities Clause of the Fourteenth Amendment does not provide a cause of action against private individuals. It prohibits states from interfering with attributes of national citizenship. Thus, the clause is not a basis for the man's suit because it is inapplicable in a suit against a private individual. (D) is incorrect. As explained above, the man would prevail in this case because even intrastate activity can be regulated under the Commerce Clause if the activity in aggregate substantially affects interstate commerce. If many restaurants in a region all refused service to certain races, it would make it very difficult for people of those races to travel in those regions. Thus, a restaurant's refusing service, in the aggregate, can substantially affect interstate commerce.

The Vice President conspired with various Pentagon and CIA officials to sell military weapons to a foreign country. When the President learned of these activities, he launched a full-scale investigation into the matter. The President presented his findings to Congress. The House of Representatives, by a one-vote majority, voted to impeach the Vice President. The Senate, by a one-vote majority, voted to remove the Vice President from office. If the Vice President refuses to leave office, what is her best argument for retaining her position? A) Only the President, not the Vice President, may be impeached. B) A two-thirds vote of both Houses of Congress is required to convict. C) The Senate did not have enough votes to convict her. D) The Senate, not the House of Representatives, is the appropriate body to impeach an executive officer.

(C) is correct. The United States House of Representatives has the sole power to impeach, an action akin to indictment in criminal law. Once the House impeaches, the case is tried in the United States Senate. A two-thirds vote is necessary to convict and remove the person from office. Here, there was only a majority vote; thus, the Vice President may retain her position. (A) is incorrect. The Constitution provides that the President, Vice President, and all civil officers of the United States can be removed from office by impeachment and conviction. Therefore, the Vice President can be impeached. (B) is incorrect. A two-thirds vote of the United States Senate is required for removal, but only a majority vote of the House is necessary for impeachment. (D) is incorrect. As explained above, the House of Representatives has the sole power to impeach, which is akin to an indictment in criminal law. Once the House impeaches, the Senate tries the case.

A state Occupational Health and Safety Board recently issued regulations valid under its statutory mandate requiring that all employers in the state provide ionizing air purification systems for all employee work areas. These regulations replaced previous guidelines for employee air quality that were generally not mandatory and did not specify the method of air purification used. The requirements regarding air purification systems are likely to be unconstitutional as applied to which of the following employers? A) A wholly owned subsidiary of a Japanese corporation with seven retail outlets within the state. B) The state supreme court, which recently completed construction of its new courthouse with a non-ionizing air purification system which the builder is contractually bound to maintain for the next three years. C) A United States Armed Forces Recruiting Center located adjacent to the state capitol building. D) A privately operated community service center funded by donations and constructed through use of a loan provided by the United States Veterans Administration and repayable to that agency.

(C) is correct. The armed forces recruiting center is least likely to be required to comply with the new state law. A state has no power to regulate activities of the federal government unless Congress consents to the regulation. Accordingly, agents and instrumentalities of the federal government, such as the armed forces recruiting center, are immune from state regulations relating to performance of their federal functions. (D) is incorrect because, although the recreation center's construction was funded by a loan from the Veterans Administration, the center itself is privately operated and funded by donations. As a result, the center has only a tenuous connection with the federal government, so that it cannot claim the immunity afforded to a federal agency or instrumentality. Accordingly, in the same sense as is employed in the federal tax immunity cases, the agency does not "stand in the shoes" of the federal government. Thus, the application of the state regulations to the recreation center would not present constitutional problems. (A) apparently refers to the principle that the power to regulate foreign commerce lies exclusively with Congress. However, the mere fact that the regulated outlets are part of a wholly owned subsidiary of a Japanese corporation does not mean that the state regulations affect foreign commerce. The subsidiary's activities are conducted entirely within the state, and do not touch upon foreign commerce in any way. Therefore, application of the regulations so as to require the subsidiary to provide an ionizing air purification system for its employee work areas will not constitute a proscribed state regulation of foreign commerce. Thus, (A) is incorrect. (B) is more troubling, but does not offer as compelling an argument as (C). The Contract Clause limits the ability of states to enact laws that substantially impair contract rights (i.e., destroy most or all of a party's rights under an existing contract). Under the Clause, the Supreme Court will subject state actions that impair their own contracts to strict scrutiny. In any case, even if state action substantially impairs rights under an existing contract, the action still may be upheld if it: (i) serves an important and legitimate public interest; and (ii) is a reasonable and narrowly tailored means of promoting that interest. Here, the state supreme court, as an instrumentality of the state, would probably not have grounds for complaining that its rights under the contract have been impaired, but the builder might have grounds (e.g., the builder might have future economic interests during the three-year service period that will be substantially impaired if the court is required to install an ionizing system). Nevertheless, the regulation still may be valid if the state can prove that it truly serves the important public interest of protecting the health and safety of workers in the state and is narrowly tailored to promoting that interest. In any case, because it is uncertain whether the vendor's rights have been substantially impaired and, if so, whether the state can prove the worth of the regulation, (C) is a better choice.

A state statute permits the state to seize and dispose of real property that was used to commit or facilitate the commission of a felony drug offense. After a drug dealer's arrest for selling cocaine out of his home, a felony, the state instituted an action of forfeiture against the drug dealer's house and property. After notice to the drug dealer and a hearing, a judge granted the order and the state seized the property. Six months later, after the time for any appeals had expired, the property was sold at a public auction to a third party. It was only when the third party brought an action to quiet title that a bank holding a properly recorded mortgage on the drug dealer's property learned of the forfeiture. Because the bank's mortgage payments were automatically deducted from an account the drug dealer had under a different name, no one at the bank was aware that the property had been seized. The only notice provided to parties other than the drug dealer was a public notice published for three weeks in a general circulation newspaper. The bank defends the quiet title action on the ground that it did not receive the notice required under the United States Constitution to protect its interest in the property. If the court rules that the bank's rights under the Due Process Clause of the Fourteenth Amendment were violated by the state's seizure of the property, what is the most likely reason? A) In any judicial proceeding affecting rights to real property, a claimant is required to provide notice and an evidentiary hearing to all parties with a legal interest in the property before taking actions affecting their rights. B) The government itself was the party that seized the property, rather than a private party using governmental processes. C) The notice was not adequate under the circumstances to apprise a party with a properly recorded legal interest in the property. D) The jurisdiction treats the mortgagee as having title to the property rather than merely a lien.

(C) is correct. The basis for finding that the bank's due process rights have been violated is that it should have received notice through personal service or by mail. When the government seeks to use a judicial or administrative process to take or terminate property interests, it must give notice to those persons whose property interests may be taken by that process. The form of notice must be reasonably designed to insure that those persons will in fact be notified of the proceedings. Here, the bank had recorded its mortgage and presumably could have been notified by mail that the property was being seized by the government. Being deprived of the opportunity to protect its interest in the property violated the bank's due process rights under the three-part test of Mathews v. Eldridge. Mathews lists three criteria that the courts should weigh in determining what constitutes fair process: (i) the importance of the individual interest involved, (ii) the value of specific procedural safeguards to that interest, and (iii) the governmental interest in fiscal and administrative efficiency. Here, the bank has an important property right that is being terminated, the procedure of publishing a general notice was not sufficient to safeguard its interests, and the government interest in efficiency would not have been overburdened by requiring notice by mail to parties with a recorded interest in the property. (A) is incorrect because it is too broad. Under Mathews, the government is not required to provide personal notice to all parties if it is not feasible, nor is it required to provide a preseizure hearing if exigent circumstances make it impracticable. (B) is incorrect because even when a private party is seeking to use a judicial or administrative process, state action is involved and the Due Process Clause must be satisfied; hence, notice to a record mortgage holder would be required even if a private party were seeking to seize the property through judicial means. (D) is incorrect because the state's characterization of the mortgagee's interest is not critical; the mortgagee has legal rights to the property that are protected by the Due Process Clause, regardless of how the rights are characterized.

A state statute makes it unlawful to willfully cause or permit the life or health of a child under the age of 18 to be endangered. The statute also makes it unlawful to willfully cause or permit a child to be placed in circumstances that endanger the child's life or health. A local prosecutor brought charges against parents who brought their child to their church's religious service in which venomous snakes were passed around. The parents were convicted and have brought an appeal on constitutional grounds. Which of the following best describes the likely result of the parents' appeal? A) Their convictions will be upheld because the parents lack standing to challenge the statute on "free exercise of religion" grounds. B) Their convictions will be upheld because the state's interest in regulating activities involving children necessarily outweighs any rights of members of a church under the Free Exercise Clause of the First Amendment. C) Their convictions will be reversed if it can be shown that the statute is being applied only to interfere with religion. D) Their convictions will be reversed because the freedom to engage in conduct connected with one's religion is absolutely protected under the First and Fourteenth Amendments.

(C) is correct. The convictions will be reversed if it can be shown that the statute is being applied only to interfere with religion. The Free Exercise Clause prohibits government from punishing religious belief. The Clause prevents government from punishing conduct merely because it is religious and from regulating conduct for the purpose of interfering with religion. However, the Clause does not prohibit government from regulating general conduct, even if the regulation happens to interfere with a person's ability to conform conduct to sincerely held religious beliefs. Thus, if it can be shown here that the statute is not really a regulation of general conduct but rather is being applied only to interfere with religion, the convictions will be reversed. (A) is incorrect because the parents would have standing. All that is required is a concrete stake in the outcome of the litigation; having been prosecuted for violating the statute, the parents' stake is about as concrete as it can get. (B) is incorrect because it implies that the court will balance the interests involved in determining the validity of the application of the statute here. Under the former constitutional test, the Court would make such a determination, but since Employment Division v. Smith (1990), the Court has abandoned the balancing approach in favor of the approach discussed above. (D) is incorrect because, as stated above, a person's conduct can be regulated by a generally applicable conduct regulation; religiously motivated conduct has very narrow protection.

A state legislature passed a statute denying in-state tuition rates at any state-supported college or university for any resident aliens who were citizens of "any country which, during the previous five-year period, has held citizens of the United States as hostages." Before becoming a resident alien of the state three years ago, the plaintiff had been a citizen of a country that had held United States citizens as hostages. Prior to passage of the statute, he had been enrolled for two years at the state university and had qualified for the in-state tuition rates because he was a resident of the state. The registrar of the university has informed the plaintiff that he no longer qualifies for the in-state tuition rate, and he cannot afford to pay the nonresident tuition. If the plaintiff files an appropriate suit asking the federal courts to strike down the state statute, which of the following is his best argument? A) Because higher education is now so important for earning a livelihood, it has become a fundamental right protected by the Privileges or Immunities Clause of the Fourteenth Amendment. B) Resident aliens cannot be blamed in retrospect for actions of their governments and, therefore, the state statute violates the constitutional prohibition of ex post facto laws. C) Alienage is a suspect category under the Equal Protection Clause of the Fourteenth Amendment, and the state has not met the appropriate standard to burden the plaintiff. D) Attending a state-supported university in a state where one is a legal resident is a property right that may not be taken away without due process of law.

(C) is correct. The state cannot show a compelling state interest for treating resident aliens differently from other residents. Under the Equal Protection Clause of the Fourteenth Amendment, no state may deny to any person within its jurisdiction the equal protection of the laws. If a government statute classifies persons based on a "suspect" class, strict scrutiny will be applied (i.e., the statute will be upheld only if it is necessary to achieve a compelling government purpose). State and local laws based on a person's alienage are subject to strict scrutiny; the state must show a compelling state interest to justify the disparate treatment. Here, the plaintiff is a resident of the state. The statute treats him differently from other residents, however, because of his status as an alien—all resident aliens are scrutinized to determine whether they were citizens of a country that had taken United States citizens hostage. The state here has no compelling interest to justify the classification; the statute appears to be simply a political statement on a foreign policy matter that the state has no power over. Hence, the Equal Protection Clause is the plaintiff's best argument to strike down the statute. (A) is incorrect for two reasons: The Privileges or Immunities Clause of the Fourteenth Amendment applies only to citizens of the United States, and it only protects certain attributes of national citizenship (such as the right to vote for federal officials). Here, the plaintiff is not a citizen of the United States, and the right to a higher education at a state university is not an attribute of national citizenship. (B) is incorrect because an ex post facto law is legislation that retroactively alters the criminal law in a substantially prejudicial manner. It does not apply to retroactive alteration of civil regulations, such as the right to attend a state university. (D) is incorrect because the plaintiff would have to overcome several hurdles to make an effective due process argument. Procedural due process principles, applicable to the states through the Fourteenth Amendment, provide that government shall not take a person's life, liberty, or property without due process of law. While property includes more than personal belongings, an abstract need for or unilateral expectation of the benefit is not enough. There must be a legitimate claim or "entitlement" to the benefit under state law. While there is a property interest in public education when school attendance is required, the Supreme Court has never specifically held that a property interest exists in public education at the college level. [See Board of Curators v. Horowitz (1978)] Even if the plaintiff could persuade the court to make that assumption, he would also have to establish that imposing the higher tuition effectively deprives him of that property interest and that the registrar's procedures for denying him the lower tuition rate did not satisfy constitutional requirements. At best, he could require the university to give him a hearing before charging him the higher tuition. To strike down the statute itself, however, the plaintiff's best argument is based on the Equal Protection Clause.

To combat rising unemployment, a state offered a $25,000 prize to anyone who could devise a scheme to create at least 200 jobs within the state and demonstrate its viability. While hiking through a national park within the state, a geologist noticed rock containing titanium. Knowing that titanium was commonly used in military aircraft built within the state and that mining and refining titanium could provide the state with thousands of jobs, the geologist chipped out a sample of the ore and took it back to the state employment division. After reviewing the geologist's ideas, the state announced in a press release that he was the first recipient of the $25,000 prize. Within a few days, the federal ranger in charge of the valley from which the sample was taken had the geologist arrested for violating a federal law making it illegal to remove any "plants, animals, or minerals from federal lands." The geologist was convicted and fined $5,000. He appeals the conviction to the federal court of appeals, claiming that the fine is unconstitutional. How should the court rule? A) For the geologist, because the state has a compelling interest in reducing unemployment and the federal statute unreasonably interferes with the state interest. B) For the geologist, because removing the ore was a purely intrastate act and had no effect on interstate commerce. C) For the government, because the federal statute providing for the fine is constitutional under the Property Clause of Article IV, Section 3 of the federal Constitution. D) For the government, because the federal statute providing for the fine is constitutional under the Commerce Clause.

(C) is correct. The court should affirm the geologist's conviction. The fine is constitutional under the Property Clause, which gives Congress the power to "make all needful rules and regulations respecting the territory or other property belonging to the United States." This power permits Congress to acquire and dispose of all kinds of property, and to protect its property with a law such as the one here. (D) is not as good an answer as (C) because the Commerce Clause is not as directly applicable to regulation of acts on federal lands as is the Property Clause. Nevertheless, (B) is incorrect because the fine could probably be upheld under the Commerce Clause. Under the Clause, Congress may regulate any act that may itself or in combination with other activities have a substantial effect on interstate commerce, even intrastate activities. If everyone removed minerals from federal lands, the necessary substantial effect on interstate commerce would be present. (A) is incorrect because, notwithstanding the state's compelling interest, by virtue of the Supremacy Clause a valid act of Congress supersedes any state or local action that actually conflicts with the federal rule.

The federal government recently constructed a radio telescope in Puerto Rico. In addition to receiving radio signals, the telescope was set up to beam radio waves far out into space. As part of the bill providing for operational funding for the facility, Congress provided for a program to "inform any aliens who might be listening in outer space of the 'American Way of Religion.'" A $10 million appropriation was provided; any religious group whose membership exceeded 500 members in the United States was permitted to prepare a five-minute presentation, and the federal government would pay for the recording of the presentations and broadcast them into space using the transmitter in Puerto Rico. The President signed the bill and it became law. A religious group with a large following in Europe, but only 100 members in the United States, protested and filed suit. Will the court find the religious broadcasts to be constitutional? A) Yes, because the power to regulate commerce with foreign nations and among the several states implies that commerce with outer space is included as well. B) Yes, because the commerce power is not limited by First Amendment prohibitions. C) No, because the requirement of a minimum number of members violates the First Amendment. D) No, because the broadcasts are a waste of money and exceed the spending powers of Congress.

(C) is correct. The court will find that the requirement of a minimum number of members violates the Establishment Clause of the First Amendment. The Establishment Clause prohibits any law "respecting an establishment of religion." While usually a three-part test based on Lemon v. Kurtzman is used to determine whether legislation creates improper government involvement with religion, the "compelling government interest" test is used if a law or government program discriminates among religions. Here, the law differentiates among different religious groups, allowing only those with larger memberships to record presentations. There is no compelling government interest for discriminating among the religious groups in this way; thus the legislation is unconstitutional. (A) is incorrect despite the fact that the federal power to regulate commerce could be interpreted as extending to commerce with outer space should the occasion arise. Nevertheless, the commerce power does not override independent constitutional restrictions (e.g., the Establishment Clause) on the conduct in question here. (B) is an incorrect statement of law. The federal commerce power cannot be used to abrogate freedom of speech or to discriminate in favor of religious groups. An exercise of the commerce power generally is subject to all limitations placed on government by the Constitution, including those of the First Amendment. (D) is incorrect. Regardless of merit, almost all expenditures made by Congress are permissible under its spending power. [U.S. Const. art. I, §8] Rather than limit the power only to spending for accomplishment of other enumerated powers, this provision grants Congress broad power to spend for the "general welfare" (i.e., any public purpose). As long as the expenditure is not conditioned on requiring a recipient to forgo an individual constitutional right, it is within the spending power of Congress.

Radon is a harmful gas found in the soil of certain regions of the United States. A state statute requires occupants of residences with basements susceptible to the intrusion of radon to have their residences tested for the presence of radon. The statute also provides that the testing for radon may be done only by testers licensed by a state agency. According to the statute, a firm may be licensed to test for radon only if it meets specified rigorous standards relating to the accuracy of its testing. These standards may easily be achieved with current technology, but the technology required to meet them is 50% more expensive than the technology required to measure radon accumulations in a slightly less accurate manner. The United States Environmental Protection Agency ("EPA") does not license radon testers. A federal statute, however, authorizes the EPA to advise on the accuracy of various methods of radon testing and to provide to the general public a list of testers that use methods it believes to be reasonably accurate. A recently established company in the state uses a testing method that the EPA has stated is reasonably accurate. The company is also included on the EPA's list of testers using methods it believes to be reasonably accurate. The company applied for a radon testing license in the state, but its application was denied because it cannot demonstrate that its method of testing for radon is sufficiently accurate to meet the rigorous state statutory standards. The company has sued appropriate state officials claiming that the state may not constitutionally exclude the company from performing the required radon tests in the state. Who is likely to prevail in the lawsuit? A) The company, because the Full Faith and Credit Clause of the United States Constitution requires the state to respect and give effect to the EPA's inclusion of the company on its list of testers that use reasonably accurate methods. B) The company, because the Supremacy Clause of the United States Constitution requires the state to respect and give effect to the EPA's inclusion of the company on its list of testers that use reasonably accurate methods. C) The state, because the federal statute and the EPA's inclusion of the company on its list of testers that use reasonably accurate methods are not inconsistent with the more rigorous state licensing requirement, and that requirement is reasonably related to a legitimate public interest. D) The state, because radon exposure is limited to basement areas, which, by their very nature, cannot move in interstate commerce.

(C) is correct. The issue here is whether the federal government intended to preempt all state action regarding radon tests. A state or local law may fail under the Supremacy Clause, even if it does not conflict with federally regulated conduct or objectives, if it appears that Congress intended to "occupy" the entire field, thus precluding any state or local regulation. The courts will look at the federal regulatory scheme to deduce Congress's intent (the more comprehensive the laws, the more likely preemption was intended). Also, when traditional state powers are involved (regulations regarding health, safety, and welfare), there is a presumption that preemption was not intended. Here, the federal action is designed to provide for reasonable radon testing, but it is not inconsistent with state action imposing more stringent standards. There is no indication in the question that the federal government intended to occupy the field, and this is an area involving health of the state's citizens. Federal preemption will most likely not be found. (A) is incorrect. The Full Faith and Credit Clause of the United States Constitution provides that "full faith and credit shall be given in each state to the public acts, records, and judicial proceedings of every other state." By virtue of the Clause, if a judgment is entitled to full faith and credit, it must be recognized in sister states. The Clause is inapplicable here because the case does not involve a state recognizing the judgments of a sister state. (B) is incorrect. The Supremacy Clause is applicable only if the state action and its laws are inconsistent with the federal action or if the federal government intends to occupy the field. As explained above, neither condition exists in this question. (D) is incorrect. While it is true that the concern over radon gas might focus on basements that cannot move in interstate commerce, the question here is about licensing radon gas testing companies. The testing companies are businesses (that is, they are commercial enterprises involved in economic activities). Thus, there is no question of the power of Congress to establish standards for radon testing under the Commerce Clause. Radon testing, in the aggregate, affects interstate commerce.

An act of Congress provides for the payment of Social Security benefits to the disabled children of deceased workers. The benefits are payable even after the child reaches the age of majority, but terminate automatically if the recipient marries, unless the recipient marries a person who is also disabled, in which case the Social Security payments continue. The plaintiff has been disabled since birth, and her father died when she was 14 years of age. She has received Social Security benefits since her father's death. However, at age 25, the plaintiff married. Her husband is not disabled, and thus her Social Security benefits were summarily terminated. She files suit in federal district court, seeking to compel the government to restore her benefits. She asserts, among other things, that there is no logical reason why she should be deprived of benefits when unmarried disabled persons and disabled persons who marry other disabled persons continue to receive them. Which of the following provides the strongest constitutional basis for the plaintiff's suit? A) The privileges and immunities of national citizenship protected by the Fourteenth Amendment. B) The Equal Protection Clause of the Fourteenth Amendment. C) The Due Process Clause of the Fifth Amendment. D) The prohibitions against bills of attainder.

(C) is correct. The plaintiff's strongest basis for challenging her denial of benefits by the federal government is the Due Process Clause of the Fifth Amendment. Her argument is that the regulations for determining who receives benefits create arbitrary and discriminatory classifications among similarly situated groups. Such classifications by a state government can be challenged under the Equal Protection Clause of the Fourteenth Amendment, but there is no counterpart to this clause applicable to the federal government. Nevertheless, the Supreme Court has held that grossly unreasonable discrimination by the federal government violates the Due Process Clause of the Fifth Amendment, and has usually applied the same standards that it uses in equal protection actions against a state. Hence, this clause constitutes the plaintiff's only plausible basis for a suit challenging the regulations. (A) is wrong because the Privileges or Immunities Clause of the Fourteenth Amendment applies only to states and not the federal government, and because it protects only certain privileges and immunities of national citizenship, none of which are implicated in this question. (B) is wrong because, as discussed above, the Equal Protection Clause of the Fourteenth Amendment does not apply to the federal government. (D) is wrong because a bill of attainder is a legislative act that inflicts punishment without a judicial trial on individuals who are designated either by name or in terms of past conduct. No aspect of the law here constitutes a bill of attainder.

A state statute required all persons operating a catering business within the state to be licensed by the state board of health. The statute further provided that licenses will be granted only to applicants who have resided in the state for five years and who are citizens of the United States. A long-time resident of the state who was not a United States citizen challenges the requirement that applicants for licenses must be citizens of the United States. This provision is most likely: A) Constitutional, as an exercise of the state's police power. B) Constitutional, as an effort to protect the health and welfare of the residents of the state. C) Unconstitutional, as a denial of equal protection. D) Unconstitutional, as a bill of attainder.

(C) is correct. The provision is most likely unconstitutional as a denial of equal protection. Alienage is a suspect class when used by the states; therefore, strict scrutiny will be applied to determine if there is a compelling state interest, unless the law discriminates against alien participation in the functioning of the state government. This is not that type of law, and no compelling interest is demonstrated by these facts. (A) and (B) are, therefore, wrong because the state has not and could not allege a compelling interest in support of this measure. (D) is wrong because the statute would not qualify as a bill of attainder, which imposes punishment on specific individuals without a judicial trial.

Congress passed legislation banning the hunting of snipe birds within the United States. The range of the snipe is quite limited; they are found primarily in only one state, although they migrate annually to several nearby states. Hunters from throughout the United States have traditionally traveled to the snipe's home state during snipe hunting season, bringing considerable revenue into the state. A state statute allows hunting of snipe during a two-week period in November and charges a $50 license fee for state residents and a $250 fee for hunters from other states. The bag limit is one snipe bird per licensed hunter. Is the state statute allowing snipe hunting valid? A) Yes, because states have the right to control their own natural resources and wildlife. B) Yes, because the power exercised is reserved to the states by the Tenth Amendment. C) No, because of the Supremacy Clause. D) No, because of the Commerce Clause.

(C) is correct. The state statute is invalid because of the Supremacy Clause. Under the Clause, if the federal government adopts legislation that it has the power to adopt, the federal legislation is supreme, and a conflicting state law is rendered invalid. The federal law here, banning the hunting of snipe, is within the federal government's power under the Commerce Clause, which gives the government power to regulate anything that might affect interstate commerce. Because the birds themselves are found in a few states, they probably cross state lines. Also, hunters come from out of state and generate revenue in the state, so interstate commerce is involved. The state law directly conflicts with the federal law because it allows hunting of snipe. Therefore, the state law will be held invalid under the Supremacy Clause. (A) is incorrect because, while states do have a limited right to control their natural resources, the right is concurrent with the federal government's power, and cannot be exercised to conflict with federal regulation in the area. Note further that a state's power to control its natural resources is also limited even if Congress does not act: A state may not adopt a law discriminating against interstate commerce or excessively burdening interstate commerce, even absent federal legislation. Regarding (B), the Tenth Amendment reserves all powers not granted to the federal government to the states. (B) is incorrect because the Court will not likely strike down on Tenth Amendment grounds a federal regulation that subjects state governments to the same regulations as apply to the private sector. In such cases, the states' interests are best protected by the states' representation in Congress. (D) is incorrect because the Commerce Clause does not render the state's action invalid; it merely gives Congress the power to act. It is the Supremacy Clause that makes the interfering state law invalid.

A state public employee retirement act provided that, while legitimate children under 18 qualify for survivor benefits, an employee's children born out of wedlock may recover only if they lived with the employee in a regular parent-child relationship. A state employee lived with a woman in the state for 10 years, after which they separated. They had two children, both of whom were the employee's natural children born out of wedlock. The employee supported the children under a state child support decree until he died a year ago. At the time of his death, he was covered by the retirement act. The state retirement commission determined that the children did not qualify because they were living with their mother and not living with the employee at the time of his death. The mother sued in federal court alleging that, if the children were born in wedlock, they would have been entitled to benefits, and that it was discriminatory to treat illegitimate children differently. Is the state provision constitutional? A) Yes, because a state may allocate survivor benefits to its employees without restriction. B) Yes, if there was a rational basis for the classification of illegitimate versus legitimate children. C) No, unless the classification of legitimate and illegitimate children is substantially related to an important state purpose. D) No, unless the different treatment of legitimate and illegitimate children is necessary to promote a compelling state interest.

(C) is correct. The statutory provision should be found unconstitutional unless the classification is substantially related to an important government purpose. Discrimination against illegitimate children is judged by a quasi-suspect standard similar to that used in gender discrimination cases. The standard requires that the classification be substantially related to an important state purpose. (A) is incorrect because state action is restricted by federal constitutional protections. (B) is incorrect because the rational basis standard does not apply to legitimacy classifications or any other quasi-suspect classifications. (D) is incorrect because the compelling interest standard applies to suspect classifications, such as race and national origin, not to quasi-suspect classifications such as legitimacy.

A state enacted legislation providing for the testing of all high school students in the performance of certain adult-world tasks. In public schools, the test was administered at the school by government employees. In private schools—both religious and secular—the test was administered by school personnel, and the schools were reimbursed for the costs of administering the tests and reporting the results to the state. An association with appropriate standing filed suit seeking to enjoin the reimbursement to the religious schools, claiming that the reimbursement violates the Establishment Clause of the First Amendment to the United States Constitution. Should the trial court grant the injunction? A) Yes, because direct state payment to religious schools violates the Establishment Clause. B) Yes, because the state may not require the religious schools to conduct tests that violate their religious principles. C) No, because the reimbursement has a secular purpose that neither advances nor inhibits religion and that will not entangle the state in religion. D) No, because the tests do not involve matters of religion.

(C) is correct. The trial court should not grant the injunction, because the reimbursement does not violate the Establishment Clause. In Committee for Public Education and Religious Liberty v. Regan (1980), the United States Supreme Court held that a similar statute passed by the state of New York did not violate the Establishment Clause because it had a secular legislative purpose. Its principal effect did not advance or inhibit religion, and it did not foster an excessive government entanglement with religion. Here, our facts show a state law that provides for secular testing and reimbursement for testing administration costs to religious schools. As such, the state law does not violate the Establishment Clause because it had a secular legislative purpose (namely, reimbursing schools for secular testing that was conducted in all schools—religious and secular alike), its principal effect did not advance or inhibit religion, and it did not foster an excessive government entanglement with religion. (A) is therefore incorrect; aid can be given to religious schools that pass this test. (B) is wrong. It misstates the facts given, which make no mention of a specific religious group. Rather, the suit was filed by an association challenging the reimbursement. (D) is irrelevant. The question is not whether the tests involve "matters of religion," but whether the reimbursement program impermissibly advances religion.

A state law required all automobile drivers to carry liability insurance; however, because of the high number of auto accidents in the state, the cost of insurance became prohibitive. A study sponsored by the state legislature showed that males under the age of 21 were four times more likely to get into automobile accidents than any other group, including females in the same age group. The study predicted that prohibiting males under the age of 21 from driving would result in a 15% reduction in all other persons' automobile insurance rates. Ultimately, the legislature raised the minimum age for obtaining a driver's license to age 21 for males. Females were still allowed to obtain licenses at age 16. An 18-year-old male living in the state when the limit was raised, and who worked as a pizza delivery driver, was fired from his job and replaced by a 17-year-old female. If the young man sues to have the law set aside and prevails, what is the most likely reason? A) The state could not prove that the law was the least restrictive means of achieving a compelling government purpose. B) The state could not prove that the law was rationally related to a legitimate government purpose. C) The state could not prove that the law was substantially related to an important government interest. D) The state could not prove that the law was necessary to achieve a compelling government purpose.

(C) is correct. The young man will prevail if the state cannot establish that the restriction is substantially related to an important government interest. Classifications based on gender usually are tested against an intermediate standard of review; i.e., the Supreme Court will strike down the classification unless the government offers an exceedingly persuasive justification that the classification is substantially related to an important government interest. Classifications intentionally discriminating against men generally are invalid, and (C) states the proper standard for review. (A) and (D) are substantially the same and incorrect because they state the standard to be applied to classifications involving a suspect class or fundamental right. However, a gender-based distinction is characterized as a quasi-suspect classification. (B) is incorrect because it states the standard to be applied when no fundamental right or suspect or quasi-suspect class is involved.

A state recently enacted a statute making it illegal for "anyone to cause any substance that is likely to harm fish or aquatic plants" to be discharged into any body of water in the state. The statute was aimed at curtailing the dumping of industrial waste into the state's waters. The state attorney general has sole authority to enforce the statute and may seek either fines or injunctive relief. Farmers in the state use fertilizer on their crops, which could be carried by rain into lakes and streams and might harm aquatic life. The farmers fear that the statute will be applied to them. When the statute became effective, three local farmers brought an action in federal district court, seeking a declaratory judgment that the statute is unconstitutionally vague and injunctive relief against its enforcement because enforcement against the farmers would destroy their livelihood. Which of the following doctrines would be most likely to prevent the case from being heard on its merits? A) Mootness, because the state has not enforced the law against the farmers. B) Standing, because the three farmers cannot properly represent all state farmers. C) Abstention, because there is an unsettled question of state law. D) Political question, because this issue should be decided by the state legislature.

(C) is correct. When a federal constitutional claim is premised on an unsettled question of state law, the federal court should abstain temporarily so the state courts have a chance to settle the underlying state law question and, thus, potentially avoid the needless resolution of a constitutional issue. Here, the scope of the statute is unclear. State courts might construe it in a manner that avoids a federal constitutional issue. Therefore, the federal courts would abstain from ruling on the vagueness issue until the statute's scope is determined by a state court. (A) is incorrect. A federal court will not hear a case that has become moot (where there is no real, live controversy because the claims have been resolved). That is not the case here; the claims have not been resolved. Here, the farmers are engaged in activities that could subject them to prosecution under the statute. (B) is incorrect. To have standing in a case, a person has to show a concrete stake in the outcome of the case; he must show an injury in fact that can be remedied by a decision in his favor. Here, the plaintiffs will suffer substantial economic and legal harm if the statute is enforced against them. Therefore, they have standing and are proper plaintiffs in the suit. They are not actually representing other farmers in the state. They are seeking a remedy for themselves, which may incidentally benefit other farmers. (D) is incorrect. Political questions are those issues committed by the Constitution to another branch of government or those inherently incapable of resolution and enforcement by the judicial process. The vagueness issue is one with which courts are equipped to and, in fact, are mandated to deal, and the resolution of this issue has not been committed to a coordinate branch of the federal government. Therefore, no political question is involved in this case.

Congress determined that bees were important to agricultural and consumer interests. Therefore, it passed a statute requiring beekeepers to be licensed, regulating the standards for maintaining hives, and establishing safety standards for the removal of honey from hives. Congress appropriated $50 million for use by the Department of Agriculture to enforce the act. A taxpayer has challenged the validity of the act, arguing that the federal government lacks the power to legislate with respect to bee-keeping and so is unconstitutionally spending tax revenues. What is the most likely disposition by the federal court? A) The court will determine the validity of the statute if the taxpayer can show a direct benefit from a determination that the act is unconstitutional. B) The court will determine the validity of the statute if the appropriations in support of the act can be considered spending under Congress's power to tax and spend. C) The court will dismiss for failure to state a cause of action. D) The court will dismiss for lack of standing.

(D) is correct. A federal court will not decide a constitutional challenge to a government action unless the person challenging the action has standing to raise the issue. A person has standing only if she can demonstrate a concrete stake in the outcome of the controversy— an injury in fact that will be remedied by a decision in her favor. A taxpayer, of course, has standing to litigate her tax bill, but she generally does not have standing as a taxpayer to challenge the way tax dollars are spent by the government because her interest is too remote. There is an exception if the federal taxpayer can establish that the challenged measure was enacted under Congress's taxing and spending power, and it exceeds some specific limitation on the power. To date, the only limit that the Supreme Court has found on the taxing power is the Establishment Clause. In this case, the taxpayer would not have standing to challenge the act's validity, because the act is a regulatory rather than a spending statute and there is no Establishment Clause challenge here. Therefore, the suit will be dismissed. (A) is incorrect. This answer choice misstates the requirement for federal taxpayer standing. The act was passed under Congress's commerce power, not its taxing and spending power, and the associated outlay of money is incidental to that. Moreover, the mere fact that the taxpayer may benefit indirectly from saving the government some money is not enough. Because taxpayer standing is so limited, it is highly unusual for the taxpayer ever to have standing to challenge an act of Congress. (B) is incorrect. Even if the appropriation were considered an exercise of Congress's power to tax and spend, the plaintiff still lacks standing because standing exists only to challenge expenditures as violations of the Establishment Clause, which is not at issue here. (C) is incorrect. There is a cause of action here, which may be adjudicated if brought by a plaintiff with standing. For example, a beekeeper might be able to challenge the act as an improper exercise of Congress's commerce powers. The beekeeper would probably not be successful but could establish standing by showing direct economic harm as a result of the legislation.

A zoning ordinance in a town prohibits more than three unrelated individuals from living in a dwelling unit. A husband and wife who live in a five-bedroom home provide foster care for children who are temporarily without parents. At a time when there were six foster children living in their house, a zoning officer cited the husband and wife for violation of the ordinance. The couple brought suit in federal court challenging the constitutionality of the ordinance. When the case came to trial, there was only one foster child living in their house. Consequently, the town revoked the citation and moved to dismiss the complaint, arguing the matter had been resolved. The couple objected to the motion. How should the court rule on the town's motion? A) Grant the motion, because the plaintiffs no longer are in violation of the ordinance. B) Grant the motion, because constitutional issues are no longer ripe for review by the court. C) Deny the motion, because once a constitutional issue is raised, a change of circumstances cannot defeat the constitutional claim. D) Deny the motion, because the plaintiffs may in the future want to take care of more than one foster child at a time in their home.

(D) is correct. A federal court will not hear a case that has become moot; a real, live controversy must exist at all stages of review, not merely when the complaint is filed. When a change in facts or law resolves a case or controversy, and the matter at issue no longer exists, the case will be dismissed as moot. However, where there is a reasonable expectation that the same complaining party will be subjected to the same action again and would again be unable to resolve the issue because of the short duration of the action, the controversy will not be deemed moot. Here, even though there was no ongoing violation of the ordinance when the case went to trial, the couple may well resume caring for additional foster children in violation of the ordinance. Since the couple could be subjected to the same action again if they were to take in additional foster children, the court should deny the town's motion because the injury to the plaintiffs is capable of repetition. (A) is incorrect. As explained above, when the controversy is resolved before trial, the suit will generally be dismissed as moot. But an exception applies when, as here, the case is capable of repetition yet evading review. Thus, the fact that the couple is no longer in violation of the ordinance is not a basis to dismiss here. (B) is incorrect. Ripeness bars consideration of claims before they have been developed; mootness bars their consideration after they have been resolved. The issue here is not ripeness; the case was mature enough to warrant review when it was filed. "No longer ripe" implies a claim of mootness, but as explained above, the case here will not be dismissed for mootness. (C) is incorrect. Although this choice has the correct conclusion, it states an invalid rationale—a change of circumstances can defeat a constitutional claim under the doctrine of mootness, as explained above.

A new federal law prohibited the use of various pesticides in areas with a certain population density near navigable waters. A city located in the southeastern United States was plagued by a sharp increase in disease-carrying mosquitoes. The city's board of health recommended that all residential areas be sprayed with a pesticide proven to be highly effective against mosquitoes. Despite the fact that the federal law would prohibit use of that pesticide in these areas, the city council passed an ordinance adopting the board of health plan, relying on the opinions of several independent experts that the health benefits of reducing the mosquito population outweighed the risks of spraying. An environmentally minded citizen of the city brought an action in federal court challenging the ordinance. Assuming that the citizen has standing, is the court likely to find the ordinance valid? A) Yes, because pursuant to the police power, cities have a compelling interest in laws designed to protect the health, safety, and welfare of their citizens. B) Yes, because controlling health hazards is an integral governmental function. C) No, because it is superseded by the power of Congress to adopt laws to protect the health, safety, and welfare of citizens. D) No, because it conflicts with a federal law that Congress had the power to make under the Commerce Clause.

(D) is correct. Congress's power to regulate commerce has been construed broadly, so that it may regulate any activity, local or interstate, that either in itself or in combination with other activities has a substantial economic effect on interstate commerce. If Congress has determined that the use of chemical pesticides and their runoff into waterways (which are channels of interstate commerce) will have an overall detrimental impact on the environment, this determination will be sufficient in this case to satisfy the standards established by the Supreme Court. Therefore, the law probably is a valid exercise of the commerce power. Any state or local action that conflicts with a valid act of Congress is invalid under the Supremacy Clause. (A) is incorrect because while the police power (the power to adopt regulations for the health, safety, and welfare of citizens) belongs to the states, a police power regulation that conflicts with a federal law is invalid under the Supremacy Clause. (B) is incorrect because state and local government activities may be regulated by a general law that applies to both the public and private sectors, even if the regulation affects integral governmental functions, as long as there is a constitutional basis for the law. (C) is incorrect because Congress does not have a general "police power" to adopt laws on health and safety. The laws that Congress has passed banning activities that it has deemed harmful to public health have been based on its power to regulate interstate commerce.

A state wants to open a pilot daycare program for preschool children and, to that end, it submitted the project for open bids as required by state law. The lowest bid that met the criteria established by the state was submitted by a religious organization. The organization's bid is low, in part, because the building in which the program would be housed is tax exempt under state law under a religious institution exemption to property taxes. The organization agreed to operate under the conditions of the project contract, including the provision prohibiting the program from being used to espouse or promote any particular religious view. Studies have shown, however, that children in such situations tend to emulate their caregivers. The state decides to award the contract to the organization. A citizen of the state brings an action in a proper state court seeking to set the award aside, claiming that it violates the United States Constitution. Is the citizen likely to prevail? A) Yes, because the program would constitute an establishment of religion. B) Yes, because the organization's First Amendment rights of freedom of religion and speech would be violated by the contract provision prohibiting it from espousing its religious views. C) No, because the program is necessary to achieve the compelling state interest of educating youth. D) No, because any entanglement with religion is not excessive.

(D) is correct. The Establishment Clause prohibits government action respecting the establishment of religion. If government action does not involve a sect preference, it will be valid under the Establishment Clause if it (i) has a secular purpose, (ii) has a primary effect that neither advances nor inhibits religion, and (iii) does not produce excessive government entanglement with religion. Here, the program between the organization and the state would not constitute an establishment of religion because the purpose of the program is clearly secular, and the program's primary effect does not advance religion. The program does not excessively entangle the religious organization and the state because the organization agreed to operate under the conditions of the project contract, including the provision prohibiting the program from being used to espouse or promote any particular religious view. (A) is incorrect. As explained above, the program would not constitute an establishment of religion. The organization would not be allowed to promote its religion as part of its services to the state, and the mere fact that the children may consider the organization's staff to be role models is insufficient to constitute an establishment of religion. (B) is incorrect. The organization's constitutional rights of free speech and freedom of religion have not been violated. The government cannot condition the provision of fundamental rights on an individual's agreement to give up his constitutional rights. However, the organization has no right to this contract to provide daycare services. Therefore, the government can require the organization to comply with the terms of the contract. (C) is incorrect. There is no need for this program to satisfy the strict scrutiny standard. No parties are being deprived of fundamental rights or being treated unequally due to a suspect classification.

A high school held a graduation ceremony. Attendance at the ceremony is voluntary. The principal of the school is a good friend and follower of a local minister and he convinced the minister to attend the ceremony, say an opening prayer, and congratulate the students for their achievement. The principal instructed the minister to make sure that her opening prayer is nonsectarian, just a general invocation of blessings. The principal even provided the minister with a sample prayer that he believed would pass First Amendment scrutiny. A student who was raised in an atheist home saw the schedule and filed an action to enjoin the school from including the prayer. Should the court grant the injunction? A) No, because the prayer is nonsectarian and, therefore, does not promote any particular religion. B) No, because attendance at graduation ceremonies is voluntary. C) No, because the student is not forced to participate in the prayer and could choose to remain silent. D) Yes, because the prayer violates the Establishment Clause.

(D) is correct. The Establishment Clause prohibits government action respecting the establishment of religion. Prayer and Bible reading in school are invalid as establishments of religion. This rule extends to prohibit public school officials from having clerics give invocation and benediction prayers at graduation ceremonies. (A) is incorrect. A nonsectarian prayer still violates the Establishment Clause by promoting religion in general, even if it does not promote any particular religion. (B) is incorrect. A high school graduation is a significant, public, state-sponsored occasion, and the fact that attendance is voluntary does not save it from having to satisfy scrutiny under the Establishment Clause. (C) is incorrect. Although the student is not forced to participate per se, there likely was peer pressure to stand or at least maintain a respectful silence, and these factors taken together essentially force the student to participate in the prayer.

The President pardoned a drug dealer who was convicted on federal drug charges. After that unpopular pardon, Congress passed a law (over the President's veto) providing that the President can pardon individuals convicted for drug offenses and sentenced to prison only after they have served one-half of their minimum term. After the law was passed, the President issued a pardon to another person who had been convicted of a drug offense. The offender had served less than half of her minimum term. The warden of the prison in which the offender was held refused to release the offender, citing the legislation. The President then directed the attorney general to file a suit to force the warden to recognize the pardon and release the offender, arguing the legislation is unconstitutional. Is the court likely to find the legislation constitutional? A) Yes, because Congress has power to pass legislation to create federal crimes. B) Yes, because Congress has power to enact laws necessary and proper to its enumerated powers. C) No, because the law improperly restricts the President's inherent power as chief executive. D) No, because the power of the President to pardon for federal crimes is explicitly granted by the United States Constitution.

(D) is correct. The President is empowered by the Constitution "to grant reprieves and pardons for offenses against the United States, except in cases of impeachment." The pardon power cannot be limited by Congress. Here, Congress is limiting the President's pardon power by preventing the President's ability to pardon certain individuals until they have served one-half of their minimum term. (A) is incorrect. While Congress can properly legislate to create federal crimes, it cannot interfere with a power, such as the pardon power, granted to the President by the Constitution. (B) is incorrect. The Necessary and Proper Clause grants Congress the power to make all laws necessary and proper for carrying into execution any power granted to any branch of the federal government. However, the powers granted to Congress do not permit it to abrogate a power specifically given to the President, such as the pardon power. (C) is incorrect. The Constitution explicitly grants to the President the pardon power and, therefore, that power is not merely inherent.

A Congressman was indicted for conspiracy to use improper influence to prevent a tax fraud investigation of his ex-law partner. The prosecution desires to show that, as part of the conspiracy, the Congressman made a speech on the floor of the United States House of Representatives at a relevant time, urging a cutback in appropriations for the Internal Revenue Service. The Congressman's legislative aide and speechwriter has been called as a witness to the circumstances of the preparation of that speech. The Congressman objects to the calling of his speechwriter. How should the court rule on the objection? A) Deny the objection, because the Speech or Debate Clause does not protect speeches when they are the subject of a criminal prosecution. B) Deny the objection, because the Speech or Debate Clause only protects speeches made on the floor of Congress, not relations with congressional aides. C) Sustain the objection, because the Speech or Debate Clause protects all confidential communications between a Congressman and his aide. D) Sustain the objection, because the aide is protected by the Speech or Debate Clause when preparing a speech for the floor of Congress.

(D) is correct. The Speech or Debate Clause provides that "For any speech or debate in either House [members of Congress] shall not be questioned in any other place." The immunity extends to aides who engage in acts that would be immune if performed by a legislator. Here, the aide assisted in the legislative process by preparing a speech to be delivered during a legislative session and, thus, is protected under the Speech or Debate Clause. Therefore, (B) is incorrect. (A) is incorrect. Under the Speech or Debate Clause, the deliberative process on the floor of Congress is a protected activity, even when it is relevant to a criminal investigation. (C) is incorrect. This statement is too broad. The scope of the Speech or Debate Clause extends only to material prepared for the deliberative process. It does not create a general privilege between a member of Congress and his aides.

A town in a rural state facing financial difficulties passed a variety of "sin taxes," including one aimed at electronic game arcades frequented by local juveniles. The tax is a one cent per game tax imposed on the manufacturers of the games based on the estimated number of plays over a machine's lifetime. There are no electronic game manufacturers in the state. Which of the following constitutional provisions would support the best argument against enforcement of the tax? A) The Equal Protection Clause. B) Substantive due process. C) The Privileges and Immunities Clause of Article IV. D) The Commerce Clause.

(D) is correct. The best argument against enforcement of the tax is that it violates the Commerce Clause. If Congress has not adopted laws regarding a subject, local governments are free to tax or regulate local aspects of the subject area as long as the tax or regulation does not discriminate against interstate commerce or unduly burden it. Here, the tax does not discriminate against interstate commerce, since it does not single out interstate commerce for taxation in order to benefit the local economy. However, it could be argued that the tax unduly burdens interstate commerce. A local tax will be held to unduly burden interstate commerce if the locality's need for the revenue does not outweigh the burden on interstate commerce. The Supreme Court will consider whether there is a substantial nexus between the activity or property taxed and the taxing state, whether the tax is fairly apportioned, and whether there is a fair relationship between the tax and the benefit the taxed party receives from the state. Here, there is little nexus between the manufacturer and the town. The facts indicate that out-of-state manufacturers' machines are used in the town, but do not indicate whether the manufacturers conduct any selling activity in the town. Similarly, nothing indicates that there is a relationship between the tax and any benefit that the manufacturers derive from the town. Thus, the tax would probably be unconstitutional under the Commerce Clause. (A) is not as good an argument as (D) because the Equal Protection Clause prohibits the states from treating similarly situated persons differently without sufficient justification. Where a classification does not involve a suspect or quasi-suspect class or a fundamental right, the classification will be upheld as long as it is rationally related to a legitimate government interest. While the tax here singles out arcade game manufacturers for special tax treatment, no suspect or quasi-suspect class is involved, nor is a fundamental right affected. Thus, the tax will be valid under the Equal Protection Clause because it is rationally related to the legitimate government interest of raising revenue. (B) is not a good argument because substantive due process requires that laws not be arbitrary. When laws do not involve a fundamental right, they will be held valid under the Due Process Clause as long as they are rationally related to a legitimate government interest. As established above, no fundamental right is involved and the tax is rationally related to a legitimate government interest. Thus, under the Due Process Clause the tax may be enforced. (C) is not a good argument because the Privileges and Immunities Clause of Article IV prohibits states from discriminating against out-of-state residents when a fundamental right is involved, and the tax here does not differentiate between residents and nonresidents.

A state that is subject to severe winters generally allows the use of studded tires between October 1 and March 31. However, the legislation allows counties to opt out and prohibit the use of studded tires year round, because studded tires tend to tear up pavement more than nonstudded tires, thus necessitating more frequent road repairs. No other state in the region allows use of studded snow tires at all. The state law contains one exception: it excludes "doctors" from any county ban on the use of snow tires because they might have to cross county lines in emergencies. After the passage of the legislation, only one county in the state invoked its right to ban the use of studded snow tires. A lawyer who lives in the state was angered that the legislature had given special privileges to doctors but not to lawyers. One January day, with studded tires on his car, he drove from his home county, which allowed use of studded tires, into the county that banned them. A sheriff's officer noticed the lawyer's studded tires and cited him. After being convicted and fined, the lawyer appealed. What is the lawyer's best argument for getting the ban invalidated? A) The statute interferes with his fundamental right to practice his profession in violation of the Privileges and Immunities Clause of Article IV. B) The statute violates his right to travel. C) The statute violates the Commerce Clause by placing an unreasonable restraint on interstate commerce. D) The ban on studded snow tires is not rationally related to a legitimate state interest because it will likely result in an increased loss of life.

(D) is correct. The best argument for getting the law invalidated is that it is not rationally related to a legitimate state interest. The lawyer would argue that the statute violates equal protection because it singles out one class of citizens for special treatment. Because neither a fundamental right nor a suspect nor quasi-suspect class is involved here, the case would be decided under the rational basis standard. For a law to be held invalid under the rational basis standard, the plaintiff must show that the law is not rationally related to a legitimate state interest. Toward this end, the lawyer might argue that the law will really cost more money than it will save, perhaps because the resulting number of injuries due to the absence of studded tires will more than offset the money saved in road repair. (This argument will likely fail, however, because courts give legislatures broad discretion in making such determinations, and the statute does appear to be rational. Nevertheless, this is the lawyer's best argument.) (A) is incorrect because the Privileges and Immunities Clause of Article IV applies only to discrimination by a state against nonresidents, and here the lawyer is a resident of the state that enacted the legislation. (B) is incorrect because nothing in the facts indicates that the right to travel is involved—at least not the constitutionally protected right to travel. The right to travel involves interstate travel, and here, the legislation concerns only an intrastate travel issue. (C) is incorrect because there is no unreasonable restraint on interstate commerce. If Congress has not allowed or prohibited state regulation in the area, a nondiscriminatory state regulation will be upheld only if its burden on commerce does not outweigh a legitimate local interest. Here, because no other state allows studded snow tires, the ban does not discriminate against out-of-state vehicles and does not burden commerce. (If other states allowed these snow tires, there might be a viable Commerce Clause issue.)

A man was arrested in a state for armed robbery. A combined preliminary hearing to determine probable cause and initial appearance was held within 20 hours of his arrest. Probable cause was found, and bail was properly denied under the state's Bail Reform Act. A state statute provided that when a defendant is in custody, his trial must begin within 50 days of his arrest. After 50 days had passed since the man's arrest and no trial had been held, he filed a motion for dismissal for violation of his right to a speedy trial under the state constitution, which tracked verbatim the speedy trial provision of the United States Constitution. The trial judge held that he was bound to follow federal interpretations of the speedy trial provision and granted the man's motion on that basis. On appeal, the state supreme court agreed with the trial judge. The state prosecutor seeks to challenge the ruling in the United States Supreme Court. If the Supreme Court thinks that the state court wrongly decided that the man was denied his right to a speedy trial under federal standards, how should it proceed? A) Reverse the decision because the state speedy trial provision cannot be interpreted in a manner different from federal interpretations. B) Reverse the decision and remand it to state court because the state speedy trial issue was so intertwined with the federal question that it would be difficult to determine on which ground the state court relied. C) Decline jurisdiction because the Eleventh Amendment prohibits a state from challenging a decision of its supreme court in federal court. D) Reverse the decision and remand the case to be decided on the independent state grounds only.

(D) is correct. The best way to approach this question is to eliminate the wrong answers first. (A) is incorrect because state constitutional provisions do not have to be interpreted exactly the same as federal provisions; the federal Constitution provides the minimum rights that states must provide, but states are free to grant broader rights. Thus, even though a 50-day delay may be constitutional under the federal Constitution, it can still be held unconstitutional under a state constitution. (B) is incorrect because the facts make it clear that the state court was relying on federal case interpretations. Moreover, if the Supreme Court could not decide whether the case was based on federal or state grounds, it would not reverse the case, because a federal court will not hear a case that can be based on adequate and independent state grounds; rather, it would dismiss the case or remand it to the state for clarification. (C) is incorrect because the Eleventh Amendment generally prohibits federal courts from hearing actions by a private party or foreign government against the state government; it does not bar a state from appealing a ruling from its own court system. Thus, (D) is correct. The Supreme Court had jurisdiction to hear the case, because it has jurisdiction to hear appeals from a state's highest court concerning the constitutionality of a state statute, and as indicated above, the state court's decision was not based on independent state grounds; the decision was based on federal case law interpreting an identical federal provision. Thus, jurisdiction was proper and the Court could reverse the state court decision and hold that a 50-day delay does not violate the federal Constitution. However, the case should be remanded so that the state may decide whether the delay was too long under state law, since a state is free to provide its citizens with more civil protection than is required by the federal Constitution.

A city ordinance that gave churches veto power over the grant of liquor licenses for businesses within a quarter mile of the church was struck down by the state's supreme court. In its decision, the court noted that even if the ordinance were not a violation of the federal Establishment Clause, it clearly violated a provision in the state constitution requiring the separation of church and state because it delegated governmental authority to the churches. The supreme court opinion also harshly criticized the city council for passing an ordinance that was clearly a violation of state law. To comply with the court's decision and avoid future criticism, the city council adopted an ordinance prohibiting any cleric from sitting on any public board within the city. A cleric who is currently on a school board within the city brings a suit in federal district court to have the ordinance declared unconstitutional. How should the court rule? A) Uphold the ordinance, because it has a secular purpose and it does not promote excessive entanglement between government and religion. B) Uphold the ordinance, because the previous court decision was based on adequate and independent state grounds. C) Hold the ordinance unconstitutional, because its passage violated the cleric's right to procedural due process. D) Hold the ordinance unconstitutional, because it violates the Free Exercise Clause.

(D) is correct. The ordinance should be held unconstitutional because it violates the Free Exercise Clause. The Free Exercise Clause generally prohibits government from acting in a way that interferes with the free exercise of religion. The Supreme Court has found the Clause to provide almost absolute protection for religious belief. Here, the city is prohibiting clerics from serving on public boards. Essentially, the ordinance takes away a right of citizenship merely because a person has chosen to become a religious cleric. Such punishment of religious belief is prohibited, either absolutely or at least unless the discrimination is necessary to achieve a compelling government interest. No such compelling interest is present. Although the state supreme court found that the city could not delegate its authority regarding liquor licenses to churches, and complying with the decision might be a compelling government interest, the replacement ordinance here is not necessary to achieving that interest. (A) is incorrect, at the very least because it is incomplete. It is based on the Lemon test for determining whether government action violates the Establishment Clause, but the choice is missing a crucial element: The government action cannot substantially advance nor inhibit religion. The ordinance here—prohibiting clerics from serving on public boards—would have a tendency to inhibit religion and so would run afoul of the Lemon test. (B) is incorrect because it is irrelevant. Even if the previous decision was based on state law, the current ordinance was not at issue and clearly violates the federal Constitution. (C) is incorrect because when a government action does not focus on an individual, due process does not require an individualized hearing. The process of passing the ordinance at the city council meeting was all of the process that was due the cleric.

Small, prolific mussels called zebra mussels were first introduced into the Great Lakes by a foreign cargo ship. They became a serious problem because they attached themselves to smooth, hard surfaces, and often clogged water intake pipes. Congress determined that zebra mussels posed a great threat to the economic welfare of the Great Lakes region and passed a statute requiring all Great Lakes water intakes to be coated with a special chemical compound that repels zebra mussels. Studies by biologists at a major state university showed that while the special chemical compound that the federal government has required was effective, it also was toxic to other aquatic life. The biologists recommended that Great Lakes intake pipes be coated with a less toxic and less expensive copper-based paint. On the basis of those studies and the recommendation, three Great Lakes states adopted laws permitting municipal water districts to coat their intake pipes with copper paint. Can municipalities using copper-based paint on their intake pipes successfully be prosecuted for violating the federal law? A) No, because the Tenth Amendment prevents Congress from interfering with integral government functions. B) No, because the municipalities are taking effective steps to combat zebra mussels in compliance with the spirit and purpose of the federal law. C) Yes, because Congress is in a better position to regulate the entire Great Lakes region than the individual states. D) Yes, because Congress may adopt laws regulating navigable waters.

(D) is correct. The cities can be prosecuted because state or local government action that conflicts with valid federal laws is invalid under the Supremacy Clause. The federal law here could be found valid as an exercise of the commerce power (Congress can regulate any activity that either in itself or in combination with other activities has a substantial economic effect on interstate commerce) or under the admiralty power (Congress can regulate all navigable waterways). The action of the municipalities directly conflicts with the directives of the federal law and can therefore be stopped. (B) is incorrect because the fact that the copper paint may be as effective as the special compound does not change the result. The action by the municipalities can be prohibited under the Supremacy Clause. (A) is incorrect because for regulations that apply to both the public sector and the private sector, the Supreme Court has held that states' Tenth Amendment rights are best protected by the states' representation in Congress; hence, the Tenth Amendment is not a likely ground for striking this federal legislation because it is not directed only at state or local governments. (C) is incorrect because it is irrelevant; the federal law is superior to the states' laws because it is within Congress's power, not because Congress is in a better position than the states to adopt the legislation involved.

A city has many parks, as well as a country club with a golf course. While the city's parks are accessible to all without a fee, the city charges a $1,000 application fee and $100 per month dues to belong to the country club. A resident of the city wishes to join the country club but cannot afford the application fee or monthly dues. If the resident brings suit against the city on the ground that the fee and dues discriminate against the poor in violation of the Equal Protection Clause, who likely will prevail? A) The resident, because a person cannot be deprived of a public right or benefit on the basis of inability to pay. B) The resident, because the poor qualify as a protected class. C) The city, because only de jure discrimination against the poor has been held to violate the Equal Protection Clause. D) The city, because the membership privilege is not an important enough deprivation.

(D) is correct. The city will prevail because the membership privilege is not a significant enough deprivation to implicate the Equal Protection Clause. Only the denial of particularly important rights (such as a marriage license) to those unable to pay for them has been held to violate equal protection. Therefore, (A) is wrong. (C) is wrong because a number of de facto discriminations against the poor have been held to violate equal protection. (B) is an incorrect statement of the law. The Supreme Court has never held that wealth alone is a suspect classification. Only when the lack of wealth prevents a person from exercising a fundamental constitutional right will equal protection issues be raised.

A city condemned all of the buildings in a decaying warehouse district and offered them to developers at no cost, provided the developer submits a building revitalization plan to the city's planning commission, the commission approves the plan, and the developer pays the city's redevelopment authority to perform the work needed to revitalize the building. An experienced developer submitted a plan to revitalize a block-long building, and his plan was approved by the planning commission. However, the developer wanted to perform the revitalization work himself, both to ensure that the job was done correctly and to save a substantial amount of money. Although he offered to post a surety bond to insure his performance, the city refused his offer. As applied to the contractor, is the city's requirement that the developer use the city's redevelopment authority constitutional? A) No, because it violates the privileges and immunities protected by Article IV. B) No, because it constitutes an improper exaction under the Fifth Amendment. C) Yes, because the city is seeking just compensation for the building that it would provide to the developer. D) Yes, because the requirement is rationally related to a legitimate government interest.

(D) is correct. The city's requirement that the developer use the city's redevelopment authority is constitutional under the rational basis standard. When a city acts and the actions do not affect a fundamental right or involve a suspect or quasi-suspect classification, the action will be upheld unless the challenging party can prove that the action is not rationally related to a legitimate government interest. The city's desire to use its own work crews to do all redevelopment work in the warehouse district is rationally related to many possible legitimate government interests; e.g., ensuring consistency and quality, providing work for government employees, and even making a profit or recouping some of the funds that the city expended to condemn the buildings. Moreover, the city does not classify developers, treating some differently from others, so no suspect or quasi-suspect class is involved, and as will be explained below, no fundamental right is involved either. (A) is incorrect because the Privileges and Immunities Clause is not applicable here. The Article IV Privileges and Immunities Clause prohibits states from discriminating against nonresidents regarding "fundamental" rights, and the right to remodel a particular building using the labor force of one's choice simply is not a fundamental right. (B) is incorrect. It is based on the Taking Clause of the Fifth Amendment. That clause prohibits government from taking private property without just compensation. It has been held that under the clause, government may not condition a building permit on a landowner's conveying title to part of his land to the government or granting the public access to the property unless the government can show that the condition relates to a legitimate government interest and the adverse impact of the proposed development is roughly proportional to the loss caused to the owner by the forced transfer. Here, the contractor does not own the building—the city is willing to give it to the builder only if certain conditions are met. Thus, the conditions do not amount to an exaction. (C) is incorrect. It also is based on the Taking Clause. That clause is not a source of power. Thus, it cannot be a constitutional basis for upholding the city's requirement.

To reduce incidents of violence among male gang members at a youth center, a city passed an ordinance forbidding any male between the ages of 13 and 19 to enter the center unless accompanied by a female. An 18-year-old male who was refused admission at the center because he was not escorted by a female filed suit in federal court to strike down the ordinance as unconstitutional. Should the court find the city ordinance constitutional? A) Yes, because in these situations the government is performing a parens patriae function. B) Yes, if the city can show a rational relationship between the ordinance and maintaining order at the center. C) No, if the plaintiff can show that the ordinance was not necessary to promote a compelling government interest. D) No, unless the city can show that the ordinance is substantially related to important government objectives.

(D) is correct. The court should find the ordinance unconstitutional unless the city shows that the ordinance is substantially related to important government objectives. Classifications based on gender, such as the ordinance here, are quasi-suspect and violate equal protection unless they are substantially related to an important government objective. (A) is incorrect; even if the government were performing a parens patriae function, it would not be permitted to violate equal protection. (B) states the wrong equal protection test; because a quasi-suspect class is involved, the rational relationship test does not apply. (C) states the wrong test and places the burden of proof on the wrong party.

Due to budget shortages and a critical need of funding to fight a war, Congress enacted a $25 tax on each person flying into an airport in the five most popular vacation destinations in the country, as determined by Congress. The tax was implemented, and officials in the five destinations were outraged, fearing that the number of vacationers to the taxed destinations would decrease due to the tax. If the tax is challenged in federal court by an official with standing, is the most likely result that the tax will be held constitutional? A) No, because it makes it significantly more difficult for persons to travel between the states. B) No, because the tax unfairly discriminates against certain vacation destinations by taxing them and not taxing other, similar vacation destinations. C) Yes, because the tax is necessary to achieve a compelling government interest. D) Yes, because Congress has plenary power to impose taxes to raise revenue.

(D) is correct. The destination tax will likely be held constitutional under Congress's taxing power. Congress has the power to lay taxes under Article I, Section 8, and a tax measure will usually be upheld if it bears some reasonable relationship to revenue production or if Congress has the power to regulate the taxed activity. Despite the protest from the officials of the affected locations, the tax here does appear to be related to revenue production and so will be upheld. (C) is incorrect because it is based on the wrong standard—the compelling interest test does not apply here. (A) is incorrect because the extent of the right to travel is not clearly defined. The Supreme Court has established that the right to travel from state to state is a fundamental right that may be violated by state laws designed to deter persons from moving into a state; however, the Court has not specifically applied this rule to the federal government or to the type of tax legislation present here. The state cases involved treating old vs. new residents differently for purposes of voting or some government benefit, which may have violated the Privileges or Immunities Clause of the Fourteenth Amendment, or treating outsiders differently from residents, which may have violated the Equal Protection Clause of the Fourteenth Amendment. Neither clause is applicable to the federal government, so neither analysis is appropriate. (B) also is incorrect. While the federal government is not subject to the Equal Protection Clause of the Fourteenth Amendment, it is prohibited from unfair discrimination by the Due Process Clause of the Fifth Amendment. Grossly unreasonable discrimination by the federal government could be held to violate due process, but a $25 tax on flights to popular vacation destinations probably does not. The tax is rationally related to the legitimate government interest of revenue production and so will likely be upheld.

A state's highway speed limits were 65 miles per hour in its flat land regions and 55 miles per hour its mountainous regions. To reduce traffic fatalities and combat the fact that most of the vehicles on state highways were exceeding posted speed limits, the state legislature proposed banning the use of radar detectors. Citizens in the mountainous regions of the state, where most of the state's highway fatalities occurred, generally supported the ban, but citizens in the flat regions of the state opposed the ban, so the legislature adopted a law banning use of radar detectors on any road with a speed limit below 60 miles per hour. A driver whose car was equipped with a radar detector lived in the mountainous region of the state but frequently drove to the state's flat region. While on a mountain highway with a posted speed limit of 55 miles per hour, the driver was pulled over by a state trooper for speeding. While approaching the driver's car, the state trooper noticed that the driver's radar detector was turned on. The trooper ticketed the driver for both speeding and illegal use of a radar detector. The driver challenges his ticket for use of the radar detector, arguing that it is unfair to allow people in the flat lands to use radar detectors while prohibiting residents of the mountainous region from using them. Which of the following statements is correct regarding the burden of proof in such a case? A) The state will have to prove that the ban serves a compelling state interest. B) The state will have to prove that the ban is rationally related to a legitimate state interest. C) The driver will have to prove that the ban does not serve a compelling interest. D) The driver will have to prove that the ban is not rationally related to a legitimate state interest.

(D) is correct. The driver will have to prove that the ban is not rationally related to a legitimate state interest. Whether the law here is examined under the substantive provisions of the Due Process Clause or the Equal Protection Clause, the analysis is the same: If no fundamental right or suspect or quasi-suspect class is involved, the law will be assessed under the rational basis standard. Under that standard, government action will be upheld unless a challenger can prove that it is not rationally related to a legitimate government interest. Here, the right involved (using a radar detector) is not fundamental, and no suspect or quasi-suspect class is involved. Thus, the regulation will be assessed under the rational basis standard. (A) is incorrect because, as explained above, the compelling interest standard does not apply when neither a fundamental right nor a suspect classification is involved. (B) is incorrect because, although it states the correct standard, it placed the burden of proof on the wrong party. The challenger must prove that the law is irrational; the government need not prove that it is rational. (C) is incorrect because it states the wrong standard (compelling interest). Moreover, if the compelling interest test were applicable, the driver would not have to prove that the standard was met; the burden would be on the government.

A federal statute provides for subsidized health care benefits for all full-time college students who are also unemployed. The benefits terminate automatically if the recipient fails to enroll in college full-time in any two consecutive semesters or takes a job working more than 15 hours per week. A college student who had previously received the benefits was denied coverage when he reported that he had taken a full-time student teaching job his senior year, a requirement for his degree. The student did not receive health care benefits through his employer and was left with no coverage. The student filed suit in federal court to compel the government to continue providing him with the subsidized benefits while he finished his degree. He asserts, among other things, that there is no logical reason why he should be deprived of benefits when students in other fields who are not required to work as part of their degree program continue to receive them. Which of the following is the strongest ground for the government to defend the constitutionality of the federal statute in question? A) Congress's power over federal expenditures is plenary, and Congress has discretion to spend money from the federal treasury in whatever way it wishes. B) Because there is no constitutional entitlement to health care benefits, Congress is free to condition them as it sees fit. C) The classification imposed by this statute is substantially related to the important government interest of saving money. D) Congress could rationally conclude that an unemployed college student is less likely to be covered under an employer's coverage and thus more needy than an employed student.

(D) is correct. The federal statute will be upheld if Congress had a rational basis for making the classifications that it did. Governmental actions involving classifications of persons are examined under one of three standards, regardless of whether it is a federal government action scrutinized under the Fifth Amendment Due Process Clause or a state government action reviewed under the Fourteenth Amendment Equal Protection Clause. If a suspect classification or fundamental right is involved, the strict scrutiny standard will be applied and the action will be struck down unless the government proves that it is necessary to achieve a compelling interest. If a quasi-suspect classification is involved, the Court will likely require the government to prove that the action is substantially related to an important government interest. If any other classification is involved, the action will be upheld unless the challenger proves that the action is not rationally related to a legitimate government interest. Here, the classification does not involve either suspect or quasi-suspect classifications. Thus, the rational basis standard will apply to the classifications; the statute will be upheld if Congress could rationally conclude that an unemployed, full-time college student is more likely to be needy than one who is employed and thus currently earning some income and may be offered employer coverage. (A) is incorrect because Congress's power to spend money to "provide for the common defense and the general welfare" is not unfettered. The Fifth Amendment Due Process Clause and other constitutional restrictions on the exercise of federal government power apply to congressional expenditures. (B) is incorrect because Congress cannot condition the receipt of these benefits on some ground that itself constitutes a violation of the Bill of Rights. (C) is incorrect because it employs the wrong test (intermediate scrutiny). Moreover, saving money is not likely to be found a sufficiently important interest to justify government discrimination under the intermediate scrutiny test. (D) offers the best justification for upholding the classification under the rational basis standard.

Congress has enacted a law providing that all disagreements between the United States and a state over federal grant-in-aid funds shall be settled by the filing of a suit in the federal district court in the affected state. The law further provides: "The judgment of the federal court shall be transmitted to the head of the federal agency dispensing such funds, who, if satisfied that the judgment is fair and lawful, shall execute the judgment according to its terms." Is this law constitutional? A) Yes, because disagreements over federal grant-in-aid funds necessarily involve federal questions within the judicial power of the United States. B) Yes, because the spending of federal monies necessarily includes the authority to provide for the effective settlement of disputes involving them. C) No, because it vests authority in the federal court to determine a matter prohibited to it by the Eleventh Amendment. D) No, because it vests authority in a federal court to render an advisory opinion.

(D) is correct. The law is unconstitutional because it gives the federal district courts authority to render advisory opinions. The Supreme Court has interpreted the constitutional power of the federal courts to hear "cases and controversies" to mean that federal courts may not render advisory opinions. Under the law here, the district court's decision is not binding on an agency dispensing funds, because the agency head is given discretion to decide whether the court's judgment is fair. Therefore, the judgment is merely advisory and so is not within the jurisdiction of the federal courts. (A) is incorrect because while it is true that the cases would present a federal question, it is not enough merely that a federal question exists; there must be a case or controversy. (B) is incorrect for the same reason. Congress certainly has authority to provide for the settlement of disputes under federal grant programs, but that authority does not permit Congress to violate the case or controversy requirement. (C) is incorrect because the Eleventh Amendment only prohibits the federal courts from hearing an action by a citizen of a state against his state; actions between the federal government and a state are not barred.

To prevent the Supreme Court from whittling away the protections that previous Supreme Court decisions had created for individuals accused of crimes, Congress passed a law eliminating from Supreme Court jurisdiction all cases in which a state supreme court has decided that a defendant's federal constitutional rights have been violated. If the statute is held unconstitutional, what is the most likely reason? A) The determination of the extent of constitutional rights is precisely the domain of the Supreme Court. B) Congress's power to limit jurisdiction applies only to cases originating in the federal courts. C) To be effective, the action taken by Congress here would require a constitutional amendment. D) Congress may not eliminate all avenues for Supreme Court review of issues vested within the judicial power of the federal courts.

(D) is correct. The most likely ground for holding the statute unconstitutional is that Congress may not limit all avenues for Supreme Court review of federal constitutional issues. Article III, Section 2 provides that the Supreme Court shall have appellate jurisdiction under such regulations as the Congress shall make. Although Ex parte McCardle (1868) gives Congress broad power to regulate the Supreme Court's appellate jurisdiction, it has been suggested that Congress may not eliminate all avenues for Supreme Court review of issues vested within the federal judicial power. This argument is the most likely basis for finding the law here unconstitutional because none of the other choices is viable. (A) is not the best argument. As indicated above, Congress has the power to regulate the appellate jurisdiction of the Supreme Court, and since the Supreme Court's primary role as an appellate court is to determine constitutional issues, it follows that Congress can eliminate from the Supreme Court's review the determination of certain cases involving constitutional issues, as long as jurisdiction remains in some lower federal courts. (B) is incorrect because Congress's power is limited to regulating the Supreme Court's appellate jurisdiction; it has no power to restrict or enlarge the Court's original jurisdiction. (C) is incorrect because Congress has the authority to adopt laws modifying appellate jurisdiction; modification of the Court's original jurisdiction would require a constitutional amendment.

A city passed an ordinance prohibiting all of its police officers and firefighters from "moonlighting" (working a second job). The ordinance was passed to ensure that all police officers and firefighters were readily available in case an emergency should arise and for overtime work when the situation warranted it. Other city employees, including members of the city council and the city manager, had no such restrictions placed on secondary employment. A police officer who wanted to moonlight as a dancer at a nightclub within city limits brought suit in federal court, alleging that the ordinance violated her constitutional rights. Will the court likely find the ordinance constitutional as applied to the police officer? A) No, because the ordinance restricts the officer's First Amendment rights to freedom of expression. B) No, because the singling out of police officers and firefighters violates equal protection. C) Yes, because the city has a significant interest that it seeks to regulate. D) Yes, because there is a rational basis for the ordinance.

(D) is correct. The ordinance is constitutional under the rational basis standard. Because the ordinance is not related to the exercise of a fundamental right or based on a suspect trait, it need only rationally relate to some legitimate governmental interest. Under the Equal Protection Clause, which is implicated because the ordinance treats some city employees differently from others, a governmental classification must be necessary to promote a compelling state interest when it relates to who may exercise a fundamental right or when it is based on a suspect trait (e.g., race or national origin). If a quasi-suspect classification (e.g., gender or legitimacy) is involved, the classification will be upheld if it is substantially related to an important government interest. In all other cases, the classification is valid if there is any conceivable basis on which it might relate to any legitimate governmental interest. This "rational basis" test is used for all classifications that relate only to matters of economics or social welfare. The right of police officers and firefighters to hold second jobs is not a fundamental right that will trigger strict scrutiny. In addition, the ordinance is not based on a suspect or quasi-suspect classification. Therefore, the validity of the ordinance is judged according to the "rational basis" test. A party attacking a classification under this test bears the difficult burden of demonstrating to the court that the classification does not have a rational relationship to a legitimate interest of government. The ordinance is intended to and does in fact promote the legitimate governmental interests of public safety and social welfare—it increases the likelihood that there will be sufficient numbers of police and firefighters to deal with emergencies. Thus, the ordinance is valid. (C) is incorrect because, as detailed above, the test applicable here is the "rational basis" test, which requires only that the city have a "legitimate" interest rather than a "significant" interest. (B) is incorrect. Although the ordinance treats police officers and firefighters differently from other city employees, this treatment, as discussed above, is rationally related to legitimate governmental interests. Regarding (A), it is doubtful from the facts that the police officer's job as a dancer was a means of exercising the freedom of expression. However, even if the officer were "expressing" herself by means of this job, the ordinance's content-neutral restriction on this freedom is only incidental, is in furtherance of the governmental interests of public safety and social welfare, and is narrowly tailored to the furtherance of those interests. Note also that the city is not prohibiting the officer from expressing herself by means of the dance art form as such, but is only prohibiting her from holding a job in addition to that of police officer. Presumably, the officer is free to join a dance troupe or otherwise engage in dance, as long as such activities do not involve her being employed outside the police force. For these reasons, (A) is incorrect.

Under a state's law, public schools in each school district are funded by a percentage of the property taxes collected in that district. The system was adopted because it allows communities to decide how much to invest in education, bringing government decision-making to a local level. Several school districts in the state are comprised almost entirely of low-income minority residents, and property tax revenue in those districts is much lower than in wealthier districts. As a result, funding for public schools is much lower in those districts. The parent of a child who attends school in low-income district brought an action against state officials seeking to enjoin the state school-funding law and force state-wide equitable funding of public schools, arguing the current funding system violates the Equal Protection Clause of the Fourteenth Amendment. Should the court uphold the state school funding law? A) Yes, because the law is substantially related to an important state interest. B) Yes, because the law is rationally related to a legitimate state interest. C) No, because the law is not necessary to achieve a compelling interest. D) No, because the system discriminates based on a suspect classification.

Choice (B) is correct. The court should uphold the law because it is rationally related to a legitimate state interest. The court will apply one of three tests when an equal protection violation is alleged. When a fundamental right or a suspect classification is involved, a court will apply strict scrutiny (law must be necessary to achieve a compelling interest); when sex discrimination is involved, the court will apply intermediate scrutiny (law must be substantially related to an important interest); and in all other cases, the court will apply a rational basis test (law must be rationally related to a legitimate government interest). Where a government classification is neutral on its face but unintentionally causes a discriminatory effect, heightened scrutiny is not required, and the law need only satisfy the rational basis standard. On its face, the law here does not create any classifications; rather, the allegation is that the law impacts minority residents. This is not sufficient to require strict scrutiny. Hence, the rational basis test will apply. Giving communities a voice in how much to spend on education is a legitimate government interest and the funding law is rationally related to that interest. Therefore, it will be upheld. Choice (A) is incorrect because it reflects the intermediate scrutiny test. As explained above, that test is inapplicable here because no sex discrimination is alleged. Choice (C) is incorrect—since strict scrutiny does not apply, it does not matter that the law is not necessary to a compelling government interest. Choice (D) is incorrect because, as stated above, it is not enough that the law might have a disparate impact on minorities; there is no suspect classification or intent to discriminate under the facts.

To protect an endangered species of salamander, Congress passed a civil statute prohibiting hiking and fishing in wetland areas located in a particular national park. Violators could be enjoined from use of the national park for one year. The law was to take effect on January 1 of the new year. After the law went into effect, a local resident was stopped by a National Park Service ranger and given a $100 citation for fishing in the wetland areas. As authorized by the statute, a federal district court also issued an injunction prohibiting the resident from fishing in the wetlands area for one year. The resident appealed the injunction, asserting that he was unaware of the penalty, but he was unsuccessful. A final judgment was rendered against him. During the next few months, several hundred violations of the statute occurred, and the population of endangered salamanders continued to decline. To better achieve its conservation goal, Congress voted to amend the statute. The new law expanded the time an injunction could prohibit use of national parks to two years, made such injunctions applicable to all national parks, and imposed a $1,000 fine for all violations. The new law was made effective retroactively to the same January 1 date. As a result of the new law, the resident was notified that the injunction against him would be extended to a total of two years and that he owed an additional $900 fine. The resident's attorney brought an action claiming that the new law was unconstitutional as applied. Will the resident's claim be successful? A) Yes, because the new law proscribed greater punishment for the violation than was prescribed at the time the violation was committed. B) Yes, because a final judgment was rendered against him on appeal of the district court's injunction. C) No, because Congress has the power to make substantive changes to federal civil statutes and direct the federal courts to apply those changes. D) No, because the Ex Post Facto Clause under Article I, Section 10 applies to the states, not to the federal government.

Choice (B) is correct. The resident's claim will be successful because a final judgment was rendered against him on appeal of the district court's injunction. Although Congress has the power to make substantive changes to civil statutes and to direct that those changes be applied to the courts to all actions, that principle does not apply where a final judgment has been rendered. Since the resident's injunction was upheld by the appeals court and a final judgment was issued, the new law cannot extend the injunction against the resident to two years and cannot increase the fine imposed. For this reason, Choice (C) is incorrect. An ex post facto law is legislation that retroactively alters the criminal law in a substantially prejudicial manner so as to deprive persons of any rights previously enjoyed for the purpose of punishing them for some past activity. However, if the law's purpose is civil, rather than punitive, it is not an ex post facto law, unless its effect is so clearly punitive as to negate the legislature's intention. In this question, the legislature's intention was not punitive. Rather, it was to protect an endangered species of salamander. The law was intended by Congress to better achieve this conservation goal. Therefore, Choice (A) is incorrect. Finally, Choice (D) is incorrect. Although it is a true statement, it reaches the wrong conclusion. There are two Ex Post Facto Clauses: Article I, Section 10, Clause 1 applies to the states, whereas, Article I, Section 9, Clause 3 applies to the federal government. Neither clause is applicable here because the amended law was not criminal and because a final judgment on the injunction had been rendered.

Congress authorized $20 million to be granted to each of five southern states that had reported outbreaks of a harmful mosquito-borne virus. The states were required to use the money for both ground and aerial spraying in an attempt to eradicate the mosquito larva. The law provided that all federal environmental protection funding to the states would be contingent on the states implementing the mosquito spraying program. One of the states, where federal environmental protection funding constituted about 1% of the state's budget, determined that spraying swampland with the chemicals could have a wide range of unintended consequences, and the state refused to implement the program. As a result, the federal government terminated all of the state's environmental protection funding. The state then filed an action to force the federal government to continue to provide funding for the state's other environmental protection projects unrelated to the mosquito spraying program. Who will most likely prevail? A) The state, because Congress may not use its spending power to coerce states into implementing federal programs. B) The state, because controlling mosquito populations is not a regulation of interstate commerce. C) The federal government, because depriving a state of federal funding amounting to 1% of the state's budget is not unduly coercive. D) The federal government, because controlling mosquito populations in multiple states is a valid regulation of interstate commerce.

Choice (C) is correct. Congress may "regulate" states through its spending power by imposing conditions on the grant of money to state governments. These conditions will not violate the Tenth Amendment merely because Congress lacked the power to directly regulate the activity that is the subject of the spending program if the conditions are clearly stated, relate to the purpose of the program, and are not unduly coercive. Therefore, the answer turns on whether the condition on spending is unduly coercive, and not whether Congress has the power to legislate in this area under the Commerce Clause, so (B) and (D) are wrong. (A) is wrong because a spending condition that would deprive a state of 1% of its budget is likely not unduly coercive. The Supreme Court has upheld a spending condition that deprived a state of one half of one percent of its budget and has rejected a condition that deprived a state of 10% of its budget.

In exchange for an agreement by a foreign nation to stop all testing and development of its nuclear arsenal, Congress passed a bill authorizing $100 million to enable students from that country to attend colleges and universities in the United States to obtain higher education. Two weeks after the measure was passed, human rights violations were committed in the foreign nation, during which a government-sponsored coalition used chemical weapons to kill hundreds of women and children. Immediately, the President ordered troops to be sent to the foreign country and he canceled the appropriation of the educational funds. Is the President's action to cancel the higher education measure constitutional? A) Yes, because the President has exclusive power over the area of foreign affairs. B) Yes, because of the President's broad emergency powers as commander-in-chief of the military. C) No, because of the President's obligation to take care that the laws are faithfully executed. D) No, because the higher education measure derives from Congress' power to regulate commerce with foreign nations.

Choice (C) is correct. The President's action to cancel the educational funding measure is unconstitutional because the President, under Article 2, Section 3, has an obligation to take care that the laws are faithfully executed. The President may not unilaterally cancel the legislation. Neither the President's foreign affairs powers—Choice (A), nor the President's power as commander-in-chief—Choice (B), justify the termination of an appropriations measure passed under Congress' spending power. Where Congress passes a measure to appropriate federal funding, the taxing and spending power is implicated rather than the federal commerce power. Therefore, Choice (D) is incorrect.

An expert in art history had worked at a county art museum for many years as its executive curator. The museum had a policy that once an employee worked at the museum for five years, the employee would not be dismissed from the job unless "for cause." The county decided to sell the museum to a private corporation which promised to double the size of the museum and greatly increase its funding. Two months after the museum's ownership changed hands, the curator was informed that he was being dismissed and would be replaced by an individual chosen by the corporation. The curator brought an action against the corporation in state court, claiming that termination of his position without the opportunity for a hearing violated his rights under the Due Process Clause of the Fourteenth Amendment and the Contract Clause. For which party should the court rule? A) The curator, because the Due Process Clause entitles him to an opportunity for a hearing before dismissing him. B) The curator, because dismissing him violated the Contract Clause. C) The corporation, because the curator's dismissal does not implicate either the Fourteenth Amendment or the Contract Clause. D) The corporation, because the right to pursue a livelihood is not a protected liberty interest.

Choice (C) is correct. The court should resolve the issue in favor of the corporation because the curator's termination does not implicate the Fourteenth Amendment or the Contract Clause. Where there is a legitimate claim or entitlement to a government benefit under state law, when that benefit is terminated, the Due Process Clause requires some type of notice to be given and an opportunity to be heard (either pre-or post-termination, depending on the importance of the interest involved). However, the Due Process Clause applies only to state action or action undertaken by private entities but that can be attributed to the state. Here, the curator was fired by a private corporation. The mere fact that the private corporation took over a business that was formerly run by the county is not sufficient to make the corporation's action state action. Moreover, the Contract Clause was not violated here. It applies only to state laws that retroactively impair contract rights. Here, there is no new law. The county sold the museum to a private corporation, and the private corporation made a personnel decision. Therefore, Fourteenth Amendment and Contract Clause protections are not applicable here and choice (A) and (B) are incorrect. Choice (D) is incorrect because it is irrelevant. Again, even if the curator has a protected liberty interest in pursuing a livelihood, it would not apply against a private corporation.

A man had been a lawful resident alien for several years. He was browsing an online website for government employment opportunities and saw an opening for a position as a state highway patrol officer. He decided to apply for the position and was granted an interview. At the interview, the man was informed that although his application had complied with all the requirements necessary for the position, he was disqualified from service solely because he was not a U.S. citizen. The man brought an action for declaratory relief, claiming that the state has violated his rights under the Fourteenth Amendment. Should the man be successful in his action? A) Yes, because the state has not shown the classification restricting aliens is necessary to achieve a compelling interest. B) Yes, because the man's application complied with all the requirements necessary for the position. C) No, because serving as a highway patrol officer is integral to the self-government process. D) No, because the state may discriminate against aliens where the justification is rationally related to a legitimate interest.

Choice (C) is correct. The man will not be successful because serving as a highway patrol officer is an integral government function. As a general rule, state laws that discriminate against aliens are subject to the strict scrutiny standard of review. However, for positions deemed to be integral to self-governance—such as serving on a jury, teaching at a public grade school or high school, or employment as a police officer—a state may limit participation to U.S. citizens only, provided the discrimination is rationally related to a legitimate state interest. For this reason, Choices (A) and (B) are incorrect. Choice (D) is incorrect because, as discussed above, rational basis only applies to certain positions.

A large parcel of privately owned land was located next to a river in an area occupied by only a few residences. These homes were largely run-down and had little economic value. The city decided to undertake an economic development plan by using its eminent domain powers to purchase the land from the homeowners, then sell it to a wealthy private development corporation. The developer planned to undertake a comprehensive redevelopment project to benefit the city. The purpose was to create new jobs, increase tax revenues, and revitalize what had been a depressed area of urban blight. One of the homeowners refused to give up her home, despite an offer of compensation from the city. The city instituted condemnation proceedings against the homeowner to take the homeowner's land in exchange for its fair market value. At trial, the homeowner claimed the forced sale was an unconstitutional taking. For which party should the court rule? A) For the homeowner, because the redevelopment plan violated the public use requirement of the Fifth Amendment. B) For the homeowner, because the Takings Clause applies to states and the federal government, not to cities. C) For the city, because the taking is necessary to achieve a compelling government purpose. D) For the city, because taking property from one private party to give it to another private party can be a "public use."

Choice (D) is correct. The court should rule for the city because taking property from a private party to give it to another private party can be a public use. The Takings Clause provides that private property shall not be taken for public use without just compensation. This rule applies to the actions of federal, state, and local governments, therefore (B) is wrong. The Supreme Court has held that the governmental taking of property from one private owner to give to another, in furtherance of economic development, constitutes a permissible "public use." Therefore, Choice (A) is incorrect. Choice (C) is incorrect because it states the wrong standard of review. Strict scrutiny is not required to show a "public use."

Thirty years ago, a man obtained a certification from the Federal Housing Administration ("FHA") as a certified real estate appraiser. At the time, few organizations hiring appraisers required certifications. For the past thirty years, the man has worked for State A, appraising properties for tax purposes. He has a contract providing that he can be dismissed only for cause. Six months ago, State A passed a law requiring anyone appraising real estate in State A to hold a State A license as a real estate appraiser. The man, who is 60-years-old and plans to retire in a few years, refuses to spend the time and money necessary to get the State A license. After a hearing, during which the man is given an opportunity to present his case for retaining his position, he is fired, and a much younger employee who has the State A license is hired to replace him. If the man challenges his dismissal in court, who will likely prevail? A) The man, because the state law is preempted. B) The man, because he was deprived of his rights under the Due Process Clause. C) The man, because he was deprived of his rights under the Equal Protection Clause. D) State A.

Choice (D) is correct. The man will not be successful because the adverse hiring decision did not violate any of the man's constitutional rights. Choice (A) is wrong because the state law is not preempted. A state law is preempted by a federal law on the same subject if it conflicts with the federal law, it prevents achievement of the federal objectives, if Congress has preempted the entire field, or if the federal law expressly indicates state laws on the same subject are preempted. None of those scenarios are true here. Indeed, all we know about the federal law is that it allowed the FHA to certify real estate appraisers. Choice (B) is wrong because the man was given procedural due process when he received a hearing. Choice (C) is wrong because if there is an issue under the Equal Protection Clause, it is based on age discrimination (under a speculative argument that the new license requirement was imposed to drive older workers out of the state's employment). But even assuming facts to support that argument, since age is not a suspect classification, the law is valid unless the challenger can show there is no rational basis for the government's action, which the man cannot do since the licensing requirement has an apparent common sense relationship to accuracy in appraisals.

A contractor employed by the state was owed $10,000 in back wages by a real estate developer for remodeling work he had completed six months earlier. Pursuant to a state law regulating delinquent payment of wages, the contractor filed an action in state court to resolve the claim within 21 days. In response, the real estate developer filed a voluntary bankruptcy petition in federal bankruptcy court and sought a stay of further proceedings on the unpaid claim. Federal law provided for an automatic stay in both federal and state courts for persons filing federal bankruptcy claims. If the contractor opposes imposition of the stay as applied to his state court claim, what is the proper resolution of the issue? A) The stay cannot be applied in the case, because the state court claim was already filed at the time of the federal bankruptcy petition. B) The stay cannot be applied in the case, because state law requires claims dealing with delinquent payment of wages to be adjudicated by state substantive law. C) The stay can be applied in the case, because Article I gives Congress power to regulate the jurisdiction and procedures of both federal and state courts. D) The stay can be applied in the case, because the Supremacy Clause gives Congress the power to supersede state law with proper legislation.

Choice (D) is the correct answer. The stay can be justified under the Supremacy Clause and under the federal bankruptcy power. Choices (A) and (B) are wrong because neither the fact that the contractor is bringing a claim based on state substantive law, nor the fact that the state employee filed the state court claim first matters, since Congress has the power to regulate bankruptcy, and placing an automatic stay on all pending state court proceedings when a bankruptcy petition is filed is a necessary and proper use of that power. This enumerated power gives Congress the power to provide uniform rules of bankruptcy and provides a valid constitutional basis to support the federal statute as applied to these facts. Furthermore, under the Supremacy Clause of Article VI, the federal bankruptcy power will supersede any inconsistent state laws to the extent that they conflict with federal statutes. Choice (C) is incorrect because the Constitution does not authorize Congress to regulate state courts.

Intending to encourage long-time resident aliens to become American citizens, a state passed a law denying numerous state and municipal jobs to persons who had been resident aliens for longer than 10 years. Those already in the state had to apply for American citizenship within a year after the law took effect. Persons who had acquired resident alien status prior to achieving the age of majority had until age 30 to acquire such status or be automatically disqualified from obtaining such a job. A 40-year-old man who has been a resident alien in the state for 15 years applied for a job as a police emergency response telecommunications expert. He had not filed for citizenship within the one-year grace period. May the state constitutionally rely on the statute to refuse to hire the man? A) Yes, because a police department performs an integral governmental function and the state law does not discriminatorily classify resident aliens by race or ethnicity. B) Yes, because aliens are not entitled to the privileges and immunities of state citizenship. C) No, because the law does not apply equally to all aliens. D) No, because the reasons for application of the law to the man do not appear compelling.

The law probably is unconstitutional as applied to the man in question. An equal protection issue is involved. Under the Equal Protection Clause, state classifications based on alienage are subject to strict scrutiny and so must serve a compelling interest to be constitutional. No compelling purpose seems to be present here. Thus, (D) is correct. (A) is incorrect because, although there is an exception from the strict scrutiny standard where a state or local government discriminates against aliens when hiring persons for jobs involving "self-government" processes, the job here (emergency communications for a police department) is a technical position and probably would not be found to be related to the self-government process; in any event, the statute applies to all positions and not just to jobs involving only the self-government process. (B) is incorrect because, although it is true that aliens are not entitled to the privileges and immunities of state citizenship, the law here is still unconstitutional under the Equal Protection Clause, as discussed above, which applies to aliens. (C) is incorrect because the discrimination would be unconstitutional even if it did apply to all aliens equally, as discussed above.


Set pelajaran terkait

Unit 3.2 cell reproduction Growth and Development Study Guide

View Set

1.1.6 Free market, mixed and command economies

View Set

PSYCH 11B Chapter 8 Review Questions

View Set

Lower Respiratory Conditions(Exam 3)

View Set

8001 מילון קוויזליט

View Set

Current Asset (CA), Long-term Asset (LTA), Current Liability (CL), Long-term Liability (LTL), Stockholders' Equity (SE) or not reported on the balance sheet (N)?

View Set